4 Mens Rea: Required mental states 4 Mens Rea: Required mental states

The "Guilty Mind" Element

Mens rea--Latin for 'guilty mind'--is the traditional common law term for the state of mind that must accompany conduct in order for one to be guilty of a crime. The materials in this section introduce you to the concept of mens rea. Like actus reus, mens rea weaves together ideas about the offender's blameworthiness, the seriousness of the crime, and the punishment that should be imposed. Mental state is critical to determining culpability. It is universal intuition that there is a moral difference between doing something knowingly v. unknowingly, intentionally v. unintentionally, or intentionally-but-carefully v. intentionally-but-negligently or recklessly. Justice Oliver Wendell Holmes expressed this notion in a now-famous aphorism: “Even a dog knows the difference between being kicked and being stumbled over.”

Courts and legislatures in the U.S. (and those in the United Kingdom and other common law countries) continue to use a wide variety of traditional terms developed in the common law to describe required mens rea or mental states for crimes, such as: scienter, intent, purpose, knowledge, malice/maliciously, corrupt/corruptly, and more. But two terms are especially widely used and important--"general intent" and "specific intent." "At common law, crimes generally were classified as requiring either 'general intent' or 'specific intent.'" United States v. Bailey, 444 U.S. 394, 403 (1980). Don't expect clear, consistent definitions of these terms that are easy to understand and apply. "This venerable distinction ... has been the source of a good deal of confusion." Id. This confusion arises in part because courts have been unable to consistently define general and specific intent, which has led some courts discourage trial judges, when instructing juries on mens rea requirements, to “eschew use of difficult legal concepts like 'specific intent' and 'general intent.'" Liparota v. United States, 471 U.S. 419, 433 n.16 (1985). In part because “the mental element in criminal law encompasses more than the two possibilities of 'specific' and 'general' intent." Liparota, at 423 n.5. And in part difficulties arise because legislatures do not employ these terms consistently in criminal codes, or draft criminal statutes carefully with these terms in mind.

Additionally, as we will later see, there are also strict liability offenses: crimes for which the defendant's state of mind is irrelevant. These typically involve acts/harms determined to be particularly injurious to public health and must be deterred/punished regardless of intent. To be convicted under a strict liability statute, the defendant must only have committed a voluntary act (actus reus) that caused the harmful result (causation) the statute was designed to prevent. Some examples of strict liability offenses are traffic violations, statutory rape, and felony murder. (Note that for all these offenses, the actor is knowingly doing something--driving, having sex, committing a non-homicide felony. But no knowledge, intent, or unreasonableness is required as to the resulting strict liability crime.)

The following cases provide an introduction to some of the challenges of defining precisely what those terms mean, or require, in particular criminal statutes. As you read these cases, consider what reasoning courts use to justify their decisions and what these reasons reveal about the evolving concept of blameworthiness. Always pay close attention to how the "guilty" state of mind is framed by the relevant statute(s) and the relationships between various mens rea and the severity of punishment. 

We then turn to a modern reform effort to replace these common law terms with different, perhaps-more-precise culpability terms--those proposed by the American Law Institute's Model Penal Code, which have been adopted in a fair number (roughly half) of state criminal codes and somewhat influential even in jurisdictions that have not formally enacted them. As we will see, the MPC categorizes culpable mental states into four tiers of culpability: purposely (acting with a conscious objective to produce the offense specified in the statute); knowingly (acting while being practically certain of the offending result); recklessly (acting with a conscious disregard for the risk of causing the offending result); and negligently (causing the offending result when the actor should have been aware of the risk). See MPC s 2.02(3).

While many statutes will seem loosely based on the four-tiered MPC framework, you should not assume they are identical. Courts often cite the MPC as persuasive (that is, influential but not formally binding) authority, but it is important to remember that the enacted legislation, and not the MPC, is what controls within any jurisdiction. In other words, even if a statute uses the mens rea term "intentional" in a manner akin to the MPC's definition of "purposeful," one may not necessarily substitute one term for another. As you work through these cases, train yourself to focus squarely on the actual language of a given statute and how a lawyer advocating for either side might interpret it. 

4.1 Common Law Mens Rea 4.1 Common Law Mens Rea

4.1.1 General Intent 4.1.1 General Intent

People v. Stark People v. Stark

General intent

[No. C013808.

Third Dist.

July 18, 1994.]

THE PEOPLE, Plaintiff and Respondent, v. RODNEY ELGIN STARK, Defendant and Appellant.

[Opinion certified for partial publication.*]

*1180Counsel

Cara DeVito, under appointment by the Court of Appeal, for Defendant and Appellant.

Daniel E. Lungren, Attorney General, George Williamson, Chief Assistant Attorney General, Robert A. Anderson, Assistant Attorney General, J. Robert Jibson and Janine R. Busch, Deputy Attorneys General, for Plaintiff and Respondent.

Opinion

PUGLIA, P. J.

— A jury convicted defendant of willful diversion of construction funds (Pen. Code, § 484b) and found the taking exceeded $25,000 (Pen. Code, § 12022.6, subd. (a)). Imposition of sentence was suspended and defendant was granted probation for five years.

*1181In the published part of this opinion we shall reject defendant’s contention that the trial court erred in instructing the jury that willful diversion of construction funds is a general intent crime.1 Finding no error, we shall affirm.

In 1989, Doctors Steven Johnson and Douglas Martin contracted with defendant, doing business as Stark Construction, to build a medical facility for approximately $350,000. The project was begun in October 1989, and as various phases of the project were completed, defendant received partial payment from a construction loan obtained by the doctors. With these payments defendant was to pay the subcontractors and materialmen who had either performed work or provided materials for the job.

By February 15, 1990, defendant had received three draws from the construction loan, totalling about $245,000. On March 1, the doctors received several calls from subcontractors and materialmen informing the doctors they had not been paid. The doctors discussed the matter with defendant and it was agreed the doctors would thereafter write the checks to the subcontractors and materialmen. The building was completed on March 1. On March 16 the doctors issued approximately $70,000 in checks to subcontractors and material suppliers from the fourth draw.

When the doctors continued to receive calls from people who were supposed to have been paid, but had not been paid, from the first three draws, they again confronted defendant. Defendant informed the doctors he had been experiencing financial difficulties on other jobs and some of the money from the medical building project had been spent to defray costs incurred for those jobs. Defendant promised to repay the money as the other jobs progressed.

Months later, when money owed subcontractors and material suppliers still had not been paid, the doctors contacted the Contractor’s State License Board and the district attorney. The doctors estimated defendant’s diversion of the funds cost them approximately $46,000.

Defendant testified, admitting he used the money from the medical building project to defray costs on some of his other jobs. However, he always intended to pay the money back.

I

Penal Code section 484b provides in pertinent part: “Any person who receives money for the purpose of obtaining or paying for services, labor, *1182materials or equipment and willfully fails to apply such money for such purpose by either willfully failing to complete the improvements for which funds were provided or willfully failing to pay for services, labor, materials or equipment provided incident to such construction, and wrongfully diverts the funds to a use other than that for which the funds were received, shall be guilty of a public offense . . . .”

Relying on People v. Dollar (1991) 228 Cal.App.3d 1335 [279 Cal.Rptr. 502], and arguing here as he did in the trial court, defendant contends Penal Code section 484b defines a specific intent crime. Therefore, he continues, the trial court prejudicially erred when it instructed the jury the offense was one requiring only general criminal intent. We conclude Penal Code section 484b defines a general intent crime and that Dollar is inapposite.

In People v. Whitfield (1994) 7 Cal.4th 437 [27 Cal.Rptr.2d 858, 868 P.2d 272], the court reiterated the general rule for differentiating between specific and general criminal intent offenses: “[As a] general rule ‘[w]hen the definition of a crime consists of only the description of a particular act, without reference to intent to do a further act or achieve a future consequence, we ask whether the defendant intended to do the proscribed act. This intention is deemed to be a general criminal intent. When the definition refers to defendant’s intent to do some further act or achieve some additional consequence, the crime is deemed to be one of specific intent.’ ” (At p. 449, quoting People v. Hood (1969) 1 Cal.3d 444, 456-457 [82 Cal.Rptr. 618, 462 P.2d 370].)

Defendant concedes there was a wrongful diversion of the construction funds and that this act requires only general criminal intent, i.e., merely the intent to do the act. He contends there must exist in the mind of the defendant the additional intent either to fail to complete the work or, as in this case, not to pay either material suppliers or subcontractors. Therefore, defendant argues, the offense is one of specific intent.

The only “act” described by Penal Code section 484b is the wrongful diversion, i.e., “a diversion to a use other than bona fide project costs” (People v. Butcher (1986) 185 Cal.App.3d 929, 938 [229 Cal.Rptr. 910]), of funds accepted for one or more of the specific purposes set forth in Penal Code section 484b. Nothing in section 484b suggests that when the defendant wrongfully diverts the funds that he intend to do a further act or to achieve a future consequence. The offense is complete if the wrongful diversion was the cause of failure either to complete the improvement or, as here, to pay for services, labor, materials or equipment. It is immaterial *1183whether defendant intended that there be a failure either to complete the project or to pay subcontractors or material suppliers. Consequently, the offense defined by section 484b is one of general criminal intent, and the trial court did not err in so instructing the jury.

People v. Dollar, supra, does not aid defendant. In that case the accused, who had previously been convicted of committing a lewd and lascivious act upon the victim, was thereafter convicted of threatening her (Pen. Code, § 139) after he had unsuccessfully attempted to grab her and, as she was running away, yelled, “I’ll get you soon, bitch.” (228 Cal.App.3d at p. 1338.) On appeal the accused contended the trial court erred in instructing the jury the offense was a general rather than specific intent crime.

In pertinent part, Penal Code section 139 reads: “(a) Except as provided in Sections 71 and 136.1, any person who has been convicted of any felony offense specified in Section 12021.1 who willfully and maliciously communicates to a witness to, or a victim of, the crime for which the person was convicted, a credible threat to use force or violence upon that person or that person’s immediate family, shall be punished by imprisonment in the county jail not exceeding one year or by imprisonment in the state prison . . . . [¶] . . . d] (c) As used in this section, ‘a credible threat’ is a threat made with the intent and the apparent ability to carry out the threat so as to cause the target of the threat to reasonably fear for Ms or her safety or the safety of Ms or her immediate family.”

In concluding the offense was one requiring a specific intent, the court stated: “A threat as defined in section 139 would be a general intent crime if it is merely the threat itself that is being proscribed by the statute. If section 139 falls witMn the category of general intent crimes, it cannot proscribe additional future goals or consequences beyond the act of making the threat itself. HD ... [¶] Section 139 draws a distinction between an idle threat, a joke or a threat that no one would believe, and a credible threat, wMch the section defines. For a threat to be credible, the criminal actor must have the apparent ability to carry out the threat and must have the additional criminal intent to cause the victim of the tMeat fear [tic] for Mm or herself or for Ms or her immediate family. The communication of the threat itself is the act. The intent to create a future or additional consequence is the intent to cause the victim to feel fear or intimidation.” (228 Cal.App.3d at p. 1341.)

As noted above, in cases arising under section 484b it is immaterial whether at the time of the wrongful diversion the defendant intends or desires that the improvement not be completed or the suppliers and subcontractors not be paid. To violate the statute all that it required is the wrongful *1184diversion of the funds, which means not applying the funds for the purpose for which they were disbursed, and that the diversion be the cause of at least one of the described failures. In contrast, a violation of Penal Code section 139 requires the making of the threat coupled with the intent to cause fear in the target of the threat, thus defining an additional mental state not required by Penal Code section 484b. We conclude Dollar is of no assistance to defendant.

II, in*

The judgment is affirmed.

Sparks, J., and Sims, J., concurred.

People v. Sargent People v. Sargent

[No. S067672.

Feb. 1, 1999.]

THE PEOPLE, Plaintiff and Respondent, v. MICHAEL DANIEL SARGENT, SR., Defendant and Appellant.

*1208Counsel

K. Douglas Cummings, under appointment by the Supreme Court; and Barry A. Zimmerman for Defendant and Appellant.

*1209Daniel E. Lungren, Attorney General, George Williamson, Chief Assistant Attorney General, Robert R. Anderson, Assistant Attorney General, Stan Cross, Cynthia G. Besemer and Janis Shank McLean, Deputy Attorneys General, for Plaintiff and Respondent.

Opinion

BROWN, J.

In this case we determine the required mens rea for a conviction of felony child abuse based on direct infliction of unjustifiable physical pain and mental suffering. (Pen. Code,1 § 273a, former subd. (1), now § 273a, subd. (a) (section 273a(1)).)2 The Court of Appeal concluded criminal negligence is required. We disagree, and therefore reverse its judgment.

*1210I. Facts and Procedural Background

Michael Sargent, Jr. (Michael), defendant’s son, was bom on March 24, 1993, three months premature. He weighed two pounds, four ounces, and spent the first three months of his life in the hospital. By August 19, 1993, he was the equivalent of a four- to six-week-old infant in terms of neck muscle development. On that date, Marysville Fire Department personnel and a paramedic responding to a “911” call found Michael in a deep coma and close to death on defendant’s apartment floor. He was not breathing, had no heartbeat, no eye or motor movement, and no response to verbal stimuli. There was bright red blood coming from his nose and mouth.

Michael was ultimately airlifted to the University of California at Davis Medical Center. While Michael’s subsequent condition was not made clear at trial, it appears he survived and was released from the hospital on September 3, 1993.

Defendant’s explanation of Michael’s injuries varied. On August 19, 1993, defendant told a neighbor, paramedic, and hospital personnel Michael had rolled off the couch and fallen 18 to 20 inches to a carpeted floor. On August 24, 1993, defendant was interviewed by the police. He denied dropping Michael or losing his temper.

On August 25, 1993, defendant was again interviewed by the police. He stated he had dropped Michael while throwing him up in the air to stop him from crying, causing Michael to strike his head on the linoleum kitchen floor. The interviewing officers told defendant a consulting physician had concluded Michael was a victim of shaken baby syndrome. Defendant initially denied Michael was ever shaken. He then changed his story a third time. He said he and Michael’s mother had an argument, and she dropped defendant and Michael off at the house. Defendant carried Michael in his car seat into the house. The infant was crying inconsolably. Defendant started to try to make a bottle for Michael. When Michael continued crying, defendant shook him front to back as Michael sat in his car seat. Defendant then returned to making the bottle. When Michael continued to cry, and refused a pacifier, “that’s when I was shaking him more hard.” Defendant shook Michael four or five times, causing the infant’s head to lose contact with the car seat “a couple of times.” Michael stopped crying. He “had this weird look in his eyes . . . like [he] was going to sleep you know like he was falling or something.” When defendant picked him up, Michael’s back bowed. His eyes closed, and he appeared to have difficulty breathing.

Defendant was charged with felony child abuse (§ 273a(l)), alleged to be a serious felony because it involved the personal infliction of great bodily *1211injury. (§§ 667, subd. (a), 1192.7, subd. (c)(8).) It was further alleged that defendant had suffered a prior serious felony conviction for burglary (§§ 459, 667, subd. (a); 1192.7, subd. (c)(18)), and served two prior prison terms (§ 667.5, subd. (b)).

At trial, Dr. John McCann, a pediatrician and director of the Davis Medical Center’s child protection program, testified as a medical expert on child abuse for the prosecution. When Michael arrived at the Davis Medical Center he was sedated and consequently paralyzed because he appeared to be having seizures. No bruises or other external trauma, except for swelling around the eyes, were observed. A CAT (computerized axial tomography) scan revealed bilateral subdural hematomas, or blood over the surface of Michael’s brain on both sides behind his ears. In addition, his brain was swollen. During his examination of Michael, Dr. McCann observed flame-shaped retinal hemorrhages in both eyes.

Based on all of this information, Dr. McCann opined Michael was a victim of shaken baby syndrome. Seventy to seventy-five percent of the children injured by shaking suffer either severe long-term disabling injury, such as blindness, seizures, and difficulty walking or talking, or death. McCann stated that shaking Michael was a circumstance likely to result in great bodily injury or death. For a fall to have caused the injuries sustained, Michael would have to fall out a second story window, not off the couch or from his father’s arms.

Dr. Todd Brandtman testified as a defense medical expert. He was an emergency room physician who had neither treated nor examined Michael. He did not have expertise in child abuse. He testified Michael’s injuries could have resulted from being dropped, and that photographs of Michael taken two days after he left the hospital showed a swollen spot on the back of his head.

Tapes of defendant’s interviews with the police were played for the jury. Defendant testified his admission of shaking the infant was false and his prior account of dropping him was accurate. He confessed to shaking the infant because that was what the interviewing officers wanted to hear and because the officers and other government agents had told him they wanted to keep the family together and did not want him to get into trouble.

Michael’s mother testified she had seen defendant throwing Michael up in the air on other occasions and had gotten angry at him for doing so. On August 19, 1993, she heard a thump and saw the infant crying, but concluded he had not been injured by his fall and left the apartment. Several *1212other family members and friends testified they saw a bump on the back of Michael’s head at various times following his discharge from the hospital.

The jury was instructed that it could find defendant guilty of violating section 273a(l) based on general criminal intent if it found defendant willfully inflicted unjustifiable physical pain or mental suffering on Michael. In addition, the jury was required to find the infliction occurred under circumstances or conditions likely to produce great bodily harm or death. The jury was further instructed that permitting a child to suffer unjustifiable physical pain or mental suffering, or when having the care or custody of a child permitting the child to be injured, or permitting the child to be placed in a situation that endangers the child’s person or health in violation of section 273a(l) required criminal negligence. Both general criminal intent and criminal negligence were defined.3

The prosecutor argued that the jury could conclude defendant committed child abuse whether it found defendant had shaken Michael with general *1213criminal intent, or dropped Michael with criminal negligence, and that this crime was a felony if it was committed under circumstances or conditions likely to produce great bodily harm or death.

The jury found defendant guilty of a violation of section 273a(l). The verdict did not indicate which branch of section 273a(l) the jury found defendant had violated, that is, whether defendant had directly inflicted the abuse by shaking Michael, or had been criminally negligent in dropping him. In a bifurcated court trial, the enhancement allegations were found true. The court sentenced defendant to the upper term of six years for the felony child abuse count, with a consecutive five-year enhancement.4

The Court of Appeal, with one justice dissenting, concluded criminal negligence must be demonstrated to convict a defendant of infliction of unjustifiable physical pain or mental suffering, and the evidence did not support the inference that defendant was criminally negligent. The court relied on cases involving other branches of section 273a(1) that generally required criminal negligence. It disagreed with People v. Atkins (1975) 53 Cal.App.3d 348 [125 Cal.Rptr. 855] and People v. Wright (1976) 60 Cal.App.3d 6 [131 Cal.Rptr. 311], disapproved on other grounds in People v. Wells (1996) 12 Cal.4th 979, 988 [50 Cal.Rptr.2d 699, 911 P.2d 1374], concluding their interpretation of section 273a(1) “reads out of the statute *1214the element which embodies criminal negligence, the requirement that the culpable conduct occur in circumstances likely to produce great bodily harm or death.”

The court further concluded that “[t]here is nothing in the record from which it can be inferred that the defendant knew or should have known of the risk of great bodily harm or death from shaking the infant, e.g., attendance at a prenatal education session concerning this risk. . . . Nor can we take judicial notice that such a risk is generally known. . . . fl[] . . . The defendant denied knowledge of the syndrome and the record is devoid of evidence showing that he knew or should have known of the syndrome. Accordingly, there is no basis for a finding that he knew or should have known that great bodily injury or death is likely to result from shaking his baby.”

Finally, the court concluded that “defendant was convicted under the second branch of section [273a(l)], the ‘infliction of] . . . unjustifiable physical pain or mental suffering’ upon a child ‘under circumstances or conditions likely to produce great bodily harm or death . . . .’” The prosecution was not predicated on criminal negligence in dropping Michael. While defendant testified “he dropped the child and that is how the child was injured . . . there was no evidence of fracture or bruises which would support that theory and the prosecution introduced extensive evidence to refute the claim. Given that strong evidence supporting the prosecution’s theory of the mechanism of injury, it is entirely unlikely that the conviction was predicated upon dropping the baby.” Rather, “there is far more than a ‘reasonable probability’ that the jury in fact found the defendant guilty solely on the shaken baby syndrome theory. Accordingly, we conclude that the defendant’s felony conviction cannot be affirmed.” The court modified the judgment to a conviction of violation of section 273a, former subdivision (2), a misdemeanor-, and remanded to the trial court for resentencing. The judgment was otherwise affirmed. The Attorney General’s petition for rehearing was denied.

We granted the Attorney General’s petition for review.

II. Discussion

A. Background

1. General Intent and Criminal Negligence

Section 20 provides, “In every crime or public offense there must exist a union, or joint operation of act and intent, or criminal negligence.” *1215(See also § 26, subd. Five [persons incapable of committing crime include those “who committed the act or made the omission charged through misfortune or by accident, when it appears that there was no evil design, intention, or culpable negligence”].) Intent can be either general or specific. “When the definition of a crime consists of only the description of a particular act, without reference to intent to do a further act or achieve a future consequence, we ask whether the defendant intended to do the proscribed act. This intention is deemed to be a general criminal intent. When the definition refers to defendant’s intent to do some further act or achieve some additional consequence, the crime is deemed to be one of specific intent.” (People v. Hood (1969) 1 Cal.3d 444, 456-457 [82 Cal.Rptr. 618, 462 P.2d 370].) General criminal intent thus requires no further mental state beyond willing commission of the act proscribed by law.

Criminal negligence is “ ‘aggravated, culpable, gross, or reckless, that is, . . . such a departure from what would be the conduct of an ordinarily prudent or careful [person] under the same circumstances as to be incompatible with a proper regard for human life . . . .’ ” (People v. Penny (1955) 44 Cal.2d 861, 879 [285 P.2d 926].) “Under the criminal negligence standard, knowledge of the risk is determined by an objective test: ‘[I]f a reasonable person in defendant’s position would have been aware of the risk involved, then defendant is presumed to have had such an awareness.’ ” (Williams v. Garcetti (1993) 5 Cal.4th 561, 574 [20 Cal.Rptr.2d 341, 853 P.2d 507], italics omitted; Walker v. Superior Court (1988) 47 Cal.3d 112, 136 [253 Cal.Rptr. 1, 763 P.2d 852] [“criminal negligence must be evaluated objectively”].) Under section 20, criminal negligence “may be sufficient to make an act a criminal offense, without a criminal intent.” (1 Witkin & Epstein, Cal. Criminal Law (2d ed. 1988) Elements of Crime, § 113, p. 133.)

2. Section 273a( 1)

Section 273a(l) is an omnibus statute that proscribes essentially four branches of conduct. At the time of defendant’s crimes, and in substantively identical form currently, it provided: “Any person who, under circumstances or conditions likely to produce great bodily harm or death, [1] willfully causes or permits any child to suffer, or [2] inflicts thereon unjustifiable physical pain or mental suffering, or [3] having the care or custody of any child, willfully causes or permits the person or health of such child to be injured, or [4] willfully causes or permits such child to be placed in such situation that its person or health is endangered, is punishable by imprisonment in the county jail not exceeding one year, or in the state prison for 2, 4, or 6 years.” (Ibid.)

We have observed that violation of section 273a(l) “can occur in a wide variety of situations: the definition broadly includes both active and *1216passive conduct, i.e., child abuse by direct assault and child endangering by extreme neglect.” (People v. Smith (1984) 35 Cal.3d 798, 806 [201 Cal.Rptr. 311, 678 P.2d 886].) We have also observed, however, that “[t]wo threshold considerations . . . govern all types of conduct prohibited by this law: first, the conduct must be willful; second, it must be committed ‘under circumstances or conditions likely to produce great bodily harm or death.’ (§ 273a, subd. (1).) Absent either of these elements, there can be no violation of the statute.” (Ibid.) Section 273a(l) is “intended to protect a child from an abusive situation in which the probability of serious injury is great.” (People v. Jaramillo (1979) 98 Cal.App.3d 830, 835 [159 Cal.Rptr. 771].) “[T]here is no requirement that the actual result be great bodily injury.” (Ibid.)

We have not previously addressed the question of the appropriate mens rea for direct infliction of abuse cases under section 273a.5 In Walker v. Superior Court, supra, 47 Cal.3d at page 135, we did not dispute the defendant’s broad statement that section 273a requires criminal negligence. However, in Walker, a Christian Science parent declined to obtain medical treatment for her four-year-old child suffering from meningitis, who eventually died. (47 Cal.3d at pp. 118-119.) Hence, that case involved indirect abuse or child endangerment, not direct infliction of abuse. Indeed, in People v. Heitzman (1994) 9 Cal.4th 189 [37 Cal.Rptr.2d 236, 886 P.2d 1229], when interpreting section 368,6 which is virtually identical to section 273a(l), we twice made it clear that the case involved a failure to act, not active abuse. (9 Cal.4th at p. 197, fn. 6; id. at p. 205, fn. 14.)

In People v. Peabody (1975) 46 Cal.App.3d 43 [119 Cal.Rptr. 780], the defendant’s four-month-old infant suffered multiple nonaccidental fractures. (Id. at pp. 45-46.) There was no evidence that the defendant, as opposed to the baby’s father, had inflicted the injuries on her child. (Id. at p. 46.) Thus, “the conviction can stand only under that portion of the statute which *1217proscribes a person from wilfully causing or permitting a child to be placed in a health endangering situation under circumstances likely to produce great bodily harm or death.” {Ibid.., italics omitted.) The court held that under these circumstances, section 273a(1) “requires proof of criminal negligence.” (46 Cal.App.3d at p. 48.)

Prior to the Third District Court of Appeal’s decision in this case, two prior decisions of that court had rejected the argument that criminal negligence is an element of section 273a(1) when the defendant directly inflicts unjustifiable physical pain or mental suffering. In People v. Atkins, supra, 53 Cal.App.3d 348, a two-and-a-half-year-old child was beaten to death. (Id. at pp. 353-355.) The mother was charged with manslaughter, inflicting corporal punishment or injury on a child (§ 273d), and endangering the person and health of a child (§ 273a(1)). (53 Cal.App.3d at p. 352.) She was convicted of involuntary manslaughter, battery, which was a lesser included misdemeanor offense within the corporal punishment felony charge, and violation of section 273a(1). (53 Cal.App.3d at p. 352.)

On appeal, the mother contended the trial court erred in failing to instruct, sua sponte, that a conviction of endangering the person and health of a child under section 273a(1) “required a finding by the jury of ‘criminal negligence’ and ‘mere inattention or mistake in judgment [was] insufficient for such a conviction.’ ” (People v. Atkins, supra, 53 Cal.App.3d at p. 361.) The court concluded that no such instruction was required. It noted, “The statute requires, in subdivision (1), that for a person to be guilty of the offense, he wilfully cause or permit injury to a child.” (Ibid.) Unlike People v. Peabody, supra, 46 Cal.App.3d 43, on which the mother relied, the evidence here “pointed strongly to direct infliction of physical pain and mental suffering on the child by the defendant herself. An instruction on ‘criminal negligence’ with its attendant elements of foreseeability and the like, would be entirely inappropriate in such a case.” (People v. Atkins, supra, 53 Cal.App.3d at p. 361.)

Similarly, in People v. Wright, supra, 60 Cal.App.3d 6, the defendant was convicted of involuntary manslaughter in the beating death of his 29-month-old daughter. (Id. at p. 9.) On appeal, he asserted that the trial court erred in failing sua sponte to give an instruction regarding “involuntary manslaughter of the criminal negligence variety.” (Id. at p. 12.) The Court of Appeal rejected this contention, stating here, unlike Peabody, “the defendant was charged with personally injuring the child and not with willfully permitting its injury by another. The Peabody [requirement] for an instruction on criminal negligence is inappropriate where the evidence points to direct infliction of injury by the defendant.” (Id. at p. 14; see Iiams v. Superior *1218 Court (1965) 236 Cal.App.2d 80, 82-83 [45 Cal.Rptr. 627] [Proof of criminal agency “required evidence from which it might have been concluded that the injuries resulted from the intentional act of a human being. The offenses charged against [the defendant] are not directed toward an accidental or negligently caused condition.”]; 2 Witkin & Epstein, Cal. Criminal Law, supra, Crimes Against Decency and Morals, § 840(3), p. 957 [“If there is no evidence that a defendant himself inflicted the injuries on the child, P.C. 273a(1) requires proof of criminal negligence.” (Italics omitted.)].)

In other direct infliction of abuse cases, there is no discussion of mens rea, and, in particular, no indication that a criminal negligence standard was applied. (See, e.g., People v. Smith, supra, 35 Cal.3d at pp. 801-812, 809-810; People v. Jaramillo, supra, 98 Cal.App.3d at p. 835 [the defendant struck young daughters with stick]; People v. Ewing (1977) 72 Cal.App.3d 714, 716, 718 [140 Cal.Rptr. 299] [substantial evidence that the defendant deliberately inflicted severe injuries on child]; People v. Fuentes (1967) 253 Cal.App.2d 969, 973-974 [61 Cal.Rptr. 768] [child killed by concentrated blunt force].)

Moreover, other than the Court of Appeal opinion in this case, those cases which impose a criminal negligence requirement involve indirect abuse, such as failing to seek medical treatment, child endangerment, or willfully permitting situations that imperil children. (See, e.g., Walker v. Superior Court, supra, 47 Cal.3d at pp. 118, 135 [failure to obtain medical care resulting in 4-year-old’s death]; People v. Hansen (1997) 59 Cal.App.4th 473, 476-478 [68 Cal.Rptr.2d 897] [34-year-old man encouraged 14-year-old neighbor to play fatal game of Russian roulette]; People v. Deskin (1992) 10 Cal.App.4th 1397, 1399, 1402 [13 Cal.Rptr.2d 391] [unspecified misdemeanor child endangerment]; People v. Lee (1991) 234 Cal.App.3d 1214, 1217-1218, 1221 [286 Cal.Rptr. 117] [mother convicted of second degree murder and felony child endangerment for death of emaciated and dehydrated infant]; People v. Rippberger (1991) 231 Cal.App.3d 1667, 1673, 1682 [283 Cal.Rptr. 111] [failure to seek medical treatment for child suffering from meningitis]; People v. Odom (1991) 226 Cal.App.3d 1028, 1031, 1032 [277 Cal.Rptr. 265] [children living in squalor, and surrounded by highly dangerous drug lab and weapons]; People v. Pointer (1984) 151 Cal.App.3d 1128, 1131-1134 [199 Cal.Rptr. 357] [failure to provide proper nutrition and medical care]; Cline v. Superior Court (1982) 135 Cal.App.3d 943, 945-946, 948-949 [185 Cal.Rptr. 787] [father endangered toddler by throwing him into a car driven by another and then encouraging or knowingly permitting dangerous ride].)

However, a number of cases involving indirect abuse and child endangerment broadly state, as did Walker v. Superior Court, supra, 47 Cal.3d at page *1219135, that section 273a is a criminal negligence statute. In each case, the statement is supported by citation to other endangerment or indirect abuse cases. (See, e.g., People v. Deskin, supra, 10 Cal.App.4th at p. 1402 [relying on People v. Superior Court (Holvey) (1988) 205 Cal.App.3d 51 [252 Cal.Rptr. 335], disapproved in People v. Heitzman, supra, 9 Cal.4th at p. 209, fn. 17 (interpreting section 368)]; People v. Lee, supra, 234 Cal.App.3d at p. 1221 [relying on People v. Peabody, supra, 46 Cal.App.3d 43, and People v. Hernandez (1980) 111 Cal.App.3d 888, 895 [168 Cal.Rptr. 898]]; People v. Rippberger, supra, 231 Cal.App.3d at p. 1682 [relying on People v. Pointer, supra, 151 Cal.App.3d 1128, and People v. Peabody, supra, 46 Cal.App.3d 43]; People v. Odom, supra, 226 Cal.App.3d at p. 1032 [relying on Walker v. Superior Court, supra, 47 Cal.3d 112, and People v. Peabody, supra, 46 Cal.App.3d 43]; People v. Pointer, supra, 151 Cal.App.3d at p. 1134 [relying on People v. Peabody, supra, 46 Cal.App.3d 43].)

B. Appropriate Mens Rea When Child Abuse Directly Inflicted

We first consider whether a violation of section 273a(l) based on direct infliction of unjustifiable physical pain or mental suffering requires proof of general criminal intent or criminal negligence. As noted above, the Court of Appeal concluded that criminal negligence is required; defendant urges us to adopt this view. The Attorney General argues that only general criminal intent is required.

We begin with the language of the statute. The language “inflicts [on a child] unjustifiable physical pain or mental suffering” is most readily interpreted as requiring general criminal intent. That is, the statute describes “a particular act, without reference to intent to do a further act or achieve a future consequence.” (People v. Hood, supra, 1 Cal.3d at pp. 456-457.)

Moreover, the language of section 273a(1) is similar to that of section 273d, which proscribes corporal punishment or child beating.7 Indeed, sections 273a(1) and 273d have been described as “related statute[s].” (People v. Stewart (1961) 188 Cal.App.2d 88, 91 [10 Cal.Rptr. 217] [relying on section 273a case to interpret section 273d]; see also People v. Smith, supra, 35 Cal.3d at p. 801 [defendant convicted of both sections 273a(1) and 273d]; People v. Jackson (1971) 18 Cal.App.3d 504, 505 [95 Cal.Rptr. 919] [same]; People v. Mills (1991) 1 Cal.App.4th 898, 902 [2 Cal.Rptr.2d 614] [same].) Section 273d has been interpreted as a general intent statute. (People v. Atkins, supra, 53 Cal.App.3d at p. 358 [“[t]here need not be found a *1220deliberate intent to cause a traumatic condition, but only the more general intent to inflict upon a child any cruel or inhuman corporal punishment or injury”]; see People v. Mills, supra, 1 Cal.4th at p. 921 [corpus delicti of offense that child was injured and that injury was willful and intentional proven independent of the defendant’s admission]; People v. Lawrence (1956) 141 Cal.App.2d 630, 633 [297 P.2d 144] [When father struck and burned his daughter, “The corpus delicti was established when it was shown that the corporal injury was inflicted upon the child by the defendant and the circumstances and injuries to the child demonstrated that such injuries were deliberately and intentionally inflicted upon her.”]; People v. Gutierrez (1986) 180 Cal.App.3d 1076, 1079, 1080 [225 Cal.Rptr. 885] [defendant conceded section 273d general intent crime].) Furthermore, battery (§ 242), which requires general criminal intent, is a lesser included offense of section 273d. (People v. Atkins, supra, 53 Cal.App.3d at p. 352; People v. Stewart, supra, 188 Cal.App.2d at p. 90.) Given the similarities between section 273a(1) and section 273d, the mens rea element should be the same.

The similarities between felony child abuse involving direct infliction of unjustifiable physical pain or mental suffering and assault (§ 240) and assault with a deadly weapon (§ 245) also support the application of general criminal intent to section 273a(1). (People v. Smith, supra, 35 Cal.3d at p. 807 [“The elements of section 245 and the offense here [violation of section 273a(1)] are strikingly similar; the principal difference is that the assault prohibited by section 273a is committed on a child.” (Fn. omitted.)].) Assault and assault with a deadly weapon are general intent crimes. (People v. Colantuono (1994) 7 Cal.4th 206, 214-215, 218, 219, 221 [26 Cal.Rptr.2d 908, 865 P.2d 704].)

In addition, section 273ab,8 which proscribes assault resulting in a child’s death, expressly refers to assault “by means of force that to a reasonable person would be likely to produce great bodily injury . . . .” (Italics added.) Thus, when the Legislature chooses to create a reasonable person standard, it knows how to do so.

Defendant asserts that section 273a(1) and section 273d differ significantly because section 273a(1), but not section 273d, requires that the culpable conduct occur “under circumstances or conditions likely to produce great bodily harm or death.” “Thus, while only general criminal intent may apply to the actual infliction element, criminal negligence applies to the *1221condition or circumstances element of the offense.” Defendant likewise asserts that the relevant scienter in this case is “the scienter required with respect to the ‘circumstances or conditions likely to produce great bodily harm or death,’ not the scienter required with respect to the actual conduct or omission.”

There appears to be, however, no basis for concluding that the appropriate mens rea for direct infliction of unjustifiable physical pain or mental suffering is contained in the language “under circumstances or conditions likely to produce great bodily harm or death.” Section 273a does not provide that a defendant must “know or reasonably should know that his or her actions occur under circumstances or conditions likely to produce great bodily harm or death.” Rather, the statute proscribes the infliction of unjustifiable physical pain or mental suffering on a child. Whether that infliction is “under circumstances or conditions likely to produce great bodily harm or death” is a question for the trier of fact. In this case for example, the jury reasonably would have considered Michael’s tender age and fragile physical development, the degree of force used by defendant in violently shaking him on two different occasions, and the likelihood of great bodily harm or death created by that force as evidenced by the medical testimony and the injuries sustained. By contrast, if Michael had been a 17-year-old varsity linebacker, those facts would also have been “circumstances or conditions” the jury would consider.

In this manner, section 273a(1) is similar to that portion of section 245, subdivision (a)(1), which proscribes “assault ... by any means of force likely to produce great bodily injury.” Numerous cases have held that whether the force used by the defendant was likely to produce great bodily injury is a question for the trier of fact to decide. (People v. Armstrong (1992) 8 Cal.App.4th 1060, 1066 [10 Cal.Rptr.2d 839] [“ ‘[T]he question of whether or not the force used was such as to have been likely to produce great bodily injury, is one of fact for the determination of the jury based on all the evidence, including but not limited to the injury inflicted.’ ”]; People v. Duke (1985) 174 Cal.App.3d 296, 303 [219 Cal.Rptr. 873] [reasonable jury could not find beyond a reasonable doubt that headlock on victim constituted force likely to produce great bodily injury]; People v. Kinman (1955) 134 Cal.App.2d 419, 422 [286 P.2d 28] [“ ‘Whether a fist would be likely to produce such injury is to be determined by the force of the impact, the manner in which it was used and the circumstances under which the force was applied.’ ”]; People v. Pullins (1950) 95 Cal.App.2d 902, 904 [214 P.2d 436] [“Whether the force applied and the means used were calculated to ‘produce great bodily injury,’ is a question for the trier of fact.”]; People v. Bumbaugh (1941) 48 Cal.App.2d 791, 797 [120 P.2d 703] [“In view of the *1222injuries sustained by the complainant, as well as the manner of the assault made upon her, the jury was authorized to conclude that the manner of as well as the means used to perpetrate the attack constituted” assault by means of force likely to produce great bodily injury.]; see 1 Witkin & Epstein, Cal. Criminal Law, supra, Crimes Against the Person, § 419, p. 481 [“where the blows are serious, but still leave a question of fact as to the character of the force used, the defendant is entitled to an instruction on the included offense of simple assault”]; see also People v. Martinez (1973) 31 Cal.App.3d 355, 359 [107 Cal.Rptr. 284] [“It is now settled that assault with force likely to produce great bodily injury, is a general intent crime rather than a specific intent crime.”].) Moreover, similar language in other penal statutes has not precluded them from being interpreted as general criminal intent statutes. (See People v. Carter (1998) 60 Cal.App.4th 752, 755-756 [70 Cal.Rptr.2d 569] [Section 12022.7, subdivision (d), which provides, “Any person who personally inflicts great bodily injury under circumstances involving domestic violence in the commission or attempted commission of a felony,” is a general intent statute.]; People v. Gonzales (1994) 29 Cal.App.4th 1684, 1691, 1695, 1698 [35 Cal.Rptr.2d 450], called into doubt on other grounds in People v. Reed (1996) 13 Cal.4th 217, 229 [52 Cal.Rptr.2d 106, 914 P.2d 184] [Section 1192.7, subdivision (c)(8), which deems “serious” “any other felony in which the defendant personally inflicts great bodily injury on any person, other than an accomplice,” is a general intent statute.].)

The scienter for any crime is inextricably linked to the proscribed act or omission. (See § 20 [“In every crime or public offense there must exist a union, or joint operation of act and intent, or criminal negligence.”].) The actus reus for section 273a(1) is infliction of unjustifiable physical pain or mental suffering on a child. Hence, the scienter requirement applies to such an act. There is no separate scienter which attaches to the phrase “circumstances or conditions likely to produce great bodily harm or death.”

We note this inquiry, as a practical matter, will in most cases not differ significantly from the imposition of a criminal negligence mens rea element into the language “under circumstances or conditions likely to produce great bodily harm or death.” Thus in this case, while defendant denied any awareness that his actions were likely to harm Michael, any reasonable person would recognize that shaking a four-and-a-half-month-old infant, who had been bom three months prematurely and had the neck development of a four- to six-week-old, with the force equivalent to dropping him out of a second story window, was a circumstance or condition likely to result in great bodily harm or death.

As Presiding Justice Puglia observed in his dissent in the Court of Appeal, section 273a(1) is not unlike other criminal statutes that gauge the precise *1223measure of the defendant’s culpability for an intentional act by matters extrinsic to the intent element of the crime. The actor may neither anticipate nor have any particular intent or knowledge with respect to these extrinsic matters. For example, a thief unaware the property stolen is worth more than $400 has no defense against conviction of grand theft because of ignorance of the value of the property. (See §§ 487, 488.) Nor may a burglar, unaware the structure entered is inhabited, avoid conviction of first degree burglary on the basis of such ignorance. (§§ 459, 460.) Thus the scienter required for felony and misdemeanor theft is precisely the same. So also is the scienter required for first degree burglary and the lesser crime of second degree burglary.

Such is the case with felony child abuse under the second branch of section 273a(1). The scienter required for both section 273a(1) and section 273a, former subdivision (2) (now section 273a, subdivision (b)) violations is the same. The reference to the infliction of unjustifiable physical pain or mental suffering occurring “under circumstances or conditions likely to produce great bodily harm or death” in subdivision (1) simply states the context in which the subdivision applies. (See People v. Carter, supra, 60 Cal.App.4th at p. 755 [“reference to domestic violence in [section 12022.7,] subdivision (d) was designed to describe the context in which the subdivision applies, not the intent requirement needed under that provision”].) It is for the trier of fact to determine whether the act was done “under circumstances or conditions likely to produce great bodily harm or death,” i.e., under conditions “in which the probability of serious injury is great.” (§ 273a(1); People v. Jaramillo, supra, 98 Cal.App.3d at p. 835.) If so, the crime is punishable as a felony; if not, solely as a misdemeanor. {People v. Deskin, supra, 10 Cal.App.4th at p. 1401.)

Defendant recognizes that “In cases where a person brutally beats a child, it appears absurd to engage in[] an inquiry as to whether an ordinary prudent person would have been aware of the risk of serious bodily harm or death.” However, there is nothing intrinsically distinct about the act of violently shaking a vulnerable infant that separates it from other methods of direct child abuse.

Nor, contrary to defendant’s assertion, will section 273a(l) be tantamount to a strict liability crime if we do not conclude criminal negligence is a required element of the statute. Generally speaking, a strict liability offense is one which dispenses with a mens rea, scienter, or wrongful intent element. (See People v. Simon (1995) 9 Cal.4th 493, 519 [37 Cal.Rptr.2d 278, 886 P.2d 1271].) Felony child abuse requires a mens rea: the defendant must willfully inflict unjustifiable physical pain or mental suffering on a child.

*1224At oral argument, defendant’s counsel asserted that concluding the second branch of section 273a(l) has a general criminal intent mens rea would result in parents being prosecuted for shaking a child or slapping it on the back to save it from choking. However, shaking or slapping a choking child, whatever physical pain or mental suffering that may involve, is justified. Section 273a(l) sanctions only the infliction of unjustifiable physical pain or mental suffering.

In sum, we conclude that when the conduct at issue involves the direct infliction of unjustifiable physical pain or mental suffering on a child, criminal negligence is not an element of the offense. Rather, the defendant must have a mens rea of general criminal intent to commit the proscribed act. In addition, the trier of fact must determine whether the infliction of the unjustifiable physical pain or mental suffering on a child was under circumstances or conditions likely to produce great bodily harm or death. If so, the crime is punishable as a felony. If not, it is punishable solely as a misdemeanor.

In light of this conclusion, there is no need to address whether the Court of Appeal properly modified the judgment to reflect a conviction for misdemeanor child abuse without giving the prosecution an opportunity to retry defendant.

Disposition

The judgment of the Court of Appeal is reversed and the matter is remanded for further proceedings consistent with this opinion.

George, C. J., Baxter, J., Werdegar, J., and Chin, J., concurred.

MOSK, J.

I concur fully in the judgment, which reverses the judgment of the Court of Appeal insofar as it modified the judgment of the superior court convicting appellant, on a jury’s verdict, of felony child abuse to one convicting him of misdemeanor child abuse.

I also concur generally in the opinion of the court prepared by Justice Brown.

I write separately to present my own analysis. I do so in order to explicate felony child abuse—a task that is greatly needed in light of the divergent views expressed by the majority and by the dissenting justice in the Court of Appeal, which reflect similarly divergent views in reported decisions generally.

*1225Felony child abuse—which, strictly speaking, is an alternative felony/ misdemeanor offense—is defined by statute as presently set out in Penal Code section 273a, subdivision (a): “Any person who, under circumstances or conditions likely to produce great bodily harm or death, willfully causes or permits any child to suffer, or inflicts thereon unjustifiable physical pain or mental suffering, or having the care or custody of any child, willfully causes or permits the person or health of that child to be injured, or willfully causes or permits that child to be placed in a situation where his or her person or health is endangered,” is guilty, alternatively, of a felony or a misdemeanor.

Misdemeanor child abuse is defined by statute as presently set out in Penal Code section 273a, subdivision (b): “Any person who, under circumstances or conditions other than those likely to produce great bodily harm or death, willfully causes or permits any child to suffer, or inflicts thereon unjustifiable physical pain or mental suffering, or having the care or custody of any child, willfully causes or permits the person or health of that child to be injured, or willfully causes or permits that child to be placed in a situation where his or her person or health may be endangered, is guilty of a misdemeanor.”

It is clear at first glance how felony child abuse differs from its misdemeanor counterpart. The felony requires that the “circumstances or conditions” (Pen. Code, § 273a, subd. (a)) establishing the “context” of the perpetrator’s conduct and mental state (People v. Carter (1998) 60 Cal.App.4th 752, 755 [70 Cal.Rptr.2d 569]) must be “likely to produce great bodily harm or death” (Pen. Code, § 273a, subd. (a)). The misdemeanor, contrariwise, requires that such circumstances or conditions must not be likely to do so. (Id., § 273a, subd. (b).)

It is not as clear, however, what felony child abuse requires concerning the perpetrator’s conduct and mental state in addition to context.

Cleaving to the syntax of the statutory language, we articulate its clauses thus: “Any person who, under circumstances or conditions likely to produce great bodily harm or death, [i] willfully [a] causes or permits any child to suffer, or [b] inflicts thereon[,] unjustifiable physical pain or mental suffering, or [ii] having the care or custody of any child, [a] willfully causes or permits the person or health of that child to be injured, or [b] willfully causes or permits that child to be placed in a situation where his or her person or health is endangered,” may be guilty of a felony. (Pen. Code, § 273a, subd. (a), brackets and bracketed material added.)

So articulating the clauses of the statutory language, we discern what have been referred to as four “branches,” which are separate but not necessarily independent:

*1226Under the first branch, a person may be guilty of felony child abuse if, “under circumstances or conditions likely to produce great bodily harm or death,” he “willfully causes or permits any child to suffer . . . unjustifiable physical pain or mental suffering . . . (Pen. Code, § 273a, subd. (a).)1 Crucial for present purposes is the adverb “willfully.” In the setting of the statutory language, he must, at least, intentionally commit an act or omission that happens to result in a child “suffer[ing] . . . unjustifiable physical pain or mental suffering.” He must do more as well. That is to say, he must “cause[] or permit[]” the child “to suffer . . . unjustifiable physical pain or mental suffering” for the purpose of bringing about, or allowing, such a result. The reason is this: The adverb “willfully” modifies the verbs “causes” and “permits”; the object of those verbs is not an act or omission that happens to result in the child “suffer[ing] . . . unjustifiable physical pain or mental suffering,” but rather that very result itself. (Cf. People v. Gorshen (1959) 51 Cal.2d 716, 730 [336 P.2d 492], disapproved on another point, People v. Wetmore (1978) 22 Cal.3d 318, 327, fn. 7 [149 Cal.Rptr. 265, 583 P.2d 1308] [holding that to kill “willfully” within the meaning of Pen. Code, § 189, means to act for the purpose of bringing about death].)2

Under the second branch—which is of particular concern here—a person may be guilty of felony child abuse if, “under circumstances or conditions likely to produce great bodily harm or death,” he “willfully . . . inflicts . . . unjustifiable physical pain or mental suffering” on “any child.” (Pen. Code, § 273a, subd. (a).)3 As explained above, he must, at least, intentionally commit an act that “inflicts” such “unjustifiable physical pain or mental *1227suffering”; in addition, he must do so for the purpose of bringing about such a result.

Under the third branch, a person may be guilty of felony child abuse if, “under circumstances or conditions likely to produce great bodily harm or death,” he “willfully causes or permits the person or health “of any child in his “care or custody” “to be injured . . . .” (Pen. Code, § 273a, subd. (a).) Again, he must, at least, intentionally commit an act or omission that “causes or permits the person or health” of such a child “to be injured”; in addition, he must do so for the purpose of bringing about, or allowing, such a result.

Under the fourth and final branch, a person may be guilty of felony child abuse if, “under circumstances or conditions likely to produce great bodily harm or death,” he “willfully causes or permits” any child in his “care or custody” “to be placed in a situation where” the child’s “person or health is endangered . . . .” (Pen. Code, § 273a, subd. (a).) Yet again, he must, at least, intentionally commit an act or omission that “causes or permits” such a child “to be placed in a situation where” the child’s “person or health is endangered”; in addition, he must do so for the purpose of bringing about, or allowing, such a result.

It follows that felony child abuse may be described as including three elements. The first goes to the perpetrator’s context, specifically, “circumstances or conditions likely to produce great bodily harm or death.” (Pen. Code, § 273a, subd. (a).) The second goes to the perpetrator’s conduct, specifically, an act or omission that “causes or permits” a child “to suffer . . . unjustifiable physical pain or mental suffering,” or “inflict[s]” on a child “unjustifiable physical pain or mental suffering,” or “causes or permits the person or health” of a child in his “care or custody” “to be injured,” or “causes or permits” such a child “to be placed in a situation where” the child’s “person or health is endangered.” (Ibid.) The third goes to the perpetrator’s mental state, specifically, an intent to commit one of the *1228proscribed acts or omissions, and a purpose to bring about, or allow, one of the proscribed results. (Ibid.) 4

Neither the majority nor the dissenting justice in the Court of Appeal explicates felony child abuse thus. Each sets forth extensive discussion. Such discussion, however, turns out to be nothing more, and nothing less, than an analytical tar baby, with which courts should decline to struggle.

In part, the dissenting justice in the Court of Appeal indulged himself, impliedly, in considerations of “general intent” and “specific intent.” Inappropriately. For such notions do not assist in performing the task at hand, which is to explicate felony child abuse. “General intent” and “specific intent” “evolved as labels to identify” particular offenses, with “specific intent” “admitting ... the defense of voluntary intoxication” and “general intent” not doing so. (People v. Cain (1995) 10 Cal.4th 1, 83 [40 Cal.Rptr.2d 481, 892 P.2d 1224] (conc. opn. of Mosk, J.); accord, People v. Hood (1969) 1 Cal.3d 444, 455-457 [82 Cal.Rptr. 618, 462 P.2d 370]; see People v. Whitfield (1994) 7 Cal.4th 437, 463 [27 Cal.Rptr.2d 858, 868 P.2d 272] (conc. and dis. opn. of Mosk, J.).) “General intent” has usually been affixed if the mental element of an offense entails only an intent to engage in certain proscribed conduct. (People v. Hood, supra, 1 Cal.3d at pp. 456-457.) In contrast, “specific intent” has usually been affixed if the mental element of an offense entails an intent to engage in certain proscribed conduct for the purpose of bringing about, or allowing, a certain proscribed result. (Id. at p. 457.) “General intent” and “specific intent” have shown themselves to be “notoriously difficult ... to define and apply.” (Id. at p. 456; accord, People v. Cain, supra, 10 Cal.4th at p. 84 (conc. opn. of Mosk, J.).) For this reason, “they have proved to be mischievous.” (People v. Cain, supra, 10 Cal.4th at p. 84 (conc. opn. of Mosk, J.).) Even if they were not so, they simply do not assist in explicating felony child abuse. Neither do they aid in describing its mental element. Indeed, were we to attempt to affix the label of “general intent” or “specific intent,” we would doubtless have to select both—“general intent,” insofar as its mental element entails an intent to commit one of the proscribed acts or omissions; and “specific intent,” insofar as it entails an intent to do so for the purpose of bringing about, or allowing, one of the proscribed results.

In other part, the dissenting justice in the Court of Appeal indulged himself, expressly, in considerations of “general intent” and “criminal negligence;.” Again, inappropriately. For such notions also do not assist in *1229performing the task at hand, which is to explicate felony child abuse. “Criminal negligence”—to quote People v. Peabody (1975) 46 Cal.App.3d 43 [119 Cal.Rptr. 780] (hereafter sometimes Peabody)—“must go beyond [ordinary negligence] required for civil liability and must amount to a ‘gross’ or ‘culpable’ departure from the required standard of care. [Citations.] [It] must be aggravated or reckless; that is, it must be such a departure from what would be the conduct of an ordinarily prudent person under the same circumstances as to be incompatible with a proper regard for human life. [It] must show an indifference to the consequences, and this has been said to require knowledge, actual or imputed, that the act tends to endanger another’s life.” (Id. at p. 47.)5 “General intent” and “specific intent” are, at least, complementary terms. “General intent” and “criminal negligence,” as is apparent, are not.

The dissenting justice in the Court of Appeal claimed to discern a mental element of “criminal negligence” in felony child abuse insofar as the perpetrator’s conduct involves a proscribed omission. He did so apparently on the assumption that such an omission implies such negligence.

The dissenting justice in the Court of Appeal was wrong: felony child abuse does not include a mental element of “criminal negligence” for the proscribed omission. To be sure, such negligence may accompany such an omission. But so may recklessness or knowledge or even purpose—as when one fails to act in order not to hinder a desired result.

For its part, the majority in the Court of Appeal claimed to discern a mental element of “criminal negligence” in felony child abuse generally, without regard to whether the perpetrator’s conduct involves either a proscribed act or a proscribed omission. It did so following Peabody and its progeny, which had done the same, albeit while looking to branches of the offense other than the second, which is of particular concern here, for the sake of assuring “consistency” in all its branches. It also did so in reliance on the offense’s context element, which covers “circumstances or conditions likely to produce great bodily harm or death” (Pen. Code, § 273a, subd. (a)), in the belief that, without “criminal negligence,” the offense would not include a mental element other than that indicated by “willfulness,” which it shared with its misdemeanor counterpart, and would thereby show itself to be “tantamount” to a “disfavored” offense of strict liability.

*1230The majority in the Court of Appeal was wrong as well: felony child abuse does not include a mental element of “criminal negligence” generally. It ought not to have followed Peabody and its progeny. These decisions did not read what the statutory language stated as much as state what they had read into such language. Neither ought it to have relied on the offense’s context element covering “circumstances or conditions likely to produce great bodily harm or death.” (Pen. Code, § 273a, subd. (a).) “Criminal negligence” is not referred to in the statutory language. Indeed, it is not even suggested. There is absolutely no “require[ment]” that the perpetrator possess either “actual or imputed” “knowledge” (People v. Peabody, supra, 46 Cal.App.3d at p. 47) that his conduct is “likely to produce great bodily harm or death” (Pen. Code, § 273a, subd. (a)). Moreover, even without “criminal negligence,” the offense cannot be deemed to be “tantamount” to one of strict liability. An offense is of this kind if it does not include any mental element whatsoever. (See People v. Simon (1995) 9 Cal.4th 493, 519-522 [37 Cal.Rptr.2d 278, 886 P.2d 1271].) This offense is not such. Neither can it be considered similar. For it obviously includes a mental element, which is indicated by “willfulness.” It does not stand in need of another. To be sure, the law requires that an offense must include a conduct element and a mental element in union. (Pen. Code, § 20.)6 But it does not prohibit it from including an element of a third kind, like a context element, not possessing aspects of either of the other two.7

The majority in the Court of Appeal evidently believed that felony child abuse could not, or at least should not, be distinguished from its misdemeanor counterpart solely by its context element. There is no basis for a belief of this sort. A felony may be distinguished from a corresponding misdemeanor solely on the basis of a context element. So is it with theft (Pen. Code, § 484), specifically, the felony (more precisely, the alternative felony/misdemeanor) of grand theft (id., §§ 487, subd. (a), 489, subd. (b)) and the misdemeanor of petty theft (id., §§ 488, 490), where the former may be distinguished from the latter solely on the basis of a context element indicating that the value of the item taken exceeds $400 (id., § 487, subd. (a); see id., § 488). Likewise with burglary (id., § 459), specifically, the felony of first degree burglary {id., §§ 460, subd. (a), 461, subd. 1) and the *1231misdemeanor (more precisely, the alternative felony/misdemeanor) of second degree burglary (id., §§ 460, subd. (b), 461, subd. 2), where the former may be distinguished from the latter solely on the basis of a context element indicating that the nature of the structure entered was that of an inhabited dwelling house (id., § 460, subd. (a); see id., § 460, subd. (b)). Unproblematically so. The context element of a felony may properly mark it as more serious than a corresponding misdemeanor because the consequences of the felony, either actual or potential, are deemed to be graver than those of the misdemeanor, as where the item taken in a theft has a value exceeding $400 or the structure entered in a burglary is an inhabited dwelling house. Thus is it here. The context element of felony child abuse covers “circumstances or conditions likely to produce great bodily harm or death.” (Id., § 273a, subd. (a).) That of its misdemeanor counterpart, in contrast, covers “circumstances or conditions other than those likely to produce great bodily harm or death.'” (Id., § 273a, subd. (b), italics added.)8

In conclusion, I join in reversing the judgment of the Court of Appeal insofar as it modified the judgment of the superior court convicting appellant of felony child abuse to one convicting him of its misdemeanor counterpart.

Kennard, J., concurred.

On March 9, 1999, the opinion was modified to read as printed above.

Notes and Questions Notes and Questions

Criminal offense definitions nearly always require proof of something more than an “act,” or conduct. In the standard description, criminal offenses may have three kinds of elements: (1) conduct, (2) circumstances, and (3) results. Murder is the classic offense defined by the result (“causing the death of another”). Not all offenses have all three types; many offenses require no result or consequence. Circumstance elements include a wide range statutory requirement—e.g., the victim’s age (as in a child abuse or statutory rape statute); the time of day or the status of the building (burglary was once defined as entry into “the dwelling house of another at nighttime”); surrounding conditions (conduct done “in circumstances likely to cause harm”); or who owns property (taking of “the property of another”). Can you identify the all the elements in the offenses at issue in Stark and Sargent?

Stark repeats the standard definition of general intent: “the intent to do the act.” For some offenses, that provides a clear answer about the mens rea of the offense. But for other offenses, such as the statute at issue Sargent, “general intent” provides a less clear answer. There, the key question is whether the prosecution must prove something more about the defendant’s mental state regarding other elements of the crime. This is one reason that courts and commentators have long complained that “Specific and general intent have been notoriously difficult terms to define and apply, and a number of text writers recommend that they be abandoned altogether.” People v. Hood, 462 P.2d 370 (Cal. 1969). But determining what else, beyond his conduct, that an offender must intend, know, or be aware of can pose difficult interpretative questions, as in Sargent.

California’s felony child abuse statute effectively defines more than one offense, because harm can be inflicted directly (or actively) by the defendant, or indirectly (or passively) by, for example, permitting someone else to abuse the child. Both forms of child abuse are general intent crimes. General intent answers the mens rea question relatively easily for direct infliction cases—the offender must intend his physical actions. But Sargent follows earlier California cases, such as Walker v. Superior Court, 763 P.2d 852 (Cal. 1988), in concluding that, in passive cases, the defendant must have intended his conduct and been criminally negligent. That is not, as we will see next, a requirement of “specific intent.” But it is something more than merely “intent to do the act.” Can you explain why the court concluded that this additional negligence requirement is important to ensuring that only culpable, blameworthy actors are convicted in such cases?

Note on 'willful blindness' Note on 'willful blindness'

For an excellent, short account of the distinctions between "actual knowledge," "willful blindness" and a "should have known" mens rea, as well as the challenges of proving a party's subjective knowledge or beliefs, see Daniel Richman, Actual Knowledge, Willful Blindness, and the Jan. 6 HearingsLawFare blog, June 21, 2022 ("defendants cannot escape criminal liability 'by deliberately shielding themselves from clear evidence of critical facts that are strongly suggested by the circumstances. ... [D]efendants who behave in this manner are just as culpable as those who have actual knowledge.'").

The U.S. Supreme Court summarized the doctrine of willful blindness (or "conscious avoidance") in Global-Tech Appliances, Inc. v. SEB S. A., 563 U.S. 754 (2011):

The doctrine of willful blindness is well established in criminal law. Many criminal statutes require proof that a defendant acted knowingly or willfully, and courts applying the doctrine of willful blindness hold that defendants cannot escape the reach of these statutes by deliberately shielding themselves from clear evidence of critical facts that are strongly suggested by the circumstances. The traditional rationale for this doctrine is that defendants who behave in this manner are just as culpable as those who have actual knowledge. Edwards, The Criminal Degrees of Knowledge, 17 Mod. L. Rev. 294, 302 (1954) (observing on the basis of English authorities that “up to the present day, no real doubt has been cast on the proposition that [willful blindness] is as culpable as actual knowledge”). It is also said that persons who know enough to blind themselves to direct proof of critical facts in effect have actual knowledge of those facts.

Practice: Charges against Deputy in 2018 Parkland School Shooting Practice: Charges against Deputy in 2018 Parkland School Shooting

Based on the following materials, assess the arguments for and against liability for former Broward County sheriff's deputy Scot Peterson arising from the 2018 mass shooting at a high school in Parkland, Florida. Does Peterson's conduct, or inaction, meet the requirements of the statutes under which he is charged? Does his mens rea, or mental state, meet the statutes' requirements? Even if there is a plausible case for guilt, should Peterson have been charged, given that prosecutors always have discretion not to file charges even when they have sufficient evidence to prove guilt?

Fourteen students and three staff members were killed, and 17 others were injured, by 19-year-old gunman Nikolas Cruz at Marjory Stoneman Douglas High School in Parkland, Florida, on February 14, 2018. Cruz pled guilty in October 2021 to 17 counts of capital murder and 17 counts of attempted murder. His jury sentencing trial is in July 2022.

Parkland Deputy Scot Peterson, the sole officer on school grounds during the shooting, was charged in June 2019 with eleven offenses based on his failure to respond during the incident. His trial has been delayed several times and is now scheduled for February 2023. His charges include seven counts felony neglect of a child because seven victims were under age 18. Charges against Peterson are based on the following Florida statutes:

Counts 1-6: Fla. Stat. 827.03(2)(b)

(b) A person who willfully or by culpable negligence neglects a child and in so doing causes great bodily harm, permanent disability, or permanent disfigurement to the child commits a felony of the second degree, punishable as provided in s. 775.082, s. 775.083, or s. 775.084.

Count 7: Fla. Stat. 827.03(2)(d)

(d) A person who willfully or by culpable negligence neglects a child without causing great bodily harm, permanent disability, or permanent disfigurement to the child commits a felony of the third degree, punishable as provided in s. 775.082, s. 775.083, or s. 775.084.

Counts 8-10: Fla. Stat. 784.05(1) Culpable negligence—

(1) Whoever, through culpable negligence, exposes another person to personal injury commits a misdemeanor of the second degree, punishable as provided in s. 775.082 or s. 775.083.

[Count 11 alleges perjury.]

Statutory definitions for terms in Counts 1-7:

Fla. Stat. § 827.01. Definitions. As used in this chapter:

  • “Caregiver” means a parent, adult household member, or other person responsible for a child’s welfare.
  • “Child” means any person under the age of 18 years.
  • “Placement” means the giving or transferring of possession or custody of a child by any person to another person for adoption or with the intent or purpose of surrendering the control of the child.

Fla. Stat. § 827.03(e). “Neglect of a child” means:

  1. A caregiver’s failure or omission to provide a child with the care, supervision, and services necessary to maintain the child’s physical and mental health, including, but not limited to, food, nutrition, clothing, shelter, supervision, medicine, and medical services that a prudent person would consider essential for the well-being of the child; or
  2. A caregiver’s failure to make a reasonable effort to protect a child from abuse, neglect, or exploitation by another person.

The following provide an account of the Peterson's conduct and the circumstances:

Parkland Officer Who Stayed Outside During Shooting Faces Criminal Charges, Audra D.S. Burch & Alan Blinder, N.Y. Times, June 4, 2019:

... The 15-month investigation by the Florida Department of Law Enforcement that led to the charges [against Scot Peterson] found that the former Broward County sheriff’s deputy, assigned as a school resource officer to Stoneman Douglas High, “did absolutely nothing to mitigate” the shooting, the department’s commissioner, Rick Swearingen, said in a statement. “There can be no excuse for his complete inaction and no question that his inaction cost lives,” he said.

The charges were an unusual instance of law enforcement officers being held criminally liable for not protecting the public.  … Experts say that criminally charging a law enforcement officer for allegedly being negligent in his response to a mass shooting is new ground. …

The Department of Law Enforcement said its inquiry showed that Mr. Peterson did not investigate the source of the gunshots, retreated during the shooting while victims were still under attack and directed other law enforcement officers to remain 500 feet away from the building.

The sweeping investigation included interviews with 184 witnesses, along with reviews of video surveillance, to piece together what officials acknowledged was a slow and chaotic law enforcement response. Officials added a perjury charge alleging that Mr. Peterson knowingly made a false statement while under oath in stating that he did not hear shots fired after he arrived at the 1200 building, except for the first two or three shots he heard.

The warrant portrayed Mr. Peterson, the only armed guard on campus, as an officer with a wealth of active shooter training who knew the gunman was inside, but did not go in to try to stop him as he killed and injured students and staff. …

Since he was first interviewed by Broward detectives two days after the killings, Mr. Peterson has said he did respond, by alerting the police, locking the school down and evacuating children in the courtyard. “There wasn’t even time to think,” Mr. Peterson told The Washington Post. “It just happened and I started reacting.” ...

States can create obligations for the police under certain circumstances — for example, a crossing guard who is assigned to keep children from getting run over. But those would be civil liabilities, said Darren L. Hutchinson, a professor at the University of Florida School of Law, not criminal ones.

Professor Hutchinson noted that the criminal statutes under which Mr. Peterson was charged ... were not specifically drafted for law enforcement officers and are usually applied to parents. “Normally we don’t think of police officers as caregivers for children,” he said.

What Officials Say Scot Peterson Did Not Do During the Parkland School Shooting, Richard A. Oppel Jr. & Shreeya Sinha, N.Y. Times, June 4, 2019

  • He didn’t go into the building during the shooting or investigate the source of gunfire. Mr. Peterson had been trained that officers responding to an active shooter must “immediately go to confront the shooter” and “move directly and quickly toward known threat.” But he did not enter the 1200 building at the school, where the gunman was shooting students and teachers, according to an affidavit filed in support of the warrant for his arrest.
  • Instead, he retreated to another building nearby. Mr. Peterson fled to a spot about 75 feet away from the 1200 building, “remaining there during the entire incident” in what the affidavit described as “a position of increased personal safety.”
  • He advised other law enforcement officers to remain away from the building. At 2:28 p.m. — seven minutes after the gunman entered the building — Mr. Peterson said over the radio: “Broward, do not approach the 12 or 1300 buildings. Stay at least 500 feet away at this point.”

During the time Mr. Peterson remained outside the building, the gunman shot and killed six of his victims, including five students, and wounded four others, according to the affidavit.

Mr. Peterson was charged with perjury for making a false statement that he did not hear any shots fired after he arrived at the building, except for the first two or three shots he heard, according to the affidavit.

Timeline of events (from N.Y. Times video compiling school surveillance footage):

2:21pm: gunmen enters school and starts shooting

2:22pm:  Deputy Peterson alerted, gets in golf cart

2:23pm:  drives for one minute, takes up a position 100 feet from Building 12 entrance

2:28pm:  Shooting continued from 2:21 to 2:28, when gunman exits Building 12 and fled. Peterson remained in his outside position away from the building.

4.1.2 Specific Intent 4.1.2 Specific Intent

State v. Schminkey State v. Schminkey

STATE of Iowa, Appellee, v. William Joseph SCHMINKEY, Appellant.

No. 97-2333.

Supreme Court of Iowa.

July 8, 1999.

*787Linda Del Gallo, State Appellate Defender, and Christopher Cooklin, Assistant State Appellate Defender, for appellant.

Thomas J. Miller, Attorney General, Richard J. Bennett, Assistant Attorney General, Ray Lough, County Attorney, and Karen Duncan, Assistant County Attorney, for appellee.

TERNUS, Justice.

The defendant, William Schminkey, entered Alford pleas to the offenses of homicide by vehicle, see Iowa Code § 707.6A(.1) (1997), and theft of a motor vehicle, see id. §§ 714.1, .2(2). See generally North Carolina v. Alford, 400 U.S. 25, 37, 91 S.Ct. 160, 167, 27 L.Ed.2d 162, 171 (1970) (holding that an accused may consent to the imposition of a prison sentence even if he is unwilling or unable to admit his participation in the acts constituting the crime). The court sentenced him to consecutive ten-year and five-year terms of imprisonment, respectively. Schminkey now claims that the record lacks a sufficient factual basis for a finding that he was guilty of the theft offense. He also claims his counsel rendered ineffective assistance in failing to object to the prosecutor’s breach of the plea agreement with respect to the State’s sentencing recommendations.

We agree that the record does not show a factual basis for Schminkey’s conviction of the crime of theft of a motor vehicle. This conclusion makes it unnecessary to consider Schminkey’s claim that the State breached the plea agreement. Accordingly, we vacate the sentence on the theft conviction and remand for further proceedings.

I. Factual and Procedural Back-grcmnd.

The underlying facts of the tragic episode culminating in Schminkey’s convictions are undisputed. Because the ultimate focus in this case is on the record before the district court at the time of the guilty plea proceedings, we recite the facts as they appeared to the court during that phase of the case.

Schminkey spent the evening of May 17, 1997 drinking, first at a party and then at a bar. Although Schminkey has no recollection of leaving the bar or of what happened after he left, witnesses established that he departed the bar and then drove a pickup owned by Dale Kimm, a man Schminkey did not know. Schminkey did not have Kimm’s permission to drive the pickup. Several witnesses observed the pickup being driven erratically and in excess of the speed limit. -They saw the vehicle heading north, approaching a controlled intersection in the town of Van Horne at an excessive rate of speed. The pickup went through the intersection without slowing down and struck two vehieleSi facing south, that were stopped at the intersection stop sign. The driver and only occupant of the first vehicle, nineteen-year-old Jason Kray, died en route to the hospital.

Notwithstanding the collision, the driver of the pickup appeared to be fleeing the scene, accelerating the engine and proceeding down the road for another block or so before crashing into a fence. Witnesses who assisted in extricating Schminkey from the pickup said he smelled strongly of alcohol. A later urine test showed his blood alcohol level to be .189, significantly over the legal limit of .10. See Iowa Code § 321J.2(1).

Schminkey was charged with homicide by vehicle in violation of Iowa Code section 707.6A(1), involuntary manslaughter in violation of Iowa Code section 707.5(1), and theft of a motor vehicle in violation of Iowa Code sections 714.1 and 714.2(2). Claiming he was unable to recall the events of the evening due to his intoxication, Schminkey entered into a plea agreement with the State, whereby, in exchange for his Alford plea to the theft and homicide charges, the State would dismiss the involuntary manslaughter charge and recom*788mend that Sehminkey’s sentences on the remaining charges run concurrently.

The State dismissed the manslaughter charge. At the plea hearing, the district court made a determination that a factual basis for the pleas appeared in the record and then accepted Schminkey’s guilty pleas. Schminkey was subsequently sentenced to consecutive ten-year and five-year terms of incarceration.

Schminkey appealed, claiming his counsel rendered ineffective assistance in two ways. First, he contends his counsel should not have allowed him to plead guilty to the theft charge because there was not a factual basis for a finding that he intended to permanently deprive the owner of his vehicle. He also claims that the prosecutor breached the plea agreement because the prosecutor did not expressly state the State’s recommendation for concurrent sentences at the sentencing hearing. Schminkey asserts his counsel rendered ineffective assistance in failing to object to this breach. The court of appeals affirmed, and we granted further review.

II. General Principles Governing Ineffective-Assistance-of-Counsel Claims.

The Sixth Amendment to the United States Constitution guarantees a defendant the right to the effective assistance of counsel. See U.S. Const. amend. VI; Taylor v. State, 352 N.W.2d 683, 685 (Iowa 1984). We review claims that this constitutional right has been violated de novo. See State v. Ray, 516 N.W.2d 863, 865 (Iowa 1994).

Claims of ineffective assistance of counsel are generally preserved for postconviction relief proceedings. See id. Where the record is adequate to address the issue, however, such claims will be considered on direct appeal. See id. As will become clear, this is such a case.

The standards required to prevail on a claim of ineffective assistance of counsel are well established. The defendant must prove by a preponderance of the evidence that his “counsel failed to perform an essential duty,” and that he “was prejudiced by counsel’s error.” State v. Brooks, 555 N.W.2d 446, 448 (Iowa 1996).

III. Did Trial Counsel Render Ineffective Assistance in Allowing Schminkey to Plead Guilty to the Offense of Theft of a Motor Vehicle?

The district court may not accept a guilty plea without first determining that the plea has a factual basis. See Iowa R.Crim. P. 8(2)(b); State v. Burtlow, 299 N.W.2d 665, 668 (Iowa 1980). This requirement exists even where the plea is an Alford plea. See Alford, 400 U.S. at 38 n. 10, 91 S.Ct. at 168 n. 10, 27 L.Ed.2d at 171-72 n. 10. Where a factual basis for a charge does not exist, and trial counsel allows the defendant to plead guilty anyway, counsel has failed to perform an essential duty. See Brooks, 555 N.W.2d at 448. Prejudice in such a case is inherent. See State v. Hack, 545 N.W.2d 262, 263 (Iowa 1996) (holding that where there is no factual basis for a guilty plea, ineffective assistance of counsel is established). Therefore, our first and only inquiry is whether the record shows a factual basis for Schminkey’s guilty plea to the charge of theft of a motor vehicle. In deciding whether a factual basis exists, we consider the entire record before the district court at the guilty plea hearing, including any statements made by the defendant, facts related by the prosecutor, the minutes of testimony, and the presentence report. See Brooks, 555 N.W.2d at 448-49; State v. Fluhr, 287 N.W.2d 857, 867-68 (Iowa 1980), overruled in part on other grounds by State v. Kirchoff 452 N.W.2d 801, 805 (Iowa 1990).

The offense of theft is defined in section 714.1(1), which states that a person commits theft when he “[t]akes possession or control of the property of another, or property in the possession of another, with the *789 intent to deprive the other thereof.” (Emphasis added.) Sehminkey challenges the factual basis for the intent element of this crime.

The intent required for the commission of a theft is an intent to deprive the owner of his or her property. See Eggman v. Scurr, 311 N.W.2d 77, 79 (Iowa 1981). Sehminkey argues that this element of the crime requires proof that he intended to permanently deprive the owner of his vehicle. He contrasts the theft statute, section 714.1(1), with Iowa Code section 714.7, defining the crime of operating a vehicle without the owner’s consent. The latter statute prohibits the “possession or control of ... any self-propelled vehicle ... without the consent of the owner of such, but imthout the intent to permanently deprive the oumer thereof.” Iowa Code § 714.7 (emphasis added). This crime, operating a vehicle without the owner’s consent, is expressly made a lesser included offense of the crime of theft. See id.

Sehminkey correctly argues that an intent to permanently deprive the owner of his property is an essential element of theft under section 714.1(1). The legislature’s distinction of the crime of theft from the crime of operating a vehicle without the owner’s consent — the existence or absence of an intent to permanently deprive the owner — supports this conclusion. In addition, this interpretation of section 714.1(1) is consistent with the crime of larceny as it was defined prior to the revision of Iowa’s criminal laws in 1978. See generally Eggman, 311 N.W.2d at 80 (“Revised criminal code offenses are to be construed as altering prior law only if a legislative intent to change the prior law is clear.”); Emery v. Fenton, 266 N.W.2d 6, 8 (Iowa 1978) (stating that the criminal law revision “is primarily a restatement of [the prior] law”). Prior to the criminal code revisions, the crime of larceny, see Iowa Code § 709.1 (1977), and the separate crime of larceny of a motor vehicle, see id. § 321.82, required proof of an intent to permanently deprive the owner of the stolen property or vehicle. See Brainard v. State, 222 N.W.2d 711, 721 (Iowa 1974); State v. Boggs, 181 Iowa 358, 360, 164 N.W. 759, 760 (1917). In one of the few cases in which this court has addressed the issue of intent since the criminal code revisions, we held .that the record must demonstrate more than an intent to temporarily deprive the owner of the property in order to prove a theft. See Fluhr, 287 N.W.2d at 867.1

Because proof that the defendant acted with the specific purpose of depriving the owner of his property requires a determination of what the defendant was thinking when an act was done, it is seldom capable of being established with direct evidence. See State v. Fratzke, 446 N.W.2d 781, 783 (Iowa 1989). Therefore, the facts and circumstances surrounding the act, as well as any reasonable inferences to be drawn from those facts and circumstances, may be relied upon to ascertain the defendant’s intent. See State v. Nance, 533 N.W.2d 557, 562 (Iowa 1995). Accordingly, we examine the record for facts and circumstances that would support an inference that Sehminkey intended to permanently take possession of the truck.

*790The record upon which the trial court could draw to determine whether a factual basis existed for Schminkey’s plea of guilty to the theft charge was minimal. Because Schminkey entered an Alford, plea, he made no admissions with respect to his commission of this crime. The county attorney made no factual statements. The presentence report had not been completed. The court merely had before it the minutes of testimony. See generally State v. Townsend, 238 N.W.2d 351, 355 (Iowa 1976) (holding factual basis for an Alford plea may be determined from the minutes).

From the minutes of testimony, it can be established that at 7 p.m. on the day of the accident, Schminkey accompanied a friend to a party where he consumed several beers. He eventually left that party and went to a bar where he drank more alcohol. Later in the evening, at approximately 10:45 p.m., Schminkey was seen driving a pickup from Blairstown to Van Horne. Minutes later, he was involved in the accident described above, and then crashed the vehicle into a fence a block or two from the accident scene. The minutes also show that the owner of the pickup had parked the vehicle in Blairstown and had not given Schminkey permission to drive it.

We find no facts or circumstances in this recitation that would allow an inference that Schminkey intended to permanently deprive the owner of his vehicle. In a similar case that arose under Iowa’s old larceny statute, this court reversed a conviction based on a guilty plea in part on the ground that the trial court could not have found a factual basis for the defendant’s guilty plea because the record did not establish a factual basis for the intent element of the crime. Brainard, 222 N.W.2d at 721 (plurality opinion). In Brainard, the defendant admitted in the plea colloquy that he took another’s automobile without permission. Id. at 720. He denied that he intended to sell the car, but he admitted that he did not intend to bring it back. Id. This court stated that “[t]he essential question as to [the defendant’s] intent is whether he intended to deprive the owner permanently of his automobile.” Id. at 721. We concluded the record did not show a factual basis for finding that the defendant had that intent. Id. Without that intent, we noted, he would at most be guilty of the lesser offense of operating a motor vehicle without the owner’s consent. Id.

Under analogous circumstances, the court of appeals has also concluded the record lacked a factual basis for a finding of intent. State v. Henning, 299 N.W.2d 909, 911 (Iowa App.1980). In Henning, the defendant pled guilty to a charge of assault with intent to inflict serious injury. Id. at 909. During the plea colloquy, he acknowledged being involved in a fight, but stated that he did not otherwise remember the event. Id. at 911. The court of appeals found the record adequate to show a factual basis for the defendant’s acts, but not to show a factual basis for the requisite intent. Id.

Other states have, under similar facts, reached the same result as did our courts in Brainard and Henning. E.g., Pottinger v. State, 122 Fla. 405, 165 So. 276, 277 (1936) (holding the evidence was insufficient to support a finding that the defendant intended to permanently deprive another of his automobile, where the record showed the defendant, after an evening of drinking, took the ear of another, which he then drove to other bars); Gibson v. State, 8 Md.App. 1, 256 A.2d 890, 892 (1969) (reversing guilty verdict on theft charge because evidence did not support finding of intent to permanently deprive owner of vehicle, where defendant took car to drive himself home after escaping from police custody); Slay v. State, 241 So.2d 362, 364 (Miss.1970) (holding evidence that defendant took another’s car and drove it around town, eventually wrecking it while being pursued by the police, did not support a finding of intent to permanently deprive the owner of his car); cf. Fluhr, *791287 N.W.2d at 866-67 (holding, in challenge to a guilty plea, that the record did not show a factual basis for the intent element of theft where the defendant merely admitted to taking car parts belonging to another and the parts were later found in the defendant’s garage, there being no indication that the defendant intended to keep the property for any particular length of time).

We acknowledge the general statements made in some of our prior cases that possession of stolen property creates an inference supporting a conviction of larceny. See State v. Rosewall, 239 N.W.2d 171, 174 (Iowa 1976); State v. Everett, 157 N.W.2d 144, 146 (Iowa 1968), overruled on other grounds by State v. Hawkins, 203 N.W.2d 555, 556 (Iowa 1973); State v. Brightman, 252 Iowa 1278, 1284, 110 N.W.2d 315, 318 (1961); State v. Girdler, 251 Iowa 868, 873, 102 N.W.2d 877, 879 (1960). In none of these cases, however, was the defendant’s intent at issue. In Brightman, a suit stolen from a dry cleaning business was discovered in the defendant’s home six months after it had been taken. 252 Iowa at 1280-81, 110 N.W.2d at 316. The defendant claimed that he had purchased the suit from an over-the-road trucker. Id. at 1285, 110 N.W.2d at 319. The element of larceny at issue in this case was whether the defendant took the suit from the cleaners; there was no discussion of his intent. The same analysis is applicable to the Roseivall and Girdler cases; the defendant’s intent was not an issue in these cases either. See Rosewall, 239 N.W.2d at 174 (“Defendants here challenge only the ‘taking’ element of the larceny charge.”); Girdler, 251 Iowa at 872-73, 102 N.W.2d at 879 (discussing sufficiency of the evidence to prove the defendant, an escapee from prison, knew that another escapee had stolen the vehicle in which the defendant was apprehended; the defendant’s intent was not mentioned as being in dispute). Finally, we also think the Everett case is not helpful. In that case, the defendant was accused of taking a vehicle from a used car lot. Everett, 157 N.W.2d at 145. The defendant claimed he had borrowed the vehicle from a bartender with whom he was acquainted. Id. at 146. Although the court discusses the sufficiency of the evidence with respect to the defendant’s “intent to steal” the vehicle, id., it is apparent from the discussion of the evidence that the real dispute was whether he took the vehicle from the used car lot or legitimately borrowed it from a friend. Thus, the court did not really focus on the precise issue that confronts us in the present case, namely, where there is an admitted taking, is that sufficient, standing alone, to support an inference of an intent to permanently deprive the owner of his vehicle.

Because the Brainard and Henning cases are more on point factually and because the courts in those cases focused on the evidence necessary to support a finding of intent, we conclude the principles applied in Brainard and Henning should govern our analysis here. Accordingly, the mere fact that Schminkey took the pickup without the owner’s consent does not give rise to an inference that he intended to permanently deprive the owner of the vehicle.

In our search for other facts or circumstances that might reveal Schminkey’s intent in taking the pickup, we find none indicating that he intended to do anything more than temporarily use the vehicle to go home or to another bar. Because Schminkey wrecked the pickup before he could dispose of it, we do not have the typical inferences that can be drawn from a defendant’s actions subsequent to the taking. Compare Slay, 241 So.2d at 364 (holding the evidence was insufficient to prove that the defendant had an intent to permanently deprive the owner of his car, stating “the extent of damage to the car was of no probative value on the issue of specific intent, since its wrecking was not purposeful”), with People v. Graham, 27 Ill.App.3d 408, 327 N.E.2d 261, 264 (1975) (finding evidence of intent to permanently deprive owner of vehicle sufficient where *792defendant had changed the license plates on the car); State v. Keeler, 238 Kan. 356, 710 P.2d 1279, 1283 (1985) (holding evidence sufficient to prove intent to permanently deprive owner of car where defendant used the vehicle for several days and then abandoned it); State v. Winkelmann, 761 S.W.2d 702, 708 (Mo.Ct.App.1988) (finding sufficient evidence of an intent to permanently deprive owner of her car where the defendant intentionally drove the car into a brick wall, inflicting severe damage to the vehicle). Furthermore, the record contains no admissions by the defendant or statements from other witnesses that would indicate Schminkey’s purpose in taking the vehicle. Under these circumstances, we conclude the record does not show a factual basis for Schminkey’s guilty plea to the charge of theft of a motor vehicle.

IV. Disposition.

Where a guilty plea has no factual basis in the record, two possible remedies exist. Where the record establishes that the defendant was charged with the wrong crime, we have vacated the judgment of conviction and sentence and remanded for dismissal of the charge. See, e.g., Hack, 545 N.W.2d at 263; State v. Schoelerman, 315 N.W.2d 67, 75 (Iowa 1982). Where, however, it is possible that a factual basis could be shown, it is more appropriate merely to vacate the sentence and remand for further proceedings to give the State an opportunity to establish a factual basis. See Burtlow, 299 N.W.2d at 670; Ryan v. Iowa State Penitentiary, 218 N.W.2d 616, 620 (Iowa 1974).

We think this case falls within the latter category. There may be additional facts and circumstances that do not appear in the minutes of testimony that would support an inference that the defendant intended to permanently deprive the pickup’s owner of his vehicle. Therefore, we vacate the sentence entered on the theft charge and remand for further proceedings at which time the State may supplement the record to establish a factual basis for the crime of theft of a motor vehicle. If a factual basis is not shown, the defendant’s plea must be set aside.

Our vacation of the sentence entered on the theft conviction makes it unnecessary to address the defendant’s claim that the county attorney breached the plea agreement by failing to recommend concurrent sentences. If a factual basis for the theft charge is established on remand, we trust that the county attorney will comply with the plea agreement by expressly recommending concurrent sentences at any future sentencing hearing.

DECISION OF COURT OF APPEALS VACATED; SENTENCE ON THEFT CHARGE VACATED AND REMANDED FOR FURTHER PROCEEDINGS.

All justices concur except CARTER, J., and McGIVERIN, C.J., and HARRIS and LARSON, JJ., who dissent.

CARTER, Justice

(dissenting).

I dissent.

The record made before the district court during the guilty-plea proceeding adequately demonstrated a factual basis for the charge of theft of a motor vehicle. The minutes of testimony indicated that (1) the owner of a 1978 brown and tan Chevrolet pickup truck would testify that the vehicle was taken without permission on the evening of May 17, 1997; and (2) a Benton County deputy sheriff would testify that he was close enough to a fatal collision involving the stolen pickup to hear the sound of the impact and moments later arrived at the crash scene to find defendant sitting crossways, by himself, in the front seat of the missing pickup. Minutes of testimony are appropriate sources for establishing a factual basis under the decisions of this court. See State v. Fluhr, 287 N.W.2d 857, 868 (Iowa 1980), overruled on other issues by State v. Kirchoff, 452 N.W.2d 801, 804-05 (Iowa 1990); State v. Marsan, 221 N.W.2d 278, 280 (Iowa 1974).

*793When a defendant pleads guilty, strong evidence of actual guilt is not required even if the defendant protests his innocence. United States v. Tunning, 69 F.3d 107, 111-12 (6th Cir.1995). Several courts that have considered the standard that should be applied in determining the factual basis for a guilty plea have agreed that the standard should be enough evidence to withstand a directed verdict at trial. United States v. Webb, 433 F.2d 400, 403 (1st Cir.1970); In re Guilty Plea Cases, 395 Mich. 96, 235 N.W.2d 132, 145 (1975); State v. Genereux, 272 N.W.2d 33, 34 (Minn.1978).

I submit that indicia of a prima facie case is the only practical standard to be employed because to require a greater showing will inappropriately • involve the court in factual determinations based on the type of abbreviated factual record that can be made available at a guilty-plea proceeding. This is particularly true in this ease because the element under consideration is the defendant’s intent, which is seldom capable of direct proof. State v. Chang, 587 N.W.2d 459, 462 (Iowa 1998). To require a greater showing will also require the court to substitute its judgment for that of defendant and defendant’s counsel concerning whether defendant would in fact be convicted if a trial were held.

If the indicia of a prima facie case is utilized as the standard for establishing a factual basis, the minutes of testimony in the present case were sufficient indicia that the State had a prima facie case. As a general proposition in prosecutions charging theft, possession of property recently taken without permission establishes a prima facie case that a theft has been perpetrated by the possessor of the missing property. See State v. Brightman, 252 Iowa 1278, 1284, 110 N.W.2d 315, 316-18 (1961). This court has consistently applied this rule in cases involving prosecutions for theft of a motor vehicle. State v. Rosewall, 239 N.W.2d 171, 173-74 (Iowa 1976); State v. Everett, 157 N.W.2d 144, 146 (Iowa 1968), overruled on other issues by State v. Hawkins, 203 N.W.2d 555, 556-57 (Iowa 1973); State v. Girdler, 251 Iowa 868, 873, 102 N.W.2d 877, 879 (1960).

A helpful illustration of the application of this principle in a vehicle theft situation similar to the present case is found in Everett. There, the evidence showed that a motor vehicle had been left on the lot of a used car dealer at the close of business on October 18, 1965. On that evening, that vehicle was observed being driven on the streets of Cedar Rapids and later parked behind a tavern in Cedar Rapids. Still later in the evening, defendant was apprehended while driving the vehicle. Defendant was convicted at trial of larceny of a motor vehicle. On appeal the presumption of guilt to which I have referred was utilized as the basis for upholding the conviction notwithstanding defendant’s testimony that he had only borrowed the car.

The majority attempts to distinguish these cases on the basis that the intent to keep the property was not made an issue on those appeals. This overlooks the fact that the inference of guilt applied therein encompassed all elements of the offense of larceny of a motor vehicle. This was expressly .recognized in Rosewall in which this court states:

Under our holding in Everett the admitted possession by defendants of the recently stolen motorcycle creates an inference barring a motion for directed verdict. The inference is not limited to any particular element of larceny. When it arises guilt of the crime of larceny is inferred.

Rosewall, 239 N.W.2d at 174. The elements of the former statutory crime of larceny of a motor vehicle are sufficiently similar to the present statutory crime of theft that the principle established in the cases under discussion should be applied in the same manner.

In concluding that the showing of a factual basis in the present case was inadequate, the majority opinion misapplies our *794holding in Brainard v. State, 222 N.W.2d 711 (Iowa 1974). In Brainard the court set forth detailed standards for establishing a knowing and voluntary plea of guilty through direct colloquy between the court and the defendant. Although Brainard was a prosecution for larceny of a motor vehicle and there are statements in the opinion that the factual basis presented was inadequate, the court’s finding of inadequacy is with regard to the limited extent of the colloquy with the defendant to establish the intent with which he had acted. The case cannot be a holding that the factual basis presented was inadequate in toto because the specially concurring opinion necessary for the five-person majority expressly declined to find an inadequate factual basis and expressed the view that an adequate factual basis had been shown. The discussion of factual basis in Fluhr, 287 N.W.2d at 867, should also be viewed as a discussion of what was deemed an inadequate colloquy with the defendant in attempting to gain his admission concerning the intent element of the crime. This is borne out by the fact that the court ordered that defendant be permitted to plead anew (a remedy for an uninformed guilty plea) as contrasted to remanding the case to permit the State to further show a factual basis.

The court’s primary concern regarding factual basis in Fluhr was stated as follows:

Nor do the facts that the [written] plea form indicated that defendant had discussed the elements and facts of the crime with his attorney and that the attorney certified, on a separate form, that he was satisfied that the plea was factually justified overcome any of the plea’s deficiencies.... [R]eliance by a trial court upon an attorney’s declaration which fails to detail the facts upon which he bases his conclusion constitutes an impermissible delegation by the court of its duty to determine the existence of a factual basis.

Fluhr, 287 N.W.2d at 867. The Fluhr opinion does state that the minutes of testimony in that case were inadequate to show that defendant intended to keep the property that was the subject of the theft charge to which he had pleaded guilty. I submit that the court was only declaring that the minutes of testimony would not be considered a substitute for personal inquiry of the defendant concerning criminal intent when the defendant before the court had recollection of the circumstances of the crime and had not declared an intent to enter an Alford plea. The court makes no reference to departing from the general rule that indicia of a prima facie case satisfies the factual-basis element. If the court did find the minutes of testimony were inadequate to show factual basis on a record that contained indicia of a prima facie case, I submit that this was an incorrect statement of the law. It was also a comment that was not necessary to the opinion because of the court’s election to permit the defendant to plead anew as the result of not having been fully informed of the elements of the crime.

The present case differs substantially from Brainard and Fluhr because the colloquy between the court and the defendant concerning the intent element of the crime is not an issue here. The defendant disavowed any memory of the circumstances surrounding the alleged crime. In such circumstances, factual basis should be deemed to have been adequately shown through minutes of testimony that give indicia of a prima facie case.

The holding of the majority in the present case, which purports to be based on the best interests of the defendant, will instead deny this defendant and other defendants similarly situated an opportunity to make a plea bargain notwithstanding the fact that the State has made a prima facie showing of guilt on one of the charges to which the defendant has agreed to plead guilty. I would hold that the showing of factual basis in the present case was ade*795quate and proceed to consider the other issues raised on appeal.

McGIVERIN, C.J., and HARRIS, and LARSON, JJ. join this dissent.

State v. Morris State v. Morris

STATE of Iowa, Appellee, v. Willis Elbert MORRIS, Appellant.

No. 02-1033.

Supreme Court of Iowa.

April 7, 2004.

Linda Del Gallo, State Appellate Defender, and Robert P. Ranshau, Assistant State Appellate Defender, for appellant.

Thomas J. Miller, Attorney General, Martha E. Boesen, Assistant Attorney General, Thomas J. Ferguson, County Attorney, James Katcher, Assistant County Attorney, for appellee.

PER CURIAM.

Defendant, Willis Elbert Morris, appeals from judgment and sentence convicting him of second-degree theft as a habitual offender in violation of Iowa Code sections 714.2(2), 902.8, and 902.9(3) (2001). The alleged theft involved a motor vehicle. He contends that the State failed to produce sufficient evidence that he intended to permanently deprive the owner of possession of the motor vehicle, an essential element of theft involving a motor vehicle. The court of appeals agreed with that contention and reversed defendant’s conviction for theft. After reviewing the record and considering the arguments presented, we affirm the decision of the court of appeals. The judgment of the district court is reversed and remanded.

On October 6, 2001, at 4:30 a.m., Brian Gonzales started the engine on his truck parked on the street in front of his home. He intended to let it warm up before driving to work. Gonzales reentered his home leaving the engine running. Shortly thereafter, he heard a “revving” of the truck’s *788engine, looked outside, and saw someone driving his truck away.

'Gonzales called Waterloo police and reported the taking of the vehicle. Officer Aaron McClelland arrived at the scene and took the necessary information from Gonzales, including a description of the truck. The officer then put out an “attempt to locate” call to Waterloo patrol officers for the location of a brown Ford Ranger pickup truck.

Approximately thirty minutes later, McClelland came upon the missing truck about five miles from Gonzales’s residence. The truck was being driven in the opposite direction from that of the officer’s vehicle. McClelland turned his police vehicle around and gave pursuit. The person driving Gonzales’s truck stopped the vehicle, got out, and fled on foot toward nearby houses. Officer McClelland radioed for backup and a K-9 unit responded. One of the dogs alerted police to the presence of a person hiding on a porch of a house. McClelland identified this person as the man who had fled from the truck. That person was ultimately determined to be defendant, Willis -Elbert Morris.

Defendant was arrested and charged with second-degree theft pursuant to Iowa Code section 714.2(2) (the motor vehicle version), a class “D” felony. He was also charged as a habitual offender, pursuant to Iówa Code section 902.8. The second-degree theft charge was submitted to the jury along with the lesser-ineluded offense of operating a motor vehicle without the owner’s consent in violation of Iowa Code section 714.7. The jury found defendant guilty of second-degree theft. He ultimately stipulated to the prior offenses on which the habitual-offender sentencing enhancement was based.

The court of appeals, relying on our decision in State v. Schminkey, 597 N.W.2d 785 (Iowa 1999), concluded that the mere fact that defendant took Gonzales’s pickup truck without consent of the owner did not give rise to an inference that he intended to permanently deprive the owner of the vehicle. The court further concluded that the evidence, viewed in its entirety, was insufficient to permit a finding of the requisite intent beyond a reasonable doubt. We agree with the court of appeals’ reading of Schminkey and its application to the present facts.

Although apprehension of the suspect within a short time of the taking of the vehicle does not defeat the possibility that there was an intent to permanently deprive the owner of the property at the time of the taking, it is a circumstance that severely limits the circumstantial evidence from which that intent can be inferred. The State urges that the circumstances under which the defendant abandoned the vehicle, i.e., stopping it and running away, are indicative of the requisite intent. We disagree. Abandoning the vehicle and fleeing upon observing the presence of police was an act that would ordinarily assure that the truck would be returned to its owner. We affirm the decision of the court of appeals holding that the evidence was insufficient to support a conviction for second-degree theft.

The court of appeals simply remanded the case for further proceedings not inconsistent with its opinion. We believe a more specific remand mandate is warranted. The offense of operating a motor vehicle without the owner’s consent in violation of Iowa Code section 714.7 was submitted to the jury as a lesser-ineluded offense. The jury did not reach a verdict on that offense because it found that the State had established all elements of the greater offense. In so doing, the jury necessarily found that the State had established all elements of the included offense. *789In such instances, we have approved entering an amended judgment of conviction with respect to the lesser-included offense. State v. Pace, 602 N.W.2d 764, 774 (Iowa 1999); see also State v. Lampman, 342 N.W.2d 77, 81 (Iowa Ct.App.1983); 5 Am. Jur.2d Appellate Revieiv §§ 836, 844 (1995). We order that this be done following remand in the present case. Defendant shall then be resentenced according to law.

DECISION OF COURT OF APPEALS AFFIRMED; DISTRICT COURT JUDGMENT REVERSED AND REMANDED.

All justices concur except CARTER, J., who concurs specially, and LARSON, CADY, and STREIT, JJ., who dissent.

CARTER, Justice

(concurring specially).

Although I am troubled with the decision of the court in State v. Schminkey, 597 N.W.2d 785 (Iowa 1999), for the reasons set forth in my dissent in that case I do not agree that that decision only applies to guilty pleas, as the dissent in the present case suggests.

Schminkey involved the taking of an Alford plea in which the defendant refused to admit guilt. When this occurs, the court, before accepting the plea, must determine whether the evidence available to the State would be sufficient to sustain a guilty verdict at trial. That is the same analysis that must be undertaken in ruling on a motion for judgment of acquittal. For this reason, Schminkey is valid authority for the issue presently before the court.

I believe Schminkey failed to recognize the strength of the inference that arises from taking an automobile without the owner’s consent. An individual’s intent is synonymous with what he wants to do. What he wants to do is usually motivated by and derived from the actor’s own best interests. It is in the best interests of one taking another’s automobile without permission that the vehicle not be returned to the owner because this may aid in the identification of the perpetrator.

Notwithstanding my conclusions in this regard, I concur in the result because I am unable to find that the totality of the evidence in the present case, including the presumption to which I refer, is sufficient to sustain a verdict of guilty on the greater charge. The defendant should be declared guilty of the lesser-included offense.

LARSON, Justice

(dissenting).

I dissent because the plurality fails to recognize and apply the well-established principle that a defendant’s possession of recently stolen property creates an inference that the defendant stole it. In concluding that proof of intent was not established in this case, the plurality expressly relies on State v. Schminkey, 597 N.W.2d 785 (Iowa 1999), stating, “fw]e agree with the court of appeals’ reading of Schminkey and its application to the present facts.”

Even if the holding in Schminkey, was correct, I disagree with the plurality’s reliance on it in this case. Both Schminkey and the present case involve the sufficiency of evidence on the element of intent to deprive the owner of possession of a vehicle. With that, however, the similarity ends. Schminkey was a guilty-plea case, while the present case was tried to a jury. In a guilty-plea case, the State may not rely on inferences to establish a factual basis. In contrast, a jury-tried case is one in which inferences are standard stock-in-trade. See County Court of Ulster County, New York v. Allen, 442 U.S. 140, 99 S.Ct. 2213, 60 L.Ed.2d 777 (1979).

Inferences and presumptions are a staple of our adversary system of fact-*790finding. It is often necessary for the trier of fact to determine the existencé of an element of the crime — that is, an “ultimate” or “elemental” fact — from the existence of one or more “evidentiary” or “basic” facts.

Id. at 156, 99 S.Ct. at 2224, 60 L.Ed.2d at 791. The Supreme Court has further said that:

The most common evidentiary device is the entirely permissive inference or presumption, which allows — but does not require — the trier of fact to infer the elemental fact from proof by the prosecutor of the basic one and that places no burden of any kind on the defendant. In that situation the basic fact may constitute prima facie evidence of the elemental fact. When reviewing this type of device, the Court has required the party challenging it to demonstrate its invalidity as applied to him.

Id. at 157, 99 S.Ct. at 2224-25, 60 L.Ed.2d at 792 (emphasis added) (citations omitted).

In a guilty-plea case, it is not permissible for a judge to fill gaps in the factual-basis record by inferring elements, of the crime. The difference between guilty-plea cases and fact-tried cases vis-a-vis the function of inferences is significant. In one case, the court observed:

The government argues that this court can infer that [the defendant] had an intent to defraud when he obtained the American Express card using a false name and a false date of birth. Although we agree that a rational fact finder could infer an intent to. defraud from such evidence, in the context of a challenge to the factual basis supporting a guilty plea, we have previously rejected a request by the government to infer a “critical element” of the offense charged.

United States v. Tunning, 69 F.3d 107, 113 (6th Cir.1995) (emphasis added) (citation omitted); see also McCarthy v. United States, 394 U.S. 459, 467, 89 S.Ct. 1166, 1171, 22 L.Ed.2d 418, 426 (1969) (holding that a factual-basis record on a guilty plea cannot be supplemented by a district court’s “assumptions” of facts). In United States v. Goldberg, 862 F.2d 101 (6th Cir.1988), the court stated:

In our judgment, to permit the district court to infer a factual basis in the absence of a record demonstrating the existence of a factual basis would tend to negate the well-established safeguards inherent in the [federal guilty-plea-rule] mandate.

Goldberg, 862 F.2d at 106.

The problem for the state in motor-vehicle larceny cases is that it must prove the defendant knowingly took the vehicle, but it also must prove the defendant took it with the intent to deprive the owner of it. Establishing the first element does not establish the second. We have so held in guilty-plea cases involving theft, holding a factual basis on intent was not established. Schminkey, of course, was one of them. In addition, other cases support that proposition. See, e.g., State v. Auerbach, 283 N.W.2d 278, 280 (Iowa 1979); Brainard v. State, 222 N.W.2d 711, 721 (Iowa 1974). However, in cases that have gone to trial, we have accepted inferences arising from possession of recently stolen property as a means of proving theft. See, e.g., State v. Rosewall, 239 N.W.2d 171, 173 (Iowa 1976); State v. Morrison, 183 N.W.2d 696, 697 (Iowa 1971); State v. Everett, 157 N.W.2d 144, 146 (Iowa 1968). The reliance on use of inferences recognizes the difficulty of proving subjective intent by direct evidence.

In the past, it was customary for our courts to instruct specifically on the inferences to be drawn from possession of re*791cently stolen property. See, e.g., Morrison, 183 N.W.2d at 697. This practice, however, has been criticized:

The instruction on the inferences to be drawn from possession of recently stolen property should be condemned. It is nothing more nor less than a singling out of, and judicial comment on, a specific item of evidence.

Id. at 699 (Becker, J., dissenting). Now, juries are presumably instructed in a manner that does not emphasize the defendant’s possession of stolen property. Under our uniform instructions, that intent may be based on “circumstances” in the case, but the instructions do not refer to the defendant’s possession of recently stolen property as the basis for a presumption. This sanitized form of the instruction was given in the present case, as I will discuss later.

In this case, the court instructed the jury on the elements of both larceny and the lesser offense of operating a vehicle without the owner’s consent. The distinguishing feature of the theft charge is the element of intent to deprive the owner of the property. The jury was instructed that, to prove the higher offense of larceny, the State must prove the defendant acted “with the specific intent to deprive [the owner] of the automobile.” “Deprivation” was defined in a separate instruction:

To “deprive the [owner]” of the property, means to withhold permanently or to withhold for so long or under such circumstances the value or benefit of the property is lost to the owner. If a person disposes of property so that it is unlikely the owner will recover it, you may conclude, but are not required to conclude, that the person disposed of the property with the intent to permanently deprive the owner of its use.

The key element of larceny, the intent to deprive, was explained in this instruction:

Specific intent means not only being aware of doing an act and doing it voluntarily, but also doing it with a specific purpose in mind. Specific intent need not exist for any particular length of time before the act.
Determining a defendant’s specific intent requires you to decide what the defendant was thinking when the defendant did an act. It is seldom capable of direct proof. Consider the facts and circumstances surrounding the act to determine the defendant’s specific intent. You may conclude, but are not required to conclude, that people intend the natural results of their acts.

(Emphasis added.) This instruction, substantially the same as Uniform Criminal Instruction 200.2, incorporates the concept of inferences.

The facts presented at trial are briefly stated in the plurality opinion. It is undisputed that Morris took the truck, at about 4:30 a.m., without the owner’s consent. The owner immediately called the police who soon located the vehicle and began to give chase. When the police stopped the vehicle, Morris fled on foot. These facts are clearly sufficient to give rise to an inference of guilt, including the element of intent.

The Supreme Court has discussed the nature and limitations on inferences and their status under the Constitution:

[T]he entirely permissive inference or presumption ... allows — but does not require — the trier of fact to infer the elemental fact from proof by the prosecutor of the basic one and that places no burden of any kind on the defendant. ... Because this permissive presumption leaves the trier of fact free to credit or reject the inference and does not shift the burden of proof, it affects the application of the “beyond a reason*792able doubt” standard only if, under the facts of the case, there is no rational way the trier could make the connection permitted by the inference. For only in that situation is there any risk that an explanation of the permissible inference to a jury, or its use by a jury, has caused the presumptively rational factfinder to make an erroneous factual determination.

Allen, 442 U.S. at 157, 99 S.Ct. at 2224-25, 60 L.Ed.2d at 792.

The plurality points to facts it contends militate against proof of an intent to deprive the owner of possession. These facts include the distance Morris traveled with the truck (approximately five miles) before he was apprehended and the time he had the truck (approximately thirty minutes). These facts are not necessarily helpful to Morris’s case, however; his relatively short time of possession might well have been viewed by the jury to be the result of speedy police work — not evidence that he lacked the intent to keep it. In any event, that was a question for the jury. While the facts relied on by Morris may affect the strength of the inference, they do not preclude, as a matter of law, an inference of intent. There is clearly a “rational way the [jury] could make the connection permitted by the inference” under the Allen test.

I believe the plurality erred in relying on Schminkey, in refusing to recognize the inference of intent, and in failing to conclude that sufficient evidence supports the conviction. I would vacate the decision of the court of appeals and affirm the judgment of the district court.

CADY and STREIT, JJ., join this dissent.

Notes and Questions Notes and Questions

Schminkey and Morris provide examples of an offense that requires specific intent: the defendant must not only possess general intent (i.e., intent to do the conduct of driving another’s vehicle) but also intend to achieve some harmful result or consequence defined in the statute, such as depriving another of their property. Compare this to the mens rea requirements for passive child abuse cases described in Sargent. In those cases, the defendant must intend his conduct and be criminally negligent about the risk of some consequence—harm to a child. In specific intent offenses, by contrast, the defendant must intend his conduct and intend some result. Both are combinations are ways designed to ensure that only blameworthy people doing intentional conduct are held criminally liable. If the specific intent requirement were removed from this offense, can you think of a scenario in which a morally blameless person might be eligible for punishment?

For some offenses, specific intent serves not simply to ensure that a person is blameworthy but to identify those who are more blameworthy than other offenders. That is, specific intent is one tool for grading offenses—for defining relatively more wrongful conduct compared to other, lesser (but still criminal) conduct. Assault statutes provide a common example.

A standard definition of assault is something like “striking another in a manner likely to cause injury”—a general intent offense. But a graver assault offense carrying a harsher punishment might add a specific intent requirement: “striking another with the intent to kill or cause serious injury.” You see the specific intent requirement serving a similar function for the offenses at issue in Schminkey and Morris. Auto theft is more serious than the crime of “joyriding” because the thief specifically intends to permanently deprive the owner of the vehicle. The joyrider intends to do so only for a while.

In the abstract, it might seem difficult to prove beyond a reasonable doubt (absent a confession) what was in someone’s head—what they knew and intended. But as Henry David Thoreau said, “Some circumstantial evidence is very strong, as when you find a trout in the milk.” (That was back in the days in which unscrupulous dairy farmers might try to sell large vats of milk diluted with river water.) Often, it is quite easy to infer a person’s intent from their conduct and circumstances, no matter their denial. How hard was it for the prosecution in Schminkey or Morris to prove that each defendant intended his conduct of driving the vehicle? The harder question in these two cases—as you can see in the dissents—is drawing an inference about the driver’s specific intent to deprive another of their property. Notice that part of the decisions do is not only specify that the prosecution must prove the defendant’s specific intent, but also what evidence is sufficient to do that—or what conduct in what circumstances permit juries reasonably to conclude that the actor had the requisite intent.

Mens Rea Problems Mens Rea Problems

Mens Rea Problems 

Use the following criminal statutes to determine what crimes, if any, might apply to the hypotheticals posed below

S 140.17 Criminal trespass in the first degree.

  A person is guilty of criminal trespass in the first degree when he knowingly enters or remains unlawfully in a building that is the property of another.

S 140.10 Theft

  A person is guilty of theft if he unlawfully takes the property of another with the purpose to deprive him thereof.

S 140.20 Burglary in the third degree.

  A person is guilty of burglary in the third degree when he knowingly enters a building with intent to commit a crime therein.

S 140.25 Burglary in the second degree.

  A person is guilty of burglary in the second degree when he knowingly enters a building with intent to commit a crime therein, and when:

  1. In effecting entry or while in the building or in immediate flight therefrom, he or another participant in the crime is armed with explosives or a deadly weapon;

or

  2. The building is a dwelling.

 
S 140.35 Possession of burglar’s tools.

  A person is guilty of possession of burglar`s tools when he possesses any tool, instrument or other article adapted, designed or commonly used for committing or facilitating offenses involving forcible entry into premises, … under circumstances evincing an intent to use or knowledge that some person intends to use the same in the commission of an offense of such character.

 

Problem 1:

Sam has agreed to feed his friend's dog when she is away.  Forgetting the address of her house, he enters a stranger's house by mistake.  Looking around and seeing some money lying on a table, Sam takes it and leaves. Which of the above crimes, if any, might Sam have committed?

Problem 2:  

Fred, looking for something of value to steal, breaks into what he takes to be an uninhabited house under construction and starts looking around.  He is surprised to find Owen, the owner, living there. Which of the above crimes, if any, might Fred have committed?

Problem 3:

Bored one night, Jane and Larry break into the local mall after closing, as they later say, "just for kicks."  But unbeknownst to Jane, Larry has placed explosives in Jane's backpack. Which of the above crimes, if any, might Jane have committed?

Problem 4:

Mags is holding a crowbar, and says to an undercover officer, "Here's what my buddy Jack uses when he's short on cash."  When arrested, Mags says, "How can I be charged -- who knew that holding a crowbar was a crime?" Is Mags right? 

Notes on Common Law Mens Rea Notes on Common Law Mens Rea

One commits a crime with “general intent” by acting voluntarily with the knowledge that they are committing a prohibited act. (Although one need not know that the act is prohibited; more on that soon.) By contrast, one acts with “specific intent” when one, in addition to acting knowingly, desires the specific outcome or result of the act. Thus, ‘specific intent crimes’ virtually always require both general intent and specific intent—e.g., possessing cocaine (general intent) with intent to distribute it (specific intent). We can see, then, that for specific intent crimes, prosecutors bear an additional burden of proving the defendant intended their actions to result in the specified crime. (Note: We will soon see that the distinction between general intent and specific intent matters in cases in which a defendant argues that he is not guilty because he made a mistake—or held a mistaken belief—about some fact or circumstance relevant to the charged crime. Under some state criminal codes, the distinction also matters when defendants present evidence that they were too intoxicated to have the requisite intent.)

The easy specific intent offenses to identify are those statutes that includes phrases like "with intent to commit a felony" or "with intent to distribute"; the specific intent requirement is quite clear and literal. But also look for adverbs such as knowingly, purposely, recklessly, maliciously, etc. Often but not always, in the context of the full statute, these terms also are understood to impose a specific intent requirement.

Some crimes require one act merely recklessly or negligently—not with intent, specific or otherwise. When you see such statutes, however, notice they also have implicit general intent requirements, in the sense that one must be doing the conduct intentionally, but also in a reckless or negligent manner. For example: “one who drives a motor vehicle recklessly on a public road” implicitly requires that one be intentionally driving the vehicle, and also doing so in a reckless manner (say, at an excessive speed, thus disregarding the risks created by such speeding). Don’t be confused by this observation. It just states the obvious, intuitive point—one that very rarely is disputed, or even mentioned, in criminal litigation—that, in order to “act recklessly,” one must also be acting intentionally.

Finally, there is subset of criminal offenses described as “strict liability” crimes, meaning one can be liable without having any mens rea at all—no general intent, recklessness, or negligence. As we will see, these divide into two rough categories: so-called “regulatory” offenses that tend to carry modest penalties, and a smaller group of more offenses that carry more serious punishments. Statutory rape—having sexual intercourse with a person under the age of consent—is the classic example of this latter kind of a strict liability crime: one is guilty even if one did not intend to have sex with an underage person. (Thus, one might be punished for making a factual mistake—i.e., for having sex with another whom one believed was above the age of consent.) Note, however, that even in this offense—and most others described as ‘strict liability’—you could say that there is an implicit general intent requirement as to the conduct of sexual intercourse. To be guilty, one must intentionally have sex with another, and ‘strict liability’ really describes only the requirement that the other person be underage. (Granted, it’s hard to imagine how someone might unknowingly or unintentionally have sex, but that just means ‘unintentional sex’ never occurs in real life; the crime nonetheless implicitly requires intentional conduct.)

Notes on intoxication and intent Notes on intoxication and intent

What happens when an actor is too intoxicated to be meaningfully aware of his actions or to intend his conduct? The question is difficult and arises often. The law varies among U.S. jurisdictions. The Model Penal Code addresses this issue MPC § 2.08. New York Penal Code § 15.25 is an example of a statute that adopts the core of MPC 2.08 in somewhat different wording: 

§ 15.25 Effect of intoxication upon liability.

Intoxication is not, as such, a defense to a criminal charge; but in
any prosecution for an offense, evidence of intoxication of the
defendant may be offered by the defendant whenever it is relevant to negative an element of the crime charged.

Section 2.08 has been less influential with state legislatures than other MPC provisons. Many provisions in Pennsylvania's criminal code are drawn from the Model Penal Code, but not the rule on intoxication:

18 Pa. C.S. § 308.  Intoxication or drugged condition.

Neither voluntary intoxication nor voluntary drugged condition is a defense to a criminal charge, nor may evidence of such conditions be introduced to negative the element of intent of the offense, except that evidence of such intoxication or drugged condition of the defendant may be offered by the defendant whenever it is relevant to reduce murder from a higher degree to a lower degree of murder.

Notice the specification of voluntary intoxication. Many jurisdictions allow drug or alcohol intoxication to negative a general or specific intent if the defendant had been involuntarily (unknowingly, perhaps by another's deception) intoxication. See Kan. Stat. 21-5205:

(a) The fact that a person charged with a crime was in an intoxicated condition at the time the alleged crime was committed is a defense only if such condition was involuntarily produced and rendered such person substantially incapable of knowing or understanding the wrongfulness of such person's conduct and of conforming such person's conduct to the requirements of law.

MPC 2.08(4) provides an "affirmative defense" (meaning the defendant has the burden of proof) for "intoxication that (a) is not self-induced or (b) is pathological." Cases of defendants succeeding with a defense of involuntary intoxication are hard to find. Merely being an alcoholic who is severely addicted to alcohol is not recognized as involuntary intoxication. Cf. Powell v. Texas, 392 U.S. 514 (1968).

Most states  continue to follow some version of common law rules on this point. There are basically two approaches. Perhaps the traditional common law approach, still the law in some states, is that evidence of intoxication is not relevant on the issue of whether a defendant possessed "general intent," but that intoxication evidence is permitted on the issue of whether he had "specific intent"--that is, intoxication can negative specific intent. See second part of Kan. Stat. 21-5205:

(b) An act committed while in a state of voluntary intoxication is not less criminal by reason thereof, but when a particular intent or other state of mind is a necessary element to constitute a particular crime, the fact of intoxication may be taken into consideration in determining such intent or state of mind.

A good number of states, however, reject the latter half of that formulation and dictate that intoxication is relevant neither for questions of general intent nor of specific intent.

For a more thorough survey on this point, see, e.g., Modern status of the rules as to voluntary intoxication as defense to criminal charge, 8 A.L.R.3d 1236.

4.2 Mistake of Fact 4.2 Mistake of Fact

MPC and state rules on mistakes MPC and state rules on mistakes

MPC § 2.04 provides the following rules regarding the relevance of mistakes to culpability and criminal liability:

(1)  Ignorance or mistake as to a matter of fact or law is a defense if:

(a)  the ignorance or mistake negatives the purpose, knowledge, belief, recklessness or negligence required to establish a material element of the offense; or

(b)  the law provides that the state of mind established by such ignorance or mistake constitutes a defense.

(2)  Although ignorance or mistake would otherwise afford a defense to the offense charged, the defense is not available if the defendant would be guilty of another offense had the situation been as he supposed. In such case, however, the ignorance or mistake of the defendant shall reduce the grade and degree of the offense of which he may be convicted to those of the offense of which he would be guilty had the situation been as he supposed.

(3)  A belief that conduct does not legally constitute an offense is a defense to a prosecution for that offense based upon such conduct when:

(a)  the statute or other enactment defining the offense is not known to the actor and has not been published or otherwise reasonably made available prior to the conduct alleged; or

(b)  he acts in reasonable reliance upon an official statement of the law, afterward determined to be invalid or erroneous, contained in (i) a statute or other enactment; (ii) a judicial decision, opinion or judgment; (iii) an administrative order or grant of permission; or (iv) an official interpretation of the public officer or body charged by law with responsibility for the interpretation, administration or enforcement of the law defining the offense.

[You may benefit from reading the MPC Commentary to this provlsion as well, available on Lexis and Hein.]

Compare 18 Pa. C.S. § 304. Ignorance or mistake.

Ignorance or mistake as to a matter of fact, for which there is reasonable explanation or excuse, is a defense if:

(1)  the ignorance or mistake negatives the intent, knowledge, belief, recklessness, or negligence required to establish a material element of the offense; or

(2)  the law provides that the state of mind established by such ignorance or mistake constitutes a defense.

Why might the Pennsylvania legislature, which adopted much of the MPC verbatim, opted for this statute on mistakes instead?

You might ask the same question of the New York legislature after reviewing N.Y. Penal Law § 15.20, which likewise is based upon, but differs somewhat from, the MPC § 2.04.

United States v. Oglivie United States v. Oglivie

UNITED STATES, Appellee, v. Staff Sergeant Mauricio A. OGLIVIE, [ XXX-XX-XXXX ], United States Army, Appellant.

ACMR 8901244.

U.S. Army Court of Military Review.

14 Feb. 1990.

*1070For Appellant: Major Peter V. Train, JAGC US AR, Captain Thomas A. Sieg, JAGC, Captain Alan M. Boyd, JAGC, Captain Robin K. Neff, JAGC, Captain Jeannine C. Hinman, JAGC (on brief).

For Appellee: Colonel Alfred F. Arquilla, JAGC, Lieutenant Colonel Daniel J. Dell’Orto, JAGC, Major Gary L. Hausken, JAGC, Major Martin D. Carpenter, JAGC, Captain Randy V. Cargill, JAGC, Captain Jonathan F. Potter, JAGC (on brief).

Before FOREMAN, SMITH, and VARO, Appellate Military Judges.

OPINION OF THE COURT

FOREMAN, Senior Judge:

A military judge sitting as a special court-martial convicted the appellant, in accordance with his plea of guilty, of altering a public record in violation of Article 134, Uniform Code of Military Justice, 10 U.S.C. § 934 (1982) [hereinafter UCMJ], Contrary to his pleas, the military judge also convicted the appellant of signing a false official statement (two specifications), wrongful appropriation of a copy of a divorce decree, and bigamy, in violation of Articles 107, 121 and 134, UCMJ, 10 U.S.C. §§ 907, 921 and 934 (1982). His approved sentence provides for a bad-conduct discharge, confinement for three months, and reduction to Private E1.

The appellant married his first wife, Amparo, in December 1986, while stationed in Panama. In January 1987, the appellant was reassigned from Panama to Germany, but his wife remained in Panama. While in Germany, the appellant did not know Amparo’s address or telephone number, but sent letters to a friend, who passed them on to her. The appellant returned from Germany in March 1988 and was reassigned to Fort Sill, Oklahoma. In August 1988, the appellant sent his wife a money order with his telephone number written on it. The appellant filed for divorce in Oklahoma and sent a copy of the petition to a friend’s post office box in Panama for delivery to his wife. In September or October 1988, Amparo called the appellant from Panama and informed him that she had filed for divorce in Panama, that there was “nothing between the two of us” and that he “didn’t have to worry about her anymore.” The appellant testified that he thought he was divorced at that point. In November 1988, the Red Cross notified the appellant that Amparo had been hospitalized. The Red Cross referred to Amparo as his “ex-wife.” On 9 November 1988, the appellant requested that his basic allowance for quarters (BAQ) at the “with dependents” rate be terminated because he was divorced. The finance clerk told him that she could not stop the BAQ without a divorce decree. Since the appellant did not have a copy of a divorce decree, he took another sergeant’s divorce decree, made a copy of it, inserted his name and Amparo’s in the text (but neglected to change the caption) and attached it to his request to terminate his BAQ. In December 1988, the appellant married Jackeline, and requested that his BAQ at the “with dependents” rate be reinstated.

The appellant was charged with making two false official statements to officials of the Fort Sill finance office, first that he was divorced from Amparo (Specification 1 of Charge II) and second that he was married to Jackeline (Specification 2 of Charge II). He contends that the evidence was insufficient to prove both specifications because he honestly believed that he was divorced from Amparo.

This court specified the following issues: (1) whether creation of an altered copy of a public record without altering the original constitutes the offense of altering a public record, and (2) whether the appellant’s plea of guilty to altering a public record was provident to the offense charged or any lesser included offense.

I. False Official Statements

Making a false official statement in violation of UCMJ, Article 107, 10 U.S.C. *1071§ 907 (1982), is a specific intent crime. An honest mistake of fact regarding the truth of the statement made is a defense. United States v. Rowan, 16 C.M.R. 4 (C.M.A.1954); Manual for Courts-Martial, United States, 1984, Rule for Courts-Martial 916(j) [hereinafter M.C.M., 1984 and R.C.M.]; M.C.M., Part IV, para. 31c(5). The evidence establishes that Amparo told the appellant she had filed for divorce, that the appellant received correspondence from the Red Cross referring to Amparo as his “ex-wife,” and that he attempted to terminate his entitlement to BAQ, on the ground that he was divorced. He then participated in a marriage ceremony and received a marriage certificate indicating that he was married to Jackeline. Based upon the entire record, we find that the defense of an honest mistake of fact was raised and not overcome by the government’s evidence. R.C.M. 916(b). Accordingly, we find that the evidence is insufficient to prove appellant’s guilt of making false official statements (Charge II and its two specifications).

II. Bigamy

Bigamy is a general intent crime. To constitute a defense to bigamy, a mistake of fact must be both honest and reasonable. M.C.M., 1984, Part IV, para. 65c. While the appellant may have honestly believed that he was divorced from Amparo, we find that he did not take the steps which a reasonable man would have taken to determine the validity of his honest belief. United States v. McCluskey, 20 C.M.R. 261 (C.M.A.1955). He was not reasonable in assuming that he was divorced. See United States v. Bateman, 23 C.M.R. 312 (C.M.A.1957) (knowledge that wife intended to proceed with divorce proceedings insufficient); United States v. Avery, 9 C.M.R. 648 (A.F.B.R.1953) (reliance on attorney’s prediction that divorce would occur on a given date not reasonable). Accordingly, we find that the evidence is sufficient to prove bigamy.

III. Altering a Public Record

The offense of altering a public record in violation of UCMJ, Article 134, is based on 18 U.S.C. § 2071. The offense is committed by altering, concealing, removing, mutilating, obliterating, destroying, or taking a public record with the intent to do any of the foregoing. M.C.M., 1984, Part IV, para. 99b. A public record is defined as including “records, reports, statements, or data compilations, in any form, of public offices or agencies, setting forth the activities of the office or agency, or matters observed pursuant to duty imposed by law as to which there was a duty to report.” M.C.M., 1984, Part IV, para. 99c. The offense of altering a public record in violation of Article 134 is “substantially identical” with the crime denounced by 18 U.S.C. § 2071. United States v. Spain, 38 C.M.R. 145 (C.M.A.1968). The purpose of 18 U.S.C. § 2071 is to “prevent any conduct which deprives the government of the use of its documents, be it by concealment, destruction, or removal.” United States v. Rosner, 352 F.Supp. 915, 919 (S.D.N.Y.1972), petition denied, 497 F.2d 919 (2d Cir.1974).

The government argues that the UCMJ, Article 134 offense is broader than 18 U.S.C. § 2071 and that it encompasses alteration of an unofficial copy of a public record because the Manual for Courts-Martial definition of a public record includes the words “in any form,” which are not found in 18 U.S.C. § 2071. We disagree. The UCMJ, Article 134 offense is broader than 18 U.S.C. § 2071 only in that it specifically proscribes “altering” public records. United States v. Maze, 42 C.M.R. 376, 379 (A.C.M.R.1970). The term “altering” does not appear in 18 U.S.C. § 2071. The words “in any form” are taken from Mil.R.Evid. 803. M.C.M., 1984, Article 134, Analysis, app. 21, at A21-103. This language also appears in Federal Rule of Evidence 803 [hereinafter Fed.R.Evid.], and is intended to include public records other than paper documents, e.g., microfiche, videotapes, computer tapes and disks. See e.g., Oriental Health Spa v. City of Fort Wayne, 864 F.2d 486, 490 (7th Cir.1988) (videotape admissible as an official public record under Fed.R.Evid. 803(8)); State ex rel. Harmon v. Bender, 25 Ohio St.3d 15, 494 N.E.2d *10721135, 1136 (1986) (videotapes of trial proceedings are public records). In the case before us the appellant created and altered an unofficial, unauthenticated photocopy of a public record, but did not disturb the integrity of the public record itself. We hold that the unofficial, unauthenticated' photocopy altered by the appellant was not a public record.

On the facts before us, we need not decide whether altering an authenticated copy of a public record would be a violation of UCMJ, Article 134. While intentional introduction of the altered unauthenticated photocopy into government channels may have violated another proscription, e.g., UCMJ, Article 132, 10 U.S.C. § 932 or 18 U.S.C. § 1001, it did not constitute the crime of altering a public record in violation of UCMJ, Article 134, or any lesser included offense. Accordingly, we hold that the appellant’s plea of guilty to altering a public record in violation of UCMJ, Article 134 was improvident.

The findings of guilty of Charge II and its two Specifications (false official statements), and Specification 2 of Charge IV (altering a public record) are set aside. Charge II and its two Specifications, and Specification 2 of Charge IV are dismissed. The remaining findings of guilty are affirmed. The sentence is set aside. A rehearing on the sentence may be ordered by the same or a different convening authority.

Judge SMITH and Judge VARO concur.

United States v. Binegar United States v. Binegar

UNITED STATES, Appellee, v. Charles D. BINEGAR, Senior Airman, U.S. Air Force, Appellant.

No. 00-0207.

Crim.App. No. 32854.

U.S. Court of Appeals for the Armed Forces.

Argued Oct. 10, 2000.

Decided May 4, 2001.

Gierke, J., filed opinion concurring in the result.

Crawford, Chief Judge, filed dissenting opinion.

SULLIVAN, J., delivered the opinion of the Court, in which EFFRON and BAKER, JJ., joined. GIERKE, J., filed an opinion concurring in the result. CRAWFORD, C.J., filed a dissenting opinion.

For Appellant: Captain Bryan A Bonner (argued); Colonel Jeanne M. Rueth (on brief).

For Appellee: Major Jennifer R. Rider (argued); Colonel Anthony P. Dattilo and Lieutenant Colonel Ronald A Rodgers (on brief).

Judge SULLIVAN

delivered the opinion of the Court.

In March of 1997, appellant was tried by a general court-martial composed of officer members at Hanscom Ah’ Force Base in Massachusetts. Contrary to his pleas, he was found guilty of four specifications of stealing contact lenses which were military property of the United States, and one specification of conspiring to steal those contact lenses, in violation of Articles 121 and 81, Uniform Code of Military Justice, 10 USC §§ 921 and 881, respectively. On March 28, 1997, he was sentenced to a bad-conduct discharge, 3 months of hard labor without confinement, and forfeiture of $300 pay per month for 3 months. The convening authority approved this sentence on July 24, 1997, and the Court of Criminal Appeals affirmed on November 1,1999, in an unpublished opinion.

This Court granted review in this case on two issues on April 12, 2000. They ask:

I. WHETHER THE MILITARY JUDGE ERRED IN NOT ALLOWING DEFENSE COUNSEL TO ELICIT STATE OF MIND HEARSAY EVI*2DENCE OF APPELLANT FROM A WITNESS.
II. WHETHER THE MILITARY JUDGE ERRED IN PROVIDING THE COURT MEMBERS A MISTAKE OF FACT INSTRUCTION WHERE THE COURT MEMBERS HAD TO FIND THAT APPELLANT’S MISTAKE OF FACT WAS BOTH “HONEST AND REASONABLE” INSTEAD OF JUST “HONEST.”

We hold that Issue I need not be decided in this case because, relying on United States v. Turner, 27 MJ 217 (CMA 1988), we must reverse this case on Issue II. See United States v. Gillenwater, 43 MJ 10 (1995).

Evidence was admitted in this case that shows that in September 1995, appellant began work in the Medical Logistics Office at Hanseom Air Force Base, Massachusetts. One of his duties was to order contact lenses for servicemembers who brought a prescription from the base Optometry Clinic to the Medical Logistics Office. Personnel who needed contact lenses to perform their duties or for a medical condition were entitled to receive them free of charge. Other personnel had to obtain contact lenses off-base and pay for the lenses themselves. (R. 99)

It was also shown that Air Force regulations governed the process of ordering contact lenses at the Medical Logistics Office. (R. 165) The Optometry Clinic was required to produce a purchase letter for all prescriptions sent to the Medical Logistics Office. (R. 99-102) If the lenses were required “for the performance of duties,” the Logistics Office was to code the purchase order with a “fund cite” indicating the servicemember’s section. If, however, the lenses were required for a medical condition, the Optometry Clinic fund cite was used. (R. 113, 172-73) Once a month, the office generated reports of how many lenses had been billed to each account. (R. 103, 310-11)

Evidence was further admitted that appellant’s supervisors neglected to follow these procedures with any regularity. The Optometry Clinic rarely generated purchases letters, and Medical Logistics would order contact lenses without them. One of appellant’s supervisors, Senior Master Sergeant (SMSgt) Kremer, testified that he believed at one time that all clinic personnel were entitled to free contact lenses, even if not medically required. (R. 298, 302, & 320) In fact, SMSgt Kremer instructed appellant to sign a purchase order for him to get free lenses soon after appellant began work at Medical Logistics. (R. 293-94) SMSgt Kremer did not provide appellant with a purchase letter or prescription for his lenses, although he testified that he later discussed with appellant a clarified command policy requiring a medical reason for contact lenses with some exceptions. (R. 321)

Finally, evidence was admitted that appellant continued to order contact lenses, filing the appropriate purchase orders and keeping his paperwork in order.1 Appellant’s supervisors never reviewed the monthly reports to determine whether lenses were being billed to the proper accounts. Moreover, appellant’s supervisors failed to provide appellant with formal training or specific guidelines for any of these procedures. (R. 320) The next supervisor of the office, Staff Sergeant (SSgt) Smith, even authorized appellant to sign purchase orders for him between October 1995 to January 1996. (R. 149, 161) Appellant signed SSgt Smith’s name to over 90% of the purchase orders filed between September 1995 and March 1996. Neither SSgt Smith nor his predecessor, SMSgt Kremer, ever told appellant that he was doing his job improperly. Furthermore, appellant had public conversations concerning his conduct in ordering contact lenses with the servicemembers for whom he ordered those lenses.

The record of trial shows the following concerning the defense’s request for a mistake of fact instruction:

MJ: Now, concerning the proposed mistake-of-fact instruction, I’ll hear from you *3first, defense counsel, since it’s your request.
DC: Your Honor, the defense requests that a mistake-of-fact defense be given in this case. Defense believes the issue has been raised and defense believes the issue’s been raised due to the testimony of— basically every witness at this court-martial except for Airman Beasley, whose testimony was not — and Airman Danieli. But every other witness, we believe, had something to say ivhich would lead the court members and lead anyone to believe that there could have been a mistake in this case.
MJ: Let me stop you there, counsel. I agree that a mistake-of-facts instruction of some sort should be given. So would you like to append as Appellate Exhibit X your proposed instruction?
DC: Yes, sir. I have typed up — as far as the wording of the instruction. However, as far as the witnesses, I did not put them in there. I can type up a complete one, but — or else we can append this, Your Honor. I don’t know how you would like— those are the witnesses I’d like referred to in the instruction as to — to give them an idea of why it’s been raised.
MJ: All right, well, I can certainly mark this as Appellate Exhibit X. I note this relates to specific intent.
DC: Yes, Your Honor.
MJ: Is the government in agreement with what has been marked as Appellate Exhibit X, which is ignorance, or mistake with specific intent or actual knowledge is an issue.
TC: No, sir. The language we agreed on was only to the extent that we could characterize what the mistake was. And I believe that was a blank page with just a short paragraph on “you must determine whether the accused was mistaken,” et cetera. That was the language that we agreed on to be instructed as to what the mistake was. But the government’s position is that the mistake-of-fact defense should be raised as a general intent crime because it involves whether it ivas just wrongful, or in this case that would be required to be honest and reasonable. So—
MJ: Well, what you’re suggesting is that the mistake doesn’t necessarily go to the issue of specific intent to permanently deprive; is that the idea, trial counsel?
TC: That’s correct, sir, because we don’t believe it fits the specific intent element of the offense itself.
MJ: Defense counsel?
DC: Your Honor, defense requests that the instruction be given without the instruction regarding “it must be reasonable.” The offense is larceny, which is a specific intent crime. And the fact that Airman Binegar is charged with permanently depriving the Air Force of basically contact lenses, we believe that’s what he’s defending against. And I believe, in general, since this is a specific intent offense and since it’s the defense’s responsibility to defend against that offense, the fact that— I believe it’s relevant, Your Honor, that he’s defending against not only permanently deprive — with the intent to permanently deprive the Air Force of contact lenses. And I do not — the defense does not believe that that has to be a reasonable — the defense would have to be reasonable in this case. And we ask that the instruction be given without the limiting factor of, “would have to be reasonable,” Your Honor.
MJ: Okay. My analysis of the evidence and the offenses leads me to believe that a mistake-of-fact instruction that would be given would be general, rather than the one related to a specific intent.
I note that paragraph 5-11 of the Bench-book does give some guidance in that regard, and I note that it states as follows “... moreover, in some ‘specific intent’ crimes, the alleged ignorance or mistake may not go to the element requiring specific intent or knowledge, and thus may have to be both reasonable and honest
The court feels in this particular instance that the mistake — or mistaken belief, as it were — of the accused, goes generally to the offense of larceny itself, whether that larceny be a specific intent larceny or, quite frankly, even if it were a wrongful appro*4priation larceny. I believe that the mistake concerning whether or not he was authorized to order contact lenses at government expense relates generally to the offense and is not related to that element which requires a specific intent. Therefore, I’m not going to give the instruction as propounded by defense counsel, shown in Appellate Exhibit X, although this will be attached to the record of trial.
What I am going to give is some modified version of the general intent version of mistake-of-fact. I would ask, at the next recess, that counsel try and pull together the final draft of what that instruction would encompass based upon this ruling.

(R. 361-63) (emphasis added).

The military judge gave the following instructions in this case:

As to the various charges, the evidence has raised the issue of ignorance or mistake on the part of the accused concerning whether he was under the mistaken belief that he was authorized to order contact lenses at government expense with a purchase order from military members who provided him only a prescription from the Optometry Clinic. If the accused mistakenly believed that he was authorized to order contact lenses at government expense with a purchase order for military members who provided him only a prescription from the Optometry Clinic, he is not guilty of the offense of larceny or conspiracy to commit larceny if his ignorance or belief was reasonable. To be reasonable, the ignorance or belief must have been based upon information or lack of it which would indicate to a reasonable person that he was authorized to order contacts at government expense. Additionally, the ignorance or mistake cannot be based on a negligent failure to discover the true facts.
In resolving this issue, you should consider the accused’s age, education, and rank along with the testimony of Senior Master Sergeant Kremer, Staff Sergeant Smith, Captain Bleuel, Staff Sergeant Allen, Staff Sergeant Smith, Airman Basic Maurice, Special Agent Collazo, Staff Sergeant McGee, Senior Airman Antoine, Airman First Class Marsh, Senior Airman Yallee, and Mr. Yenger.
The burden is on the prosecution to establish the accused’s guilt. If you are convinced beyond a reasonable doubt that at the time of the charged offenses the accused was not ignorant of a fact or under the mistaken belief that he was authorized to order contact lenses at government expense with a purchase order for military members who provided him only a prescription from the Optometry Clinic, the defense of ignorance or mistake of fact does not exist. Even if you conclude that the accused was ignorant of the fact or the mistaken belief that he was authorized to order contact lenses at government expense or the purchase order for military members who provided him only a prescription from the Optometry Clinic, if you are convinced beyond a reasonable doubt that at the time of the charged offenses the accused’s ignorance or mistake was unreasonable, the defense of ignorance or mistake of fact does not exist.

(R. 381-82) (emphasis added).

Appellant contends that the military judge at his court-martial erred in refusing to give an “honest” mistake-of-fact instruction as requested by the defense. We note that the military judge made this ruling because he concluded appellant’s purported mistake of fact went to a matter unrelated to the specific intent required for conviction of larceny under Article 121, UCMJ, 10 USC § 921. Accordingly, citing Paragraph 5-11 of the Military Judges’ Benchbook (Dept. of the Army Pamphlet 27-9 (Sept. 30, 1996)), he concluded that an honest and reasonable mistake as to such a fact was required and so instructed the members. The Court of Criminal Appeals, 1999 WL 1085836, likewise concluded that this was a correct instruction because the purported mistake “concerned the wrongfulness of the appellant’s taking, whether he had permission to order the contacts [, and] this is a general intent element and appellant’s belief must have been reasonable under all the circumstances.” Unpub. op. at 5. We disagree. United States v. *5 Turner, 27 MJ at 220 (holding that honest mistake of fact as to a superior’s permission to dispose of government property is a defense to larceny).

The military judge’s instructional decision on the defense of mistake-of-fact was generally based on his reading of Paragraph 5-11 of the Military Judges’ Benchbook. It states in pertinent part:

5-11 IGNORANCE OR MISTAKE OF FACT OR LAW-GENERAL DISCUSSION
The standard for ignorance or mistake of fact varies with the nature of the elements of the offense involved. If the ignorance or mistake concerns an element of an offense involving specific intent (e.g., desertion, larceny), willfulness {e.g., willful disobedience of an order), knowledge {e.g., assault upon commissioned officer, failure to obey lawful order), or premeditation, the ignorance or mistake need only exist in the mind of the accused. Generally, for crimes not involving specific intent, willfulness, knowledge, or premeditation, {e.g., AWOL) ignorance or mistake must be both honest (actual) and reasonable. Extreme care must be exercised in using this test, however, as ignorance or mistake in some “general intent” crimes need only be honest to be a defense. {See e.g., Instruction 5-11-4, Ignorance or Mistake in Drug Offenses.) Moreover, in some “specific intent” crimes, the alleged ignorance or mistake may not go to the element requiring specific intent or knowledge, and thus may have to be both reasonable and honest. Consequently, the military judge must carefully examine the elements of the offense, affirmative defenses, and relevant case law, in order to determine what standard applies.

This Court has suggested a similar rule in United States v. Peterson, 47 MJ 231, 234-35 (1997), and United States v. Garcia, 44 MJ 496, 498 (1996); cf. United States v. Tucker, 14 USCMA 376, 380, 34 CMR 156, 160 (1964) (honest mistake of fact applies where mistake made is to facts essential to the existence of the necessary mens rea, i.e., specific intent).

Article 121, UCMJ, states:

(a) Any person subject to this chapter who wrongfully takes, obtains, or withholds, by any means, from the possession of the owner or of any other person any money, personal property, or article of value of any kind—
(1) with intent permanently to deprive or defraud another person of the use and benefit of property or to appropriate it to his own use or the use of any person other than the owner, steals that property and is guilty of larceny;

(Emphasis added.) We have long recognized that this codal article requires the Government to prove beyond a reasonable doubt that an accused had a specific intent to steal. Moreover, it also has long been recognized that an honest mistake of fact as to a soldier’s entitlement or authorization to take property is a defense to a charge of larceny under this codal provision. See United States v. Sicley, 6 USCMA 402, 410-13, 20 CMR 118, 126-29 (1955); United States v. Rowan, 4 USCMA 430, 16 CMR 4 (1954). We have recently reiterated this holding. United States v. Gillenwater, 43 MJ 10 (1995); United States v. Turner, supra; United States v. Ward, 16 MJ 341, 346 (CMA 1983).

The military judge in appellant’s case did not acknowledge this case law in applying the Benchbook rule. See generally United States v. Tucker, supra (recognizing similar rule to Benchbook that mistake must be about essential facts related to specific intent in larceny case). Moreover, he did not explain why appellant’s averred mistake as to his authority to issue contact lenses did not undermine a specific intent on his part to permanently deprive the Government of this property. Contra United States v. Turner, supra. His exclusive reliance on the relevance of the mistaken fact to a so-called general “intent” element was misplaced. The pertinent inquiry is whether the purported mistake concerns a fact which would preclude the existence of the required specific intent. See generally 1 Wayne R. LaFave *6& Austin W. Scott, Substantive Criminal Law § 5.1(b) at 577 (1986).

This Court’s opinions in Peterson (mistake as to consent in housebreaking case) and Garcia (mistake as to sexual consent in indecent assault case) also do not support the trial judge’s ruling. They addressed mistakes as to facts which were completely unrelated to the specific intents at issue in those non-larceny cases. Moreover, those decisions do not suggest in any way that a military superior’s permission to his subordinate to dispose of government property to other servicemembers is that type of fact, ie., one unrelated to a specific intent of that subordinate to steal from the Government. See LaFave & Scott, supra; United States v. Sicley, supra at 413. Accordingly, we must find legal error.

Turning to the question of harmless error, we conclude that appellant was materially prejudiced by the “honest and reasonable” mistake-of-fact instruction actually given in this case. Article 59(a), UCMJ, 10 USC § 859(a). First of all, the prosecution-requested instruction placed a lesser burden on the Government to prove appellant’s guilt because it could now secure his conviction by disproving either the honesty or the reasonableness of appellant’s mistake. Moreover, trial counsel exploited the erroneous instruction by calling this government option to the attention of the members during deliberations. He said:

And that leads us to the defense of mistake of fact for all of these contacts. As the judge instructed you, there must be two different things here: One is, was this mistake honest; namely, did Airman Bine-gar really have this mistake? And, number two is: Was this mistake reasonable? Because regardless of whether Airman Binegar thought this, if it was not reasonable, the defense still does not exist. The government has disproven this mistake beyond a reasonable doubt.

(R. 391) (emphasis added). Finally, the Government presented a substantial case on the unreasonableness of appellant’s conduct in this case (R. 390-91), creating a reasonable possibility that the members resolved this case against appellant on this basis. See United States v. Ward, supra at 347.

The decision of the United States Air Force Court of Criminal Appeals is reversed.2 The findings of guilty and the sentence are set aside. The record of trial is returned to the Judge Advocate General of the Air Force. A rehearing may be ordered.

GIERKE, Judge (concurring in the result):

It has been said that “[n]o area of the substantive criminal law has traditionally been surrounded by more confusion than that of ignorance or mistake of fact or law.” 1 Wayne R. LaFave & Austin W. Scott, Substantive Criminal Law § 5.1(b) at 575 (1986). No doubt, this confusion arises in larceny cases because larceny has a specific intent element. See, e.g., United States v. Gillenwater, 43 MJ 10, 12 (1995) (judge erroneously failed to give mistake of fact instruction, believing that appellant’s mistake related only to a “ ‘lack of the element’ of intent”). Even the Supreme Court has recognized that the “venerable distinction” between general intent and specific intent “has been the source of a good deal of confusion.” United States v. Bailey, 444 U.S. 394, 403, 100 S.Ct. 624, 62 L.Ed.2d 575 (1980).

For the reasons set out below, I agree with the majority’s conclusion that the military judge erred by instructing the members that appellant’s asserted mistake of fact was required to be both honest and reasonable. However, I reach that conclusion by a different route than the majority.

The statutory elements of larceny are (1) a wrongful taking, obtaining, or withholding; and (2) an intent permanently to do one of the following: (a) “deprive ... another person of the use and benefit of property”; (b) “defraud another person of the use and bene*7fit of property”; (c) “appropriate it [the property] to his own use”; or (d) “appropriate it to ... the use of any person other than the owner[.]” Art. 121, UCMJ, 10 USC § 921. Although the term does not appear in the statute, “[t]hese intents are collectively called an intent to steal.” Para. 46c(l)(f)(l)(i), Part IV, Manual for Courts-Martial, United States (2000 ed.).1 The first element of this offense requires only general intent. See Simmons v. United States, 554 A.2d 1167, 1170 (D.C.App.1989). The second element requires specific intent. See United States v. Turner, 27 MJ 217, 220 (CMA 1988).

This Court previously has recognized the distinction between general intent and specific intent elements in other offenses. See United States v. Peterson, 47 MJ 231, 234-35 (1997) (indecent assault includes both a general intent assault element and a specific intent element to satisfy the lust or sexual desires of the accused). RCM 916(j), Manual, supra, also recognizes this distinction. It provides:

If the ignorance or mistake goes to an element requiring premeditation, specific intent, willfulness, or knowledge of a particular fact, the ignorance or mistake need only have existed in the mind of the accused. If the ignorance or mistake goes to any other element requiring only general intent or knowledge, the ignorance or mistake must have existed in the mind of the accused and must have been reasonable under all the circumstances.

In order to avoid further confusion in this complex area of law, I believe that this Court must focus on the precise significance of an appellant’s claimed mistake of fact and ask two questions: (1) What is the specific fact about which the appellant claims to have been mistaken? and (2) To what element or elements does that specific fact relate? The majority resolves this case by focusing on the second element of larceny. Chief Judge Crawford’s dissent focuses on the first element.2 In my view, appellant’s asserted mistake of fact in this case relates to both elements: (1) the wrongfulness of his act of obtaining the contact lenses; and (2) his specific intent to defraud the United States by issuing the contact lenses to persons who were not entitled to them.

Appellant asserted a mistake as to his authority to order the contact lenses for applicants who had prescriptions from the Optometry Clinic, but who had no documentation that the contact lenses were required for performance of duty. Appellant denied having both the general intent to wrongfully order the contact lenses and the specific intent to defraud the United States by ordering them for persons who were not entitled to them.

The majority opinion correctly states the rule: If the mistake goes to an element requiring only general intent, the mistake must be both honest and reasonable. However, if the mistake goes to an element requiring specific intent, the mistake need only be honest, i.e., exist in the mind of the accused.

Applying this rule in the present case, I believe that the military judge should have instructed the panel members that appellant’s mistake need only have been honest. If this case involved only a question of appellant’s general authority to order contact lenses, then the military judge’s instruction would have been correct. However, appel*8lant’s asserted mistake of fact also raised the question of appellant’s specific intent to defraud the United States by issuing contact lenses to persons who were not entitled to them. If appellant honestly believed he was authorized to order the lenses for persons who had no documentation showing that the contact lenses were required for duty, then he had no specific intent to defraud the United States.

Of course, appellant’s mistake-of-fact defense did not require two separate instructions, with one pertaining to the general intent element and one pertaining to the specific intent element. Although the asserted mistake of fact went to both elements, an instruction pertaining to the specific intent element would have subsumed an instruction pertaining to the general intent element.

Because the members were not correctly instructed regarding the legal effect of appellant’s asserted honest mistake of fact on his specific intent to defraud the United States, I join the majority in reversing the decision below.

CRAWFORD, Chief Judge (dissenting):

I would hold that the military judge did not abuse his discretion by instructing the members that appellant’s mistake related to the general intent element of “wrongful taking” and that the mistake had to be both “honest and reasonable.” Giving tens of free contact lenses to individuals not entitled to government contact lenses was neither honest nor reasonable. If the judge did err, any error was harmless because the record does not support the conclusion that appellant ever made an honest mistake of fact.

Facts

In September 1995, appellant began working in the Medical Logistics Office at Hans-com AFB clinic. Through the Medical Logistics Office, entitled personnel could obtain free contact lenses. Air Force personnel are entitled to free lenses if the lenses are required to perform their duties or if needed for a medical reason. All other personnel must order their contact lenses off base and pay for them out of their own pocket.

Appellant ordered lenses for his friends, irrespective of their eligibility, if they had a valid prescription. He did not make them hand over the prescription. They merely had to tell him what it was. Conversations occurred in the open about this practice. Appellant alleged that nobody thought it was unusual to order contact lenses. However, appellant reversed the spelling of the names of certain individuals on the purchase orders that he prepared. Additionally, evidence shows that the clinic would generate a local purchase request letter in all cases in which the military member was authorized to obtain contact lenses at government expense. Yet, there were no local purchase request letters generated for the contact lenses ordered by appellant for his friends.

Analysis

While perhaps appearing simple on its face, this issue requires a bit more complex analysis. That analysis involves recognition that there may be both general and specific intent elements. Well-established case law supports the conclusion that appellant’s mistake had to be both “honest and reasonable” in order to constitute a valid defense, because it related only to the general intent “taking” element within the crime of larceny.1

This Court’s standard of review with respect to member instructions is abuse of discretion. See United States v. Damatta-Olivera, 37 MJ 474, 478 (CMA 1993)(citing United States v. Smith, 34 MJ 200 (CMA 1992)). The test to determine if denial of a requested instruction constitutes error is whether (1) the charge is correct; (2) “it is *9not substantially covered in the main charge”; and (3) “it is on such a vital point in the case that the failure to give it deprived defendant of a defense or seriously impaired its effective presentation.” Id. (quoting United States v. Winborn, 14 USCMA 277, 282, 34 CMR 57, 62 (1963)).

In order to arrive at the proper end in this case, a series of analytical steps must be taken. The starting point is the larceny statute itself. A textual analysis of the statute should be performed to break out every actus reus element and then assign it a particular mens rea — either general or specific intent. The next step is to determine what element of the crime the alleged mistake pertains to and appoint the appropriate mistake of fact test accordingly. For example, this case deals with larceny, which Article 121, Uniform Code of Military Justice, 10 USC § 921, defines as follows:

Any person subject to this chapter who wrongfully takes, obtains, or withholds, by any means, from the possession of the owner or of any other person any money, personal property, or article of value of any kind with intent permanently to deprive or defraud another person of the use and benefit of property or to appropriate it to his own use or the use of any person other than the owner, steals that property and is guilty of larceny.

Larceny of military property can then be broken into these separate elements under a “textual” approach:

(a) That the accused wrongfully took, obtained, or withheld certain property from the possession of the owner or of any other person;
(b) That the property belonged to a certain person;
(c) That the property was of a certain value, or of some value;
(d) That the taking, obtaining, or withholding by the accused was with the intent permanently to deprive or defraud another person of the use and benefit of the property or permanently to appropriate the property for the use of the accused or for any person other than the owner; and
(e) That the property was military property.

Para. 46b(l), Part IV, Manual for Courts-Martial, United States (2000 ed.) (emphasis added).2

Appellant was mistaken as to the first element, i.e., the lawfulness of his taking the contacts in general. This actus reus element requires only a general intent. Because there is no specific intent requirement in this element, the military judge was correct in charging the members that appellant’s mistake had to be both “honest and reasonable” in order to constitute a mistake of fact defense. Therefore, the lower court should be affirmed.

1. Mistake of Fact

Appellant asserts that the military judge incorrectly gave instructions regarding his mistake of fact defense. Mistake of fact is a defense that does not deny the accused committed the objective acts constituting the offense charged, but- denies, wholly or partially, criminal responsibility for those acts. See RCM 916(a), Manual, supra. RCM 916(j)(l) provides that ignorance or mistake of fact may be a defense as follows:3

Except as otherwise provided in this subsection, it is a defense to an offense that the accused held, as a result of ignorance or mistake, an incorrect belief of the true circumstances such that, if the circumstances were as the accused believed them, the accused would not be guilty of the offense. If the ignorance or mistake goes to an element requiring ... specific intent, ... the ignorance or mistake need only have existed in the mind of the accused. If the ignorance or mistake goes to any other element requiring only general intent ... the ignorance or mistake must have existed in the mind of the accused *10 and must have been reasonable under all the circumstances.

(Emphasis added.)

This case involves the interpretation of ROM 916(j)(l). If, as in this case, appellant contends there was a lawful taking, the actus reus element, he does not get the benefit under the Manual rule of only requiring an honest rather than honest and reasonable mistake as to the taking. Under para. 46b(l)(a) & (d), supra, the mens rea term does not modify the actus reus term, the taking in this case. Thus, the taking element is a general intent element, and RCM 916(j)(l) requires a mistake as to that element to be both honest and reasonable.

In interpreting the Manual, I will not look at the consequences of the actus reus and tie that into the intent element, that is, if there is a lawful taking, then there cannot be an intent to defraud or to permanently deprive the owner of the property. This overlooks the division between the actus reus and the mens rea. The issue in this case centers on the actus reus. Thus, the taking must be both honest and reasonable.

The key to understanding this case is to recognize that one can make a mistake as to a general intent actus reus element within a crime requiring specific intent as to another element. When applying the mistake of fact defense in such an instance, a two-step analysis should be performed:

(1) Does the mistake show that the specific intent was not in fact entertained by the defendant? If it does, then the normal specific intent rule applies, and an honest mistake is a defense.
(2) If the mistake does not show that the specific intent is lacking, then the normal general intent rule applies, and only an honest and reasonable mistake is a defense.

Peter W. Law, Criminal Law 125-26 (Rev. 1st ed.1990). The following example from Law, id. at 126, nicely illustrates this hybrid category within the mistake of fact defense: Assume you have a crime defined as “receiving criminal law books known to have been stolen.” The defendant knows he received stolen books, but believes the books to cover English literature. Does his mistake negate specific intent? It depends on the interpretation of “known to have been stolen.” If it means that the defendant must know both that the books were stolen and that they were criminal law books, then the mistake is a defense under the normal approach for specific intent. However, if the specific intent is interpreted to mean only that the defendant must know that the books were stolen, the normal rule for general intent offenses will apply and mistake will be a defense only if it is both honest and reasonable. The defendant’s mistake in this instance will apply to the general intent portion of the offense — to the mental state required to commit the actus reus of receiving criminal law books.

The Military Judges’ Benchbook also acknowledges this hybrid category within mistake of fact. It states: “Moreover, in some ‘specific intent’ crimes, the alleged ignorance or mistake may not go to the element requiring specific intent or knowledge, and thus may have to be both reasonable and honest.” Para. 5-11 at 745, Dept. of the Army Pamphlet 27-9 (Sept. 30, 1996)(Ignorance or Mistake of fact or Law-General Discussion).

2. Assigning Mens Rea

The Benchbook indicates that a textual analysis should be used to determine the appropriate mens rea when the mistake of fact defense is raised. It says: “[T]he military judge must carefully examine the elements of the offense, affirmative defenses, and relevant case law, in order to determine what standard applies.” Id. (emphasis added). For example, indecent assault is a specific intent offense only with regard to the element of the accused’s intent to gratify his sexual desires, not to the offense in general. See United States v. Garcia, 44 MJ 496 (1996).

Most crimes today require a particular mental state. Unfortunately, ascertaining which terms or elements of the offense the mens rea modifies can be a complicated task. Some endeavor to resolve this problem by applying a “grammatical interpretation” after the crime has been subdivided into its elements. Joshua Dressier, Understanding *11 Criminal Law § 10.05 at 107 (1987). The grammatical interpretation says that the “placement of a mens rea term at the beginning of the definition of a crime may be interpreted to imply that the word modifies every actus reus element that follows it____ If the mental element is placed between some of the actus reus terms, however, this may mean that the mens rea does not apply to the actus reus terms that precede it.” Id. (citing United States v. Yermian, 468 U.S. 68, 104 S.Ct. 2936, 82 L.Ed.2d 53 (1984)).

For example, Dressier defines rape as “intentional sexual intercourse by a male with a female not his wife without her consent.” Id. at 106 (emphasis added). The critical issue is whether the word “intentional” modifies the attendant circumstance of the victim’s lack of consent because the word is placed at the beginning of the statute. Walking through the textual and grammatical approaches, this statute breaks down into the following:

(a) intentional sexual intercourse with a female other than your wife; and
(b) intentional intercourse without consent.

Because the mens rea word “intentional” is at the front of the statute, it can apply to the remaining elements of the crime. Therefore, any honest mistake will constitute a valid defense in this instance because both elements of the crime require specific intent.

On the other hand, Article 120, UCMJ, 10 USC § 920, states: “Any person subject to this chapter who commits an act of sexual intercourse by force and without consent, is guilty of rape.” Under a textual analysis, rape is broken down into the following elements:

(a) The accused committed an act of sexual intercourse; and
(b) The act of sexual intercourse was done by force and without consent.

Here, the statutory language of the crime does not assign a specific intent mens rea to any of the elements. Therefore, only an honest and reasonable mistake will suffice because the entire crime is one of general intent. This is markedly different from the Dressier example given above. These fine distinctions make it imperative for a judge to walk carefully through all of the necessary steps before deciding which mistake of fact test to instruct on or apply.

A second rule of thumb is that “a mens rea term ordinarily modifies the ‘result’ and ‘conduct’ elements in the actus reus — e.g., the ‘killing’ in murder, the ‘sexual intercourse’ in rape,” and the “taking” in larceny — but not the attendant circumstances. Dressier, supra at 107.

3. Relevant Case Law

Within the armed forces, there exists a line of indecent assault cases that are on point in this instance. In United States v. Garcia, 43 MJ 686, 687 (A.F.Ct.Crim.App. 1995), the appellant, on several occasions, made unwelcome and uninvited comments to and physical contact with a subordinate. The appellant argued that at the time of the incidents, he felt that the victim had given consent by coming over to his room and drinking beer with him. Id. at 688. The court held that mistake of fact as to the consent of the victim was a defense to indecent assault. Id. at 689. However, the court went on to say: “Indecent assault is a specific intent offense only with regard to the element of the accused’s intent to gratify his sexual desires, not to the offense in general.” Id. (emphasis added). Thus, to be a defense, the appellant’s mistake as to his victim’s consent must have been both “honest and reasonable.” In this instance, the military judge concluded that the appellant’s belief that the woman was consenting was not reasonable. Id.

In United States v. McFarlin, 19 MJ 790, 792 (ACMR), pet. denied, 20 MJ 314 (CMA 1985), the appellant was also charged with indecent assault. The court reasoned that although indecent assault is a specific intent crime, the applicable standard in this instance for the mistake of fact defense was “honest and reasonable.” Id. at 793. Here, the appellant inferred that his victim consented due to a lack of verbal or physical response in any way, but this particular’ mistake did not relate to the appellant’s intent. Rather it related to another element of the *12crime, namely, the presence or absence of the victim’s consent. Id.

United States v. Wooldridge, 49 MJ 513 (C.G.Ct.Crim.App.1998), again involves an indecent assault charge. In this case, the appellant entered the sleeping victim’s bedroom and sat on the floor staring at her. Id. at 514. After she awoke and was startled to find someone in her bedroom, she told the appellant to leave. He then asked if he could use her bathroom. She said yes, and he used the bathroom several times. In between each trip, he continued to ask the victim if he could sleep in her room. She said no every time. After his fifth trip to the bathroom, he removed his clothes, got in bed, and began to kiss and fondle the victim. Id. at 514. Mistake of fact was raised as an affirmative defense. The appéllant asserted that because the victim could have called for help each time he went to the bathroom, she was consenting to his acts. Id. at 515. The court held that although indecent assault entails one element requiring specific intent, the lack-of-consent element of the offense is a general intent element. Therefore, the mistake had to be both “honest and reasonable,” and the court held it was not. Id. at 514.

United States v. Peterson, 47 MJ 231 (1997), was decided by this Court. The Court held that where a person mistakes whether or not he has consent to enter another’s room and slips into bed and fondles the victim while she is sleeping, only an “honest and reasonable” mistake will constitute a valid defense. Id. at 234-35. This was so even though indecent assault entails one element requiring specific intent, because while the offensive touching was committed with the intent to satisfy the lust or sexual desires of the appellant, the consent element required only a general intent. Therefore, a mistake of fact defense on this element required both a subjective belief of consent and a belief that was reasonable under all of the circumstances. Id.

In Peterson, the Court noted that the first element of the offense of housebreaking, that the accused “unlawfully' entered a certain building or structure of a certain other person,” is a general intent element within the crime. Id. at 235. The Court also noted that the second element of housebreaking requires a specific intent to “enter with the intent to commit an offense.” Therefore, “[a]ny mistake-of-fact defense based upon [the] appellant’s belief of consent raised in respect to this element must have been both subjectively held and reasonable in light of all the circumstances.” Id.

4. Application to Appellant

Appellant argues that he honestly believed it was proper for any Air Force personnel with a valid prescription to receive contact lenses for free from the Air Force. He claims that nobody told him ordering contacts was allowed only in certain situations. He says he ordered the lenses openly and notoriously because he really did not think it was wrong. Appellant concedes, however, that his “mistake of fact defense did go directly to his knowledge — knowledge about whether or not Air Force personnel were entitled to free contact lenses with a valid prescription.” Final Brief at 13-14.

Similar to the Peterson and Garcia cases, the mistake in this case pertains to a general intent element within a specific intent crime — it relates to the wrongfulness of the taking. Thus, the military judge did not abuse his discretion by stating:

Okay. My analysis of the evidence and the offenses leads me to believe that a mistake-of-fact instruction that would be given would be general, rather than the one related to a specific intent.
I note that paragraph 5-11 of the Bench-book does give some guidance in that regard, and I note that it states as follows “... moreover, in some ‘specific intent’ crimes, the alleged ignorance or mistake may not go to the element requiring specific intent or knowledge, and thus may have to be both reasonable and hon-est____”
... I believe that the mistake concerning whether or not he was authorized to order contact lenses at government expense relates generally to the offense and is not related to that element which requires a specific intent. Therefore, I’m not going to give the instruction as propounded by *13defense counsel____ What I am going to give is some modified version of the general intent version of mistake-of-fact.

The relevant case law supports the conclusion that the military judge correctly interpreted appellant’s mistake as one relating to “wrongful taking” in general. “Wrongful taking” is a general intent element within the crime of larceny. Therefore, only an “honest and reasonable” mistake of fact will suffice as an affirmative defense.

A mistake as to the fourth element, “the intent to permanently deprive,” seems unlikely in this situation. Contact lenses are not an item that an individual uses for a time and then returns. Defense counsel cites cases dealing with property that could feasibly have been borrowed for a time and then returned.4 In the case at bar, however, such an analogy is rather far-fetched. Contact lenses are highly unique and individual; they are exposed to bodily fluids and generally are not returnable for health and hygiene reasons. Due to the sensitive nature of this product, it is highly unlikely that appellant did not intend to permanently deprive the Government of the lenses. Therefore, the only remaining element he could potentially have been mistaken about is the general wrongfulness of ordering the contacts in the first place.

Although defense counsel relies upon relevant case law, it is not on point. The underlying issue in some of the cases does involve a mistake of fact defense, but those decisions deal directly with an accused’s mistake relating to the specific intent “to permanently deprive.” The cases do not involve a “hybrid” case similar to the facts of Binegar. In other cases, the issue is a sua sponte concern, not an analysis of the appropriate mistake of fact test. For example, in United States v. Gillenwater, 43 MJ 10 (1995), the Court reversed the appellant’s conviction for wrongful appropriation of military property, holding that the lower court judge erred in refusing to give the court members any instruction on mistake of fact. Gillenwater, therefore, does not directly involve whether or not the mistake of fact test was correctly administered.

However, in Gillenwater, the Court examined the appropriate mistake of fact test, noting that the mistake applied to whether the appellant unlawfidly took or withheld the property “with the intent temporarily to deprive” the Government of the use of such property. Id. (emphasis added). We then determined that an “honest” mistake could negate the intent to steal. Id. There, the appellant’s supervisor allowed individuals to take tools home for personal use. The appellant thought that meant they could take them for as long as they wanted, provided they eventually returned them. Id. at 12. Therefore, the permission that the appellant thought he had did not pertain to the general intent to commit a wrongful act. Rather, it dealt with his specific intent to temporarily deprive. Id. at 13. This is noticeably a different factual situation than in the present case. Based on the record, it does not appear that appellant was ever mistaken as to the permanent deprivation. Contact lenses are generally not returnable or reusable. Additionally, in his brief, appellant does not rely on any part of the record to indicate that he intended to return the lenses in the future.

In United States v. Rowan, 4 USCMA 430, 16 CMR 4 (1954), the appellant was charged with larceny by check. The appellant was under the mistaken belief that he had money in the Kanawha Valley Bank to cover any checks he wrote. The Court concentrated on whether mistake of fact would be a defense at all. The main focus of the holding establishes that the test for mistake of fact with respect to larceny and larceny by false pretenses is the same, even though the language of the Manual for Courts-Martial appears to set up two different standards. Id. at 432-34, 16 CMR at 6-8. The Court held;

The requirement of a specific intent is found in military law as well as in the civilian sphere. A court-martial must find ... that the accused intended, at or after the time of the taking, permanently to *14deprive the owner of the property in question---- Because that intent is required to make out the offense of larceny, it is commonly held that there is no such thing as a negligent larceny. An honest ignorance or mistake of fact may be a defense even though either was due to carelessness.

Id. at 434, 16 CMR at 8. This holding merely indicates that the specific intent to “permanently deprive” can be negated by an honest mistake. It does not address the mistake that pertains to a general intent element within a specific intent crime.

Defense counsel also cites United States v. Sicley, 6 USCMA 402, 20 CMR 118 (1955), claiming that the appellant’s mistake did not go to the specific intent to permanently deprive, but rather, it went to his belief that he was authorized to receive reimbursement for his wife’s travel, even though she did not use her ticket. The Court determined that “it is thus reasonable to assume ... that the findings of guilty of larceny were based not on an unlawful taking effected on February 2, but rather on the view that the accused acquired the necessary intent subsequently, and thereafter wrongfully withheld funds which he had acquired lawfully.” Id. at 407, 20 CMR at 123. The appellant received the money honestly and in good faith but formed the intent to steal the money at a later date. The Court then held, “Thus, we address ourselves to the question of whether an honest mistake of law may — in this setting — negate the inference of an intent permanently to deprive the Government of property.” Id. at 411, 20 CMR at 127. Again, this is a situation where the appellant’s mistake pertained to the specific intent to permanently deprive. Therefore, an honest mistake should be the standard applied.

Finally, in United States v. Turner, 27 MJ 217 (CMA 1988), the appellant was charged with larceny of two automobile engines. The central issue of the case was whether the appellant was prejudiced by the trial judge’s failure to give the defense-requested instruction on mistake of fact. Therefore, this ease is not on point, and reliance on it is misplaced. The appellant in Turner contended that he honestly believed the engines were not government property, and that he could therefore lawfully receive them. Id. at 218-19. The Court held that the appellant’s honest belief that he was entitled to the engines negated any specific intent to steal. Id. at 220. In the present case, however, appellant’s mistake was determined by the judge to apply to the element of wrongful taking, a general intent element, within the crime of larceny. Therefore, an honest and reasonable mistake was the correct standard.

Assuming arguendo that the judge did err, any error was harmless. The record indicates that appellant did not hold an “honest” mistake regarding whether or not he could order contact lenses for all Air Force personnel. His claim of an honest mistake is refuted by his conduct in ordering contact lenses for his friends. Appellant wrote out his friends’ names in code on the order forms. He spelled the names backwards and left out a letter or added a letter in some instances. If he honestly believed his actions to be lawful, there would be no need to alter his friends’ names. Additionally, when a valid order for lenses was generated, a special form came back with the lenses. Not once did this form accompany appellant’s friends’ lenses. He worked there long enough to notice this difference. Therefore, even if the military judge did instruct the members incorrectly and appellant’s mistake need only have been “honest,” the guilty conviction would still stand because there is no evidence that appellant held an honest belief that his actions were lawful.

For all of the foregoing reasons, I respectfully dissent.

Notes on mistakes of fact Notes on mistakes of fact

Can you now clearly articulate the traditional rules regarding mistakes of fact for "general intent" and "specific intent" offenses? Can you describe precisely what element of each criminal offense--conduct, result, circumstances--that each defendant was mistaken about? Do the doctrines for factual mistakes do a good job of allowing criminal liability for blameworthy people and barring criminal liability for the nonblameworthy?

Notice the function of mistake-of-fact rules. These rules fill a gap (or resolve issues) left open by the fact that neither "general intent" nor "specific intent" doctrines generally say anything about mental state that, in order to be guilty, one must have with respect to the factual circumstances surrounding crimes. General intent relates mostly to conduct, specific intent to results.

In State v. Woods, 179 A. 1. (Vt. 1935) Ms. Woods was charged under a statute that prohibited “[a] man with another man’s wife, or a man with another woman’s husband, found in bed together, under circumstances affording presumption of an illicit intention.” Ms. Woods had traveled with Shuffelt, the man with whom she was eventually found in bed, to Reno, Nevada where he divorced his first wife and then married Ms. Woods. Unfortunately for Ms. Woods, the divorce was ineffective as neither Suffelt nor his wife was a Nevada resident. Thus, Ms. Woods believed, reasonably though wrongly, that the man she was lying with was her husband rather than “another woman’s husband.” The Court, finding that public policy against “attempted foreign divorces” would be eviscerated by claims of ignorance, disagreed: “When it is proved that the parties were found in bed together under circumstances affording a presumption of an intention to commit the act charged in the indictment or information, then the requirement of the statute is met and ignorance of the law cannot be urged as a defense.”

Did Ms. Woods make a mistake about a fact or about divorce law? What mens rea did the Woods court read into the statute with regard to whether one’s bed companion is married to another?

4.3 Mistakes of Law 4.3 Mistakes of Law

We have seen that a mistake is a defense to a crime if it negates the mens rea required by statute. As the Model Penal Code points out, whether we use the language of mistake makes no difference: A mistake provides a defense if and only if it negates the mens rea required by statute.

The challenge sometimes is determining what mens rea is required by statute, and then whether the defendant's mistake demonstrates that he did not have that mens rea.

In the next materials, we examine whether and how ignorance or mistakes about the law, rather than some fact, affects liability.

Rules on relevance of mistakes of law Rules on relevance of mistakes of law

The Model Penal Code's provision regarding the relevance of mistakes about (or ignorance of) the criminal law is fairly representative of the rule in most jurisdictions. Recall 18 Pa. C.S. § 302(h), noted earlier in the chapter, which codified MPC 2.02(9) verbatim: 

Culpability as to illegality of conduct.--Neither knowledge nor recklessness or negligence as to whether conduct constitutes an offense or as to the existence, meaning or application of the law determining the elements of an offense is an element of such offense, unless the definition of the offense or this title so provides.

People v. Marrero People v. Marrero

People v. Marrero

New York Court of Appeals

69 N.Y.2d 382 (1987)

 

BELLACOSA, J.

The defense of mistake of law (Penal Law § 15.20) is not available to a Federal corrections officer arrested in a Manhattan social club for possession of a loaded .38 caliber automatic pistol who claimed he mistakenly believed he was entitled, pursuant to the interplay of CPL 2.10, 1.20 and Penal Law § 265.20, to carry a handgun without a permit as a peace officer.

On the trial of the case, the court rejected the defendant's argument that his personal misunderstanding of the statutory definition of a peace officer is enough to excuse him from criminal liability under New York's mistake of law statute. The court refused to charge the jury on this issue and defendant was convicted of criminal possession of a weapon in the third degree. We affirm the Appellate Division order upholding the conviction.

[CPL 1.20 provided an exception to the certain firearm possession laws for peace officers and section 2.10 defined peace officer to include "correction officers of any state correction facility or of any penal correctional institution."]

Defendant was a Federal corrections officer in Danbury, Connecticut, and asserted that status at the time of his arrest in 1977. He claimed at trial that there were various interpretations of fellow officers and teachers, as well as the peace officer statute itself, upon which he relied for his mistaken belief that he could carry a weapon with legal impunity.

The starting point for our analysis is the New York mistake statute as an outgrowth of the dogmatic common-law maxim that ignorance of the law is no excuse. The central issue is whether defendant's personal misreading or misunderstanding of a statute may excuse criminal conduct in the circumstances of this case.

The common-law rule on mistake of law was clearly articulated in Gardner v People. In Gardner, the defendants misread a statute and mistakenly believed that their conduct was legal. The court insisted, however, that the "mistake of law" did not relieve the defendants of criminal liability.

The desirability of the Gardner-type outcome, which was to encourage the societal benefit of individuals' knowledge of and respect for the law, is underscored by Justice Holmes' statement: "It is no doubt true that there are many cases in which the criminal could not have known that he was breaking the law, but to admit the excuse at all would be to encourage ignorance where the law-maker has determined to make men know and obey, and justice to the individual is rightly outweighed by the larger interests on the other side of the scales" (Holmes, The Common Law, at 48 [1881]).

The revisors of New York's Penal Law intended no fundamental departure from this common-law rule in Penal Law § 15.20, which provides in pertinent part:

§ 15.20. Effect of ignorance or mistake upon liability.

(2) "A person is not relieved of criminal liability for conduct because he engages in such conduct under a mistaken belief that it does not, as a matter of law, constitute an offense, unless such mistaken belief is founded upon an official statement of the law contained in

(a) a statute or other enactment * * *

(d) an interpretation of the statute or law relating to the offense, officially made or issued by a public servant, agency, or body legally charged or empowered with the responsibility  or privilege of administering, enforcing or interpreting such statute or law."

This section was added to the Penal Law as part of the wholesale revision of the Penal Law in 1965. When this provision was first proposed, commentators viewed the new language as codifying "the established common law maxim on mistake of law, while at the same time recognizing a defense when the erroneous belief is founded upon an 'official statement of the law'"

The defendant claims as a first prong of his defense that he is entitled to raise the defense of mistake of law under section 15.20(2)(a) because his mistaken belief that his conduct was legal was founded upon an official statement of the law contained in the statute itself. Defendant argues that his mistaken interpretation of the statute was reasonable in view of the alleged ambiguous wording of the peace officer exemption statute, and that his "reasonable" interpretation of an "official statement" is enough to satisfy the requirements of subdivision (2)(a). However, the whole thrust of this exceptional exculpatory concept, in derogation of the traditional and common-law principle, was intended to be a very narrow escape valve. Application in this case would invert that thrust and make mistake of law a generally applied or available defense instead of an unusual exception which the very opening words of the mistake statute make so clear "[a] person is not relieved of criminal liability for conduct * * * unless…" The momentarily enticing argument by defendant that his view of the statute would only allow a defendant to get the issue generally before a jury further supports the contrary view because that consequence is precisely what would give the defense the unintended broad practical application.

The prosecution further counters defendant's argument by asserting that one cannot claim the protection of mistake of law under section 15.20(2)(a) simply by misconstruing the meaning of a statute but must instead establish that the statute relied on actually permitted the conduct in question and was only later found to be erroneous. To buttress that argument, the People analogize New York's official statement defense to the approach taken by the Model Penal Code (MPC). Section 2.04 of the MPC provides: 

Section 2.04. Ignorance or Mistake.

"(3) A belief that conduct does not legally constitute an offense is a defense to a prosecution for that offense based upon such conduct when * * *

(b) he acts in reasonable reliance upon an official statement of the law, afterward determined to be invalid or erroneous, contained in

(i) a statute or other enactment" (emphasis added).

Although the drafters of the New York statute did not adopt the precise language of the Model Penal Code provision with the emphasized clause, it is evident and has long been believed that the Legislature intended the New York statute to be similarly construed. In fact, the legislative history of section 15.20 is replete with references to the influence of the Model Penal Code provision.

In the case before us, the underlying statute never, in fact, authorized the defendant's conduct; the defendant only thought that the statutory exemptions permitted his conduct when, in fact, the primary statute clearly forbade his conduct. Moreover, by adjudication of the final court to speak on the subject in this very case, it turned out that even the exemption statute did not permit this defendant to possess the weapon. It would be ironic at best and an odd perversion at worst for this court now to declare that the same defendant is nevertheless free of criminal responsibility.

It must also be emphasized that, while our construction of Penal Law § 15.20 provides for narrow application of the mistake of law defense, it does not, as the dissenters contend, "rule out any defense based on mistake of law." To the contrary, mistake of law is a viable exemption in those instances where an individual demonstrates an effort to learn what the law is, relies on the validity of that law and, later, it is determined that there was a mistake in the law itself.

The modern availability of this defense is based on the theory that where the government has affirmatively, albeit unintentionally, misled an individual as to what may or may not be legally permissible conduct, the individual should not be punished as a result. This is salutary and enlightened and should be firmly supported in appropriate cases. However, it also follows that where, as here, the government is not responsible for the error (for there is none except in the defendant's own mind), mistake of law should not be available as an excuse. 

We conclude that the better and correctly construed view is that the defense should not be recognized, except where specific intent is an element of the offense or where the misrelied-upon law has later been properly adjudicated as wrong. Any broader view fosters lawlessness. It has been said in support of our preferred view in relation to other available procedural protections:

"A statute...which is so indefinite that it 'either forbids or requires the doing of an act in terms so vague that men of common intelligence must necessarily guess at its meaning and differ as to its application, violates the first essential of due process of law' and is unconstitutional. If the court feels that a statute is sufficiently definite to meet this test, it is hard to see why a defense of mistake of law is needed. Such a statute could hardly mislead the defendant into believing that his acts were not criminal, if they do in fact come under its ban ... [I]f the defense of mistake of law based on indefiniteness is raised, the court is...going to require proof...that the act was sufficiently definite to guide the conduct of reasonable men. Thus, the need for such a defense is largely supplied by the constitutional guarantee"

Strong public policy reasons underlie the legislative mandate and intent which we perceive in rejecting the defendant's construction of New York's mistake of law defense statute. If defendant's argument were accepted, the exception would swallow the rule. Mistakes about the law would be encouraged, rather than respect for and adherence to law. There would be an infinite number of mistake of law defenses which could be devised from a good-faith, perhaps reasonable but mistaken, interpretation of criminal statutes, many of which are concededly complex. Even more troublesome are the opportunities for wrongminded individuals to contrive in bad faith solely to get an exculpatory notion before the jury. These are not in terrorem arguments disrespectful of appropriate adjudicative procedures; rather, they are the realistic and practical consequences were the dissenters' views to prevail. Our holding comports with a statutory scheme which was not designed to allow false and diversionary stratagems to be provided for many more cases than the statutes contemplated. This would not serve the ends of justice but rather would serve game playing and evasion from properly imposed criminal responsibility.

Accordingly, the order of the Appellate Division should be affirmed.

 

HANCOCK, JR., J. (dissenting).

The basic difference which divides the court may be simply put. Suppose the case of a man who has committed an act which is criminal not because it is inherently wrong or immoral but solely because it violates a criminal statute. He has committed the act in complete good faith under the mistaken but entirely reasonable assumption that the act does not constitute an offense because it is permitted by the wording of the statute. Does the law require that this man be punished? The majority says that it does and holds that (1) Penal Law § 15.20(2)(a) must be construed so that the man is precluded from offering a defense based on his mistake of law and (2) such construction is compelled by prevailing considerations of public policy and criminal jurisprudence. We take issue with the majority on both propositions.

 It is difficult to imagine a case more squarely within the wording of Penal Law § 15.20(2)(a) or one more fitted to what appears clearly to be the intended purpose of the statute than the one before us. For this reason it is helpful to discuss the statute and its apparent intended effect in the light of what defendant contends was his mistaken belief founded on an official statement of the law contained in a statute.

The majority… has accepted the People's argument that to have a defense under Penal Law § 15.20(2)(a) "a defendant must show that the statute permitted his conduct, not merely that he believed it did." Here, of course, the defendant cannot show that the statute permitted his conduct. To the contrary, the question has now been decided by the Appellate Division and it is settled that defendant was not exempt under Penal Law § 265.20(a)(1)(a). Therefore, the argument goes, defendant can have no mistake of law defense. While conceding that reliance on a statutory provision which is later found to be invalid would constitute a mistake of law defense the People's flat position is that "one's mistaken reading of a statute, no matter how reasonable or well intentioned, is not a defense."

[T]he majority bases its decision on an analogous provision in the Model Penal Code and concludes that despite its totally different wording and meaning Penal Law § 15.20(2)(a) should be read as if it were Model Penal Code § 2.04(3)(b)(i). But New York, in revising the Penal Law, did not adopt the Model Penal Code. As in New Jersey, which generally adopted the Model Penal Code but added one section which is substantially more liberal, New York followed parts of the Model Penal Code provisions and rejected others. In People v. Goetz, we said that the Legislature's rejection of the verbatim provisions of the Model Penal Code was crucial in determining its intent in drafting the statute. The significance of the alterations here can be no different.

Any fair reading of the majority opinion, we submit, demonstrates that the decision to reject a mistake of law defense is based on considerations of public policy and on the conviction that such a defense would be bad, rather than on an analysis of CPL 15.20(2)(a) under the usual principles of statutory construction The majority warns, for example, that if the defense were permitted "the exception would swallow the rule"; that "[m]istakes about the law would be encouraged"; that an "infinite number of mistake of law defenses... could be devised"; and that "wrongminded individuals [could] contrive in bad faith solely to get an exculpatory notion before the jury."

These considerations, like the People's argument that the mistake of law defense "would encourage ignorance where knowledge is socially desired" (Respondent's Brief, at 28.), are the very considerations which have been consistently offered as justifications for the maxim "ignorantia legis". That these justifications are unabashedly utilitarian cannot be questioned. It could not be put more candidly than by Justice Holmes in defending the common-law maxim more than 100 years ago: "Public policy sacrifices the individual to the general good ... It is no doubt true that there are many cases in which the criminal could not have known that he was breaking the law, but to admit the excuse at all would be to encourage ignorance where the law-maker has determined to make men know and obey, and justice to the individual is rightly outweighed by the larger interests on the other side of the scales" (Holmes, The Common Law, at 48 [1881]; emphasis added). Regardless of one's attitude toward the acceptability of these views in the 1980s, the fact remains that the Legislature in abandoning the strict "ignorantia legis" maxim must be deemed to have rejected them.

We believe that the concerns expressed by the majority are matters which properly should be and have been addressed by the Legislature. We note only our conviction that a statute which recognizes a defense based on a man's good-faith mistaken belief founded on a well-grounded interpretation of an official statement of the law contained in a statute is a just law. The law embodies the ideal of contemporary criminal jurisprudence "that punishment should be conditioned on a showing of subjective moral blameworthiness.”

State v. Varszegi State v. Varszegi

State of Connecticut v. Paul Varszegi

(10818)

O’Connell, Foti and Cretella, Js.

Argued October 28

decision released December 3, 1993

*369 Paul Varszegi, pro se, the appellant (defendant).

David J. Sheldon, deputy assistant state’s attorney, with whom, on the brief, were Eugene J. Callahan, state’s attorney, and Nicholas Bove, assistant state’s attorney, for the appellee (state).

O’Connell, J.

The defendant was initially charged with larceny in the first degree in violation of General Statutes §§ 53a-122 (a) (2)1 and 53a-119,2 and criminal coercion in violation of General Statutes § 53a-49 (a) (2). Following a jury trial, he was convicted of the lesser included offense of larceny in the third degree in violation of General Statutes § 53a-1243 and acquitted of the coercion charge. The defendant appeals from his larceny conviction. The defendant claims that (1) there was insufficient evidence to support the larceny conviction and (2) the trial court’s jury instruction violated his constitutional rights to due process of law and a fair trial. We reverse.

The jury could have reasonably found the following facts. The defendant was the landlord of commercial *370property at 1372 Summer Street in Stamford. One of the defendant’s tenants was Executive Decisions Support, Inc., a computer software company headed by Catherine Topp. Topp, in her capacity as president of the company, personally signed the lease with the defendant. The lease contained a default clause authorizing the defendant to enter the tenant’s premises, seize the tenant’s personal property and sell it as a way of recovering unpaid rent or other charges.4 The defendant claimed Topp had failed to pay her rent for March, April and May, 1990.

On Saturday May 5, 1990, the defendant entered Topp’s office by picking the lock. He proceeded to remove two of Topp’s computers and attached printers. On Monday May 7, 1990, Topp arrived at work and noticed that the lock on her office door had been tampered with. Upon entering and noticing her computers were missing, Topp called the Stamford police. After the completion of an initial investigation at the scene by Officer Frank Pica, Topp telephoned the defendant, who admitted that he had taken her computers as a consequence of her failure to pay three months rent. Pica then took the phone from Topp and identified himself. The defendant advised Pica that he was Topp’s landlord and expressed his belief that his actions were *371proper and legal. Upon hearing this, Pica expressed doubt as to the lawfulness of the defendant’s conduct, at which point the defendant reiterated his belief that his actions were in accordance with his lease.

At this point, Pica called his supervisor, Sergeant Ralph Geter. Geter arrived shortly thereafter and telephoned the defendant, who again identified himself as Topp’s landlord and again admitted having taken the computers in question. Geter informed the defendant that he had no right under the law to confiscate Topp’s computer equipment and that he should make arrangements with Topp to return the goods. No such arrangements were made and the defendant sold the computers on May 23, 1990.

On May 25,1990, a detective contacted the defendant and inquired whether they could meet to discuss the matter as part of his investigation. As he had from the start, the defendant admitted during this telephone conversation that he had taken and retained the computers pursuant to his lease. On June 6, 1990, the defendant met with the detective at the Stamford police station and was asked for a formal statement. The defendant requested an attorney and the conversation ended. The defendant was subsequently arrested on June 28, 1990.

The defendant first claims that the state did not prove beyond a reasonable doubt that he was guilty of larceny. It is fundamental jurisprudence that the state bears the burden of proof beyond a reasonable doubt of each essential element of the crime charged. In re Winship, 397 U.S. 358, 364, 90 S. Ct. 1068, 25 L. Ed. 2d 368 (1970); State v. Griffin, 175 Conn. 155, 162, 397 A.2d 89 (1978).

“When a claim on appeal challenges the sufficiency of the evidence, we undertake a two-part task. We first review the evidence presented at the trial, construing *372it in the light most favorable to sustaining the jury’s verdict. We then determine whether, upon the facts thus established and the inferences reasonably drawn therefrom, the jury could reasonably have concluded that the cumulative effect of the evidence established guilt beyond a reasonable doubt.” (Internal quotation marks omitted.) State v. King, 216 Conn. 585, 600, 583 A.2d 896 (1990); State v. Glenn, 30 Conn. App. 783, 791, 622 A.2d 1024 (1993).

Larceny is defined in General Statutes § 53a-119 as follows: “A person commits larceny when, with intent to deprive another of property or to appropriate the same to himself or a third person, he wrongfully takes, obtains or withholds such property from an owner. ...” This crime has three elements and all three must be established beyond a reasonable doubt. It must be shown that (1) there was an intent to do the act complained of, (2) the act was done wrongfully, and (3) the act was committed against an owner. This appeal focuses on the elements of “intent” and “wrongfully.” Because larceny is a specific intent crime, the state must show that the defendant acted with the subjective desire or knowledge that his actions constituted stealing. A specific “intent to deprive another of property or to appropriate the same to himself ... is an essential element of larceny . . . and as such must be proved beyond a reasonable doubt by the state.” (Internal quotation marks omitted.) State v. Fernandez, 198 Conn. 1, 20, 501 A.2d 356 (1985).

“The ‘animus furandi,’ or intent to steal, is an essential element of the crime of larceny at common law.” 50 Am. Jur. 2d, Larceny § 35 (1970). Hornbook law articulates the same premise. “Since the taking must be with felonious intent . . . taking under a bona fide claim of right, however unfounded, is not larceny. . . . [Although ignorance of the law is, as a rule, no excuse, it is an excuse if it negatives the existence of a specific *373intent. Therefore, even if the taker’s claim of right is based upon ignorance or mistake of law, it is sufficient to negative a felonious intent. A fortiori, a mistake of fact, if it is the basis of a bona fide claim of right, is sufficient.” J. Miller, Handbook of Criminal Law (1934) § 114 (a), p. 367; see also 2 W. LaFave & A. Scott, Substantive Criminal Law (1986) § 8.5 (a), p. 358.

“One who takes property in good faith, under fair color of claim or title, honestly believing that ... he has a right to take it, is not guilty of larceny even though he is mistaken in such belief, since in such case the felonious intent is lacking. . . . The general rule applies . . . to one who takes it with the honest belief that he has the right to do so under a contract . . . .” 50 Am. Jur. 2d 203, Larceny § 41 (1970).

Particularly apposite to the present case is the comment in American Jurisprudence that, “[i]t is generally held that because of lack of a felonious intent, one is not guilty of larceny who, in the honest belief that he has the right to do so, openly and avowedly takes the property of another without the latter’s consent, as security for a debt bona fide claimed to be due him by the owner, or even to apply or credit it to the payment thereof.” Id., 205-206.

“A defendant who acts under the subjective belief that he or she has a lawful claim on property lacks the required felonious intent to steal. Such a defendant need not show his mistaken claim of right was reasonable, since an unreasonable belief that he had a right to take another’s property will suffice so long as he can establish his claim was made in good faith.” 50 Am. Jur. 2d, Larceny § 41 (1993 Sup.), relying on People v. Romo, 220 Cal. App. 3d 514, 269 Cal. Rptr. 440 (1990).

Connecticut case law is consistent with other jurisdictions and the text writers quoted herein. As far back as 1902, our Supreme Court recognized that the mere *374taking of another’s property was not sufficient to prove larceny. The taking must be accompanied by a felonious intent to deprive the owner of its use. If a person takes property in the honest, though mistaken belief, that he has a right to do so, he has not committed larceny. State v. Main, 75 Conn. 55, 57, 52 A. 257 (1902).

Our Supreme Court has also considered the scenario of a landlord who, like the defendant in the present case, seizes his tenant’s property as security for a debt. State v. Sawyer, 95 Conn. 34, 35-36, 110 A. 461 (1920). In Sawyer, a landlady took possession of a tenant’s handbag on the ground that the tenant was liable to her for damage done to the demised premises. The court held that the defendant had not committed a larceny because she believed that her claim justified refusal to return the handbag unless the tenant paid for the damage. Id., 38. The court based its decision on the absence of felonious intent and the presence of a color of right. The court held that “[o]ne is not guilty of theft without consciousness of the wholly unlawful and inexcusable nature of his act. The grave character of the necessary intent makes that clear. His belief in his right to take the thing involved, even though a mistaken belief, or one entertained upon insufficient grounds, is essentially inconsistent with the presence of an intent to steal, and the terms ‘colour of right or excuse’ obviously demand a construction consonant with that point of view.” (Emphasis added.) Id. The Sawyer court supported its decision with a citation from 2 Bishop’s New Crim. Law (8th Ed.) § 849, p. 496. “ ‘[0]ne who takes another’s goods to compel him, though in an irregular way, to do what the law requires him to do with them — namely, pay his debt — is on no legal principle a felon, though doubtless he is a trespasser. ... In reason, one has no more privilege to steal the effects of his debtor than those of any other person. But a trespass is not theft, except when done *375with felonious intent. And he who carries away a thing openly, and not clandestinely, to enforce a just claim, not for fraud, not to injure the owner, but to compel him to do what the law requires, is not a thief, whatever the extent of the wrong viewed otherwise.’ ” State v. Sawyer, supra, 39.

In the present case, the defendant claims that he did not possess the specific felonious intent to commit larceny but rather acted in good faith pursuant to a lease that gave him, as the landlord, authority to enter the premises, impound the lessee’s property, sell it and apply the proceeds to the unpaid rent or other charges. The state claims that it introduced evidence from which the jury could reasonably have inferred that the defendant intended to steal the computer equipment.

We hold that there was insufficient evidence introduced at trial to prove that the defendant knew that he had no right to take the lessee’s computers. During the initial phone conversation with Topp, the defendant made no attempt to conceal either his identity or that he had in fact taken the computers. The defendant emphatically stated during this conversation that the default clause in the lease provided him with the authority to impound her property based on her failure to pay the three months rent due. Moreover, the officers testifying at trial each stated that the defendant never wavered from his contention that his actions were lawful, even when faced with the threat of criminal prosecution. The defendant’s unfaltering and consistent statements to all parties involved that he acted in good faith in seizing Topp’s computers were not contradicted by the testimony of any witness.

The state cites State v. Banet, 140 Conn. 118, 98 A.2d 530 (1953), in support of its claim that the defendant possessed the requisite criminal intent. In Banet, the defendant, a used car dealer, signed over to a finance *376company to whom he was indebted all of his interest in a number of retail sales installment contracts. Id., 120-21. After two purchasers defaulted, the defendant was found guilty of wrongfully repossessing their vehicles in violation of the assignment agreement. In rejecting the defendant’s claim that he believed in good faith that he had a right to retake the automobiles, the court held that the state had introduced evidence showing the defendant knew that his actions were unlawful. This knowledge was evidenced by the assignment agreement itself, which explicitly transferred title to the automobiles to the finance company. Furthermore, the court viewed the finance company’s refusal to sign a postassignment agreement whereby the defendant attempted to retain the right to repossess the vehicles as strong proof that the defendant knew that his actions were illegal. Id., 122-23.

Unlike Banet, the defendant’s claim that he acted in good faith was not countered by any evidence, direct or circumstantial, that he intended to steal the computers. The state contends, however, that the defendant’s weekend entry into Topp’s office by picking the lock and his subsequent impounding of the computers without leaving a note was enough evidence from which the jury could have reasonably found the requisite intent. Significantly, none of the foregoing evidence contradicted the defendant’s claim that he acted in good faith in taking the computers based on the default clause in the lease. In fact, the defendant testified that he entered the office on the weekend so as to avoid an awkward confrontation. When asked by Topp where the computers were on the following Monday, the defendant concealed nothing. Notwithstanding the state’s claim to the contrary, the record is devoid of any evidence that the jury could have reasonably relied upon in finding the defendant acted with the requisite culpability. The state's bald and conclusory assertion *377that the jury simply rejected the defendant’s consistent claim that the rental agreement served as justification for his action is not enough to prove beyond a reasonable doubt the specific intent requirement for larceny.

Moreover, the state contends that even if the defendant believed in good faith that he had the right to take his tenant’s personal property to secure overdue rent payments, once he was told by three members of the police force that he was acting illegally, he could no longer reasonably maintain an honest though mistaken belief that he was acting lawfully. We do not agree. Police officers are not imbued with the authority or prerogative to declare provisions of civil contracts void, thereby converting good faith to a felonious intent.

In its brief, the state argues that the passage of time since the Sawyer case has dimmed its effect and suggests that we disregard it as a precedent. Even if we agreed that in some instances time may diminish a precedent, the present case would not be an appropriate situation in which to apply such theory. The felonious taking requirement has not given way to more modern legal concepts. “Although the present statute has broadened the scope of larceny, the element of intent has been retained from the prior statute and case law. Larceny continues to require ‘the existence of a felonious intent in the taker . . . .’ ” State v. Marra, 174 Conn. 338, 342, 387 A.2d 550 (1978). Furthermore, in drafting General Statutes § 53a-119, the penal code commission expressly referred to Sawyer. “The question of wrongfulness is meant to incorporate and to be read in the light of the common law standards enunciated in such cases as . . . State v. Sawyer, [supra].” Commission to Revise the Criminal Statutes, Penal Code Comments, Connecticut General Statutes (1969), p. 39.

*378We agree with the defendant that there was insufficient evidence to support his conviction of larceny in the third degree. Because our decision on the defendant’s first claim is dispositive of the appeal, we do not reach the defendant’s second claim.

The judgment is reversed and the case is remanded with direction to render judgment of not guilty of larceny in the third degree.

In this opinion the other judges concurred.

Notes and Questions Notes and Questions

Given prosecutors' wide discretion in deciding whether to charge a suspect with a crime, was the prosecutor's decision to charge Marrero an appropriate one? What reasons make it so (or justify declining to charge)? What about the decision to charge Varszegi?

Marrero and Varszegi represent different kinds of mistakes of law that one might make. Marrero is in most Criminal Law casebooks as a classic example of a mistake of criminal law. Varszegi was mistaken about a different kind of law--civil property law (or perhaps contract law).

Do you agree that Marrero made a "mistake of criminal law"? Marrero He was charged under a statute that prohibited the possession of a concealed weapon. His mistake was about the meaning of peace officer as used in the New York code; put differently, he was confused about who was a peace officer. Was his mistake a reasonable one?

In concluding that Marrero's mistake did not afford him a defense, did the court implicitly conclude that the statute required no mens rea with regard to whether one has the legal status--i.e., being a peace officer--that exempts one from criminal liability?

Varszegi falls into a category of mistakes described as "color of right" claims. Unlike mistakes of criminal law, claims that one has a right to some property based on a mistaken about belief about rights of ownership or possession are often successful, at least if the court is sufficiently convinced that the defendant honestly believed he owned, or had a right to, the disputed property. Is that a good idea? Why not tell people like Varszegi, who believe they are entitled to such property, to use legal processes to obtain the property or payment for an unpaid debt? Should landlords who hold mistaken beliefs about the law and use self-help to take a tenant's property as a means to collect a debt face criminal liability?

Note that it didn't matter to the court that Varszegi was informed by police officers that he had not legal entitlement to keep the tenant’s property. Should that matter? 

In Regina v. David Smith, 2 Q.B. 354 (1974) the defendant was charged under a statute that punished: “A person who, without lawful excuse destroys or damages any property belonging to another intending to destroy such property or being reckless as to whether any such property would be destroyed or damaged.” The defendant had damaged floor panels and floor boards that he had installed in his rental unit in order to remove stereo the stereo wires he had placed behind them. Although the trial court informed the jury that the defendant’s (incorrect) belief that the flooring was his was not relevant, the appellate court reversed: “[i]n our judgment no offence is committed under this section if a person destroys or causes damage to property belonging to another if he does so in the honest though mistaken belief that he property is his own.”

 

4.4 Model Penal Code 4.4 Model Penal Code

Commonwealth v. Camacho, 2019 MP 2 (N. Mariana Islands 2019), provides a basic account of the motivation for the Model Penal Code and reform of common law mens rea terms:

Prior to the MPC's implementation of mental states, "[t]he common law traditionally used a variety of overlapping and, frankly, confusing phrases to describe culpable mental states—among them, specific intent, general intent, presumed intent, willfulness, and malice." Voisine v. United States, 136 S. Ct. 2272, 2281 (2016). "In 1970, the National Commission on Reform of Federal Criminal Laws decried the 'confused and inconsistent ad hoc approach' of the federal courts" to mens rea.… The movement to redefine mens rea resulted in a "codification of workable principles for determining criminal culpability," as well as "a general rethinking of traditional mens rea analysis." This new approach moved "away from the traditional dichotomy of intent and toward an alternative analysis of mens rea," as exemplified in the American Law Institute's Model Penal Code. As a result, "[t]he MPC does not explicitly distinguish between general and specific intent, but instead classifies crimes based on whether the act is done . . . purposely, knowingly, recklessly, or negligently." [Citation omitted.]

MPC culpability requirements MPC culpability requirements

MPC § 2.02(2)(a)-(d) defines four distinct 'culpability requirements,' or mental states, that have been adopted in perhaps twenty or so state criminal codes and have proven influential in others. Those 'kinds of culpability' are listed in hierarchy from most to least blameworthy or serious: 'purposely,' 'knowingly,' 'recklessly,' and 'negligently.' Section 2.02 (and the rest of the MPC) is available on Lexis, HeinOnline, and elsewhere. These provisions are widely quoted in case law. See, for example, Commonwealth v. Camacho, 2019 MP 2, 15 & n.6 (N. Mariana Islands 2019):

The MPC states that generally, "a person is not guilty of an offense unless he acted purposely, knowingly, recklessly or negligently, as the law may require, with respect to each material element of the offense." Model Penal Code § 2.02. It also provides detailed definitions as to each level of culpability.6

[In MPC § 2.02,] purposely, knowingly, and recklessly are defined as follows:

(a) Purposely. A person acts purposely with respect to a material element of an offense when:

(i) if the element involves the nature of his conduct or a result thereof, it is his conscious object to engage in conduct of that nature or to cause such a result; and

(ii) if the element involves the attendant circumstances, he is aware of the existence of such circumstances or he believes or hopes that they exist.

(b) Knowingly. A person acts knowingly with respect to a material element of an offense when:

(i) if the element involves the nature of his conduct or the attendant circumstances, he is aware that his conduct is of that nature or that such circumstances exist; and

(ii) if the element involves a result of his conduct, he is aware that it is practically certain that his conduct will cause such a result.

(c) Recklessly. A person acts recklessly with respect to a material element of an offense when he consciously disregards a substantial and unjustifiable risk that the material element exists or will result from his conduct. The risk must be of such a nature and degree that, considering the nature and purpose of the actor's conduct and the circumstances known to him, its disregard involves a gross deviation from the standard of conduct that a law-abiding person would observe in the actor's situation.

For § 2.02 (2)(d) definition of negligently, see, e.g., Carrell v. United States, 165 A.3d 314, 322 n.21(D.C. Ct. App. 2017):

"A person acts negligently with respect to a material element of an offense when he should be aware of a substantial and unjustifiable risk that the material element exists or will result from his conduct. The risk must be of such a nature and degree that the actor's failure to perceive it, considering the nature and purpose of his conduct and the circumstances known to him, involves a gross deviation from the standard of care that a reasonable person would observe in the actor's situation."

 

MPC rules of interpretation MPC rules of interpretation

In addition to providing definitions of four distinct mental states, the MPC offers rules designed to help lawyers and judges (and, in theory, ordinary people) interpret criminal statutes—specifically, rules to determine which culpability term applies to each part of a criminal statute. The most important of these rules are those in MPC § 2.02(3), (4) & (9). Pennsylvania, for example, has adopted all three of these MPC provisions in its state criminal code. MPC § 2.02(3), (4) & (9) are identical to 18 Pa. C.S. § 302(c), (d) & (h) reprinted here:

[3] Culpability required unless otherwise provided.—When the culpability sufficient to establish a material element of an offense is not prescribed by law, such element is established if a person acts intentionally, knowingly or recklessly with respect thereto.

[4] Prescribed culpability requirement applies to all material elements.--When the law defining an offense prescribes the kind of culpability that is sufficient for the commission of an offense, without distinguishing among the material elements thereof, such provision shall apply to all the material elements of the offense, unless a contrary purpose plainly appears.

[9] Culpability as to illegality of conduct.--Neither knowledge nor recklessness or negligence as to whether conduct constitutes an offense or as to the existence, meaning or application of the law determining the elements of an offense is an element of such offense, unless the definition of the offense or this title so provides.

See also the interpretive rules in MPC § 2.02(6), (7) and (8), which have been enacted as 18 Pa. C.S. § 302(f), Del. Code Ann. tit. 11, § 255, and 18 Pa. C.S. § 302(g), respectively:

[6] Requirement of intent satisfied if intent is conditional.--When a particular intent is an element of an offense, the element is established although such intent is conditional, unless the condition negatives the harm or evil sought to be prevented by the law defining the offense. [Note: Pennsylvania, like a few other states adopting MPC provisions, chose to use the word ‘intent’ instead of ‘purpose,’ although it gave ‘intent’ the same definition as the MPC provides for ‘purpose.’]

[7] Knowledge of high probability. When knowledge of the existence of a particular fact is an element of an offense, such knowledge is established if a person is aware of a high probability of its existence, unless the person actually believes that it does not exist.

[8] Requirement of willfulness satisfied by acting knowingly.--A requirement that an offense be committed willfully is satisfied if a person acts knowingly with respect to the material elements of the offense, unless a purpose to impose further requirements appears.

MPC rules regarding causation of results MPC rules regarding causation of results

MPC § 2.03 provides guidance for applying statutes that require an actor's conduct to cause some result (such as death, or property loss). Pennsylvania enacted MPC § 2.03 at 18 Pa. C.S. § 303 (again, except that it substituted the word 'intent' where the MPC uses 'purpose'):

 Causal relationship between conduct and result.

(a)  General rule.--Conduct is the cause of a result when:

(1)  it is an antecedent but for which the result in question would not have occurred; and

(2)  the relationship between the conduct and result satisfies any additional causal requirements imposed by this title or by the law defining the offense.

(b)  Divergence between result designed or contemplated and actual result.--When intentionally or knowingly causing a particular result is an element of an offense, the element is not established if the actual result is not within the intent or the contemplation of the actor unless:

(1)  the actual result differs from that designed or contemplated as the case may be, only in the respect that a different person or different property is injured or affected or that the injury or harm designed or contemplated would have been more serious or more extensive than that caused; or

(2)  the actual result involves the same kind of injury or harm as that designed or contemplated and is not too remote or accidental in its occurrence to have a bearing on the actor's liability or on the gravity of his offense.

(c)  Divergence between probable and actual result.--When recklessly or negligently causing a particular result is an element of an offense, the element is not established if the actual result is not within the risk of which the actor is aware or, in the case of negligence, of which he should be aware unless:

(1)  the actual result differs from the probable result only in the respect that a different person or different property is injured or affected or that the probable injury or harm would have been more serious or more extensive than that caused; or

(2)  the actual result involves the same kind of injury or harm as the probable result and is not too remote or accidental in its occurrence to have a bearing on the liability of the actor or on the gravity of his offense.

(d)  Absolute liability.--When causing a particular result is a material element of an offense for which absolute liability is imposed by law, the element is not established unless the actual result is a probable consequence of the conduct of the actor.

State v. Lozier (Ohio 2004) State v. Lozier (Ohio 2004)

101 Ohio St.3d 161, 803 N.E.2d 770 (2004)

PFEIFER, J.

On February 14, 2001, the Holmes County Grand Jury indicted defendant-appellee, Chad A. Lozier, for trafficking in drugs in violation of [Ohio Rev. Code §] 2925.03(A). All … counts against appellee contained a specification that appellee sold drugs within the vicinity of a school, which enhances the penalty under R.C. 2925.03(C)(5)(b). Appellee withdrew his initial plea of not guilty and entered pleas of no contest to [all] counts.

 The sales at issue occurred at appellee’s former home, which is located approximately 745 feet from the Holmes County Job and Family Services building. That building housed, on its third floor, a remedial education program known as “Project Stay.” The state stipulated that it had no evidence that appellee knew of the existence of Project Stay and that there was nothing to mark it or to identify it as a school to the public.

 The trial court ruled that Project Stay was, in fact, a school. The court also ruled that whether appellee knew that he was selling drugs in the vicinity of the school was irrelevant, since the specification for trafficking within the vicinity of a school is written in terms of strict liability. Appellee was convicted on four counts of trafficking with the sentence-enhancement specifications.

 Appellee appealed from his convictions to the Fifth District Court of Appeals, arguing that the trial court had erred in finding that R.C. 2925.03(C)(5)(b) imposes strict liability. The appellate court agreed and reversed the judgment of the trial court, concluding that the culpable mental state associated with R.C. 2925.03(C)(5)(b) is “knowingly.”

 The sole issue raised in this appeal is whether R.C. 2925.03(C)(5)(b), which elevates trafficking in LSD to a fourth-degree felony if the offense is committed “in the vicinity of a school,” imposes strict criminal liability on a defendant. We make our determination against the backdrop of R.C. 2901.04(A), which states that “[s]ections of the Revised Code defining offenses or penalties shall be strictly construed against the state, and liberally construed in favor of the accused.”

 The case against appellee begins with his violation of R.C. 2925.03(A), which itself requires a mental state of “knowingly”:

“No person shall knowingly do any of the following:

 “(1) Sell or offer to sell a controlled substance; …

R.C. 2925.03(C)(5)(b) is in play because appellee was selling LSD, and because he was selling that substance in the vicinity of a school. The relevant statutory language is as follows:

“(C) Whoever violates division (A) of this section is guilty of one of the following: … 

“(5) If the drug involved in the violation is L.S.D. or a compound, mixture, preparation, or substance containing L.S.D., whoever violates division (A) of this section is guilty of trafficking in L.S.D. The penalty for the offense shall be determined as follows: …

 “(b) … [I]f the offense was committed in the vicinity of a school or in the vicinity of a juvenile, trafficking in L.S.D. is a felony of the fourth degree, and division (C) of section 2929.13 of the Revised Code applies in determining whether to impose a prison term on the offender.”

The mental state of the offender is a part of every criminal offense in Ohio except for those plainly imposing strict liability. R.C. 2901.21(A)(2) requires that, in order to be found guilty of a criminal offense, a person must have “the requisite degree of culpability for each element as to which a culpable mental state is specified by the section defining the offense.”

          R.C. 2901.21(B) addresses strict liability statutes and those statutes that do not address a culpable mental state. That statute reads:

“When the section defining an offense does not specify any degree of culpability, and plainly indicates a purpose to impose strict criminal liability for the conduct described in the section, then culpability is not required for a person to be guilty of the offense. When the section neither specifies culpability nor plainly indicates a purpose to impose strict liability, recklessness is sufficient culpability to commit the offense.”

Thus, recklessness is the catchall culpable mental state for criminal statutes that fail to mention any degree of culpability, except for strict liability statutes, where the accused’s mental state is irrelevant. However, for strict liability to be the mental standard, the statute must plainly indicate a purpose to impose it. …

          Here, we are dealing with another pair of discrete clauses separated by “or.” The phrase at issue in R.C. 2925.03(C)(5)(b) reads:

“[I]f the offense was committed in the vicinity of a school or in the vicinity of a juvenile, trafficking in L.S.D. is a felony of the fourth degree, and division (C) of section 2929.13 of the Revised Code applies in determining whether to impose a prison term on the offender.” (Emphasis added.)

Standing alone, “in the vicinity of a school or in the vicinity of a juvenile” does not indicate any required degree of mental culpability. However, each clause, “in the vicinity of a school” and “in the vicinity of a juvenile,” is separately defined in the chapter’s definitional section. “In the vicinity of a juvenile,” as defined in R.C. 2925.01(BB), employs strict liability terms:

“(BB) An offense is ‘committed in the vicinity of a juvenile’ if the offender commits the offense within one hundred feet of a juvenile or within the view of a juvenile, regardless of whether the offender knows the age of the juvenile, whether the offender knows the offense is being committed within one hundred feet of or within view of the juvenile, or whether the juvenile actually views the commission of the offense.”

R.C. 2925.01(BB) makes it abundantly clear that the offender’s mental state is irrelevant in determining whether the offender has committed an offense “in the vicinity of a juvenile.” An offender is liable whether or not he knows the age of the juvenile, or whether he realizes that a juvenile is in the vicinity.

          By contrast, the definition of “in the vicinity of a school,” contained in R.C. 2925.01(P), lacks the express strict liability language of R.C. 2925.01(BB). That definition reads:

“(P) An offense is ‘committed in the vicinity of a school’ if the offender commits the offense on school premises, in a school building, or within one thousand feet of the boundaries of any school premises.”

The General Assembly, in imposing the strict liability requirement for drug sales “in the vicinity of a juvenile,” perfectly illustrates what R.C. 2901.21(B) calls a “purpose to impose strict liability.” Additionally, the stark contrast between the definition of “committed in the vicinity of a school” and the definition of “committed within the vicinity of a juvenile” indicates that the General Assembly did not intend to impose strict liability for selling LSD in the “vicinity of a school” section.

          … [I]f one part of a clause explicitly sets forth a mental state, that mental state does not apply to another discrete clause within that subsection. In fact, it is an indication that the General Assembly is attaching differing mental states as to the two distinct clauses. As the majority wrote in Maxwell, “[t]he decision in Wac demonstrates that a crime may have different degrees of mental culpability for different elements.” Maxwell, 95 Ohio St.3d 254, 2002-Ohio-2121, 767 N.E.2d 242, ¶ 30. We find that the language employed by the General Assembly in the R.C. 2925.01(P) and (BB) definitions establishes differing levels of culpability for offenses committed “in the vicinity of a juvenile” and “in the vicinity of a school,” plainly indicating that the General Assembly’s purpose was to impose strict liability for acts committed “in the vicinity of a juvenile” but not for acts committed “in the vicinity of a school.” 

Moreover, we find that the differing degrees of mental culpability for offenses committed near a school as opposed to near a juvenile are consistent with a coherent legislative policy. 

The distance requirement for an act to be committed within “the vicinity of a juvenile” is only 100 feet or “within view of the juvenile.” Drug trafficking is a dangerous activity. Beyond the psychic danger of seeing drugs being sold, there is a very real physical danger surrounding a drug transaction, even for nonparticipants. Thus, a child, whether in view or not, could become a part of the collateral damage of a failed transaction. The threat to a child is real and imminent. 

On the other hand, to be “in the vicinity of a school,” an offender could, by definition, be 1,000 feet away from a school. A child may not necessarily be nearby, or even in the school. The transaction could occur in the late evening hours, or in summer, or during any other period of the year that the school is closed. 

The difference between the potential peril of a transaction that occurs “in the vicinity of a school” and “in the vicinity of a juvenile” is significant. “In the vicinity of a school” addresses danger that can be theoretical; “in the vicinity of a juvenile” addresses a real, present danger. Because the “vicinity of a juvenile” offense is so much more dangerous, the General Assembly has determined that the offender’s knowledge that a juvenile is nearby is irrelevant. Also, since “in the vicinity of a juvenile” includes being in view of a juvenile, its parameters can expand well beyond 100 feet. Thus, if an offense occurs within 1,000 feet of a school, the offender still can be subject to strict criminal liability if there is a juvenile within view. As the danger to children becomes more real, the culpable mental state gets stricter. 

Having determined that the General Assembly’s purpose was to impose differing culpable mental states for acts committed “in the vicinity of a juvenile” and “in the vicinity of a school,” we must determine what culpable mental state applies to offenses that occur “in the vicinity of a school.” Since the definition of “in the vicinity of a school” includes no culpable mental state, we employ R.C. 2901.21(B) to provide the requisite culpability for the offense in question. We therefore hold that the culpable mental state of recklessness applies to the offense of trafficking in LSD “in the vicinity of a school” under R.C. 2925.03(C)(5)(b). 

Although the court of appeals found that the requisite mental state is “knowingly,” and we conclude that recklessness is the appropriate standard, we nevertheless affirm. A reviewing court is not authorized to reverse a correct judgment merely because it was reached for the wrong reason. 

Accordingly, we affirm the judgment of the court of appeals and remand the cause to the trial court for further proceedings.

O’CONNOR, J., dissenting.

Because I determine that R.C. 2925.03 imposes strict liability for trafficking in a controlled substance in the vicinity of schools, I respectfully dissent. 

R.C. 2901.21(B) cannot operate to supply the mental element of recklessness to R.C. 2925.03(C)(5)(b) because R.C. 2925.03 includes a mental element in defining Lozier’s offense. R.C. 2925.03(A) requires the mens rea of knowingly selling or offering to sell a controlled substance. R.C. 2925.03(C)(5)(b) then provides a sentencing enhancement “if the offense [of knowingly selling or offering to sell LSD] was committed in the vicinity of a school * * *.” Although the statute’s enhancement provision does not specify a degree of culpability, the section defining Lozier’s offense does. Thus, R.C. 2925.03(A)’s knowledge requirement precludes imputing recklessness to R.C. 2925.03(C)(5)(b) via R.C. 2901.21(B). 

Having resolved that R.C. 2901.21(B) is inapplicable here, we still must determine the degree of culpability required for a sentencing enhancement under R.C. 2925.03(C)(5)(b). …

[A] mental element cannot be imputed from one discrete clause to another and that the exclusion of a mental element from a discrete clause indicates the applicability of strict liability where the section defining the offense includes a mental element. I would follow Wac and Maxwell and hold that the knowledge requirement of R.C. 2925.03(A) cannot be imputed to R.C. 2925.03(C)(5)(b), and that the exclusion of a mental element from R.C. 2925.03(C)(5)(b), in light of R.C. 2925.03(A)’s knowledge requirement, subjects the described conduct to strict liability. … 

I find further support for my position that R.C. 2925.03(C)(5)(b) describes a strict liability offense in United States v. Falu, 776 F.2d 46(2nd Cir. 1985), which resolved the same issue presented herein within the context of former 21 U.S.C. § 845(a), a federal statute that was analogous to R.C. 2925.03. In Falu, the district court resolved that Section 845(a) imposed strict liability. Although I would not subjugate my independent judicial analysis to that of my learned colleagues on the federal bench, I recognize that the universal purpose of enhancing penalties for trafficking in drugs in the vicinity of schools is to protect children from the pariahs that are drug dealers. Thus, the Falu court found, “[A] requirement that the dealer know that a sale is geographically within the prohibited area would undercut this unambiguous legislative design.” Falu at 50. I wholeheartedly agree, and I believe that the same rationale applies to requiring recklessness.

Notes and Questions Notes and Questions

  1. The dissent in Lozier is persuaded by a federal court’s analysis of a similar statute. Does it matter that the federal Code does not contain an equivalent to MPC § 2.02(3) or § 2.02(4) while the Ohio legislature has enacted (roughly) equivalent provisions in Ohio Rev. Code § 2901.21(A), (B) & (C)(1)?
  2. Does it matter whether the “school vicinity” provision, Ohio Rev. Code § 2925.03(C)(5)(b), defines an “element” or a “sentencing enhancement”? Neither term is defined by the legislature—not in Ohio Code Rev. § 2901.01’s list of “general provisions definitions,” nor in Chapter 2929, which covers “Penalties and Sentencing” and includes a list of “penalties and sentencing general definitions.” Does that absence mean courts should not draw such a distinction? If they should, how can one tell which label applies to which parts of a criminal statute?

 

Ohio Rev. Code § 2901.21 Criminal liability, culpability.

(A) Except as provided in division (B) of this section, a person is not guilty of an offense unless both of the following apply:

(1) The person's liability is based on conduct that includes either a voluntary act, or an omission to perform an act or duty that the person is capable of performing;

(2) The person has the requisite degree of culpability for each element as to which a culpable mental state is specified by the language defining the offense.

(B) When the language defining an offense does not specify any degree of culpability, and plainly indicates a purpose to impose strict criminal liability for the conduct described in the section, then culpability is not required for a person to be guilty of the offense. The fact that one division of a section plainly indicates a purpose to impose strict liability for an offense defined in that division does not by itself plainly indicate a purpose to impose strict criminal liability for an offense defined in other divisions of the section that do not specify a degree of culpability.

(C)

(1) When language defining an element of an offense that is related to knowledge or intent or to which mens rea could fairly be applied neither specifies culpability nor plainly indicates a purpose to impose strict liability, the element of the offense is established only if a person acts recklessly. …

(F) As used in this section:

(1) Possession is a voluntary act if the possessor knowingly procured or received the thing possessed, or was aware of the possessor's control of the thing possessed for a sufficient time to have ended possession. …

(3) "Culpability" means purpose, knowledge, recklessness, or negligence, as defined in section 2901.22 of the Revised Code.

MPC Practice Problem MPC Practice Problem

Consider the following in light of the MPC's culpability provisions:

D wants to kill V, so D drives a car into V at a very high speed. At the time, V is holding their infant X, whom D loves and does not want to kill. Both V and X die.

a) With regard to V, what mens rea did D possess under 2.02?

b) With regard to X, what mens rea did D possess under 2.02?

Additional Mens Rea Problems Additional Mens Rea Problems

MENS REA PROBLEMS 

PROBLEM 1: Jane runs a business buying and reselling used goods on eBay. She recently bought 5 laptop computers with “Property of Santa Clara University” stickers on the backs.

Consider the consequences of her actions under these statutes:

California Penal Code sec.496 “Receiving Stolen Property” 

 Every person who buys or receives any property that has been stolen…, knowing the property to be so stolen or obtained, shall be punished by imprisonment in a county jail for not more than one year, …

California Penal Code section 2-24-7-1: 

The word "knowingly" imports only a knowledge that the facts exist which bring the act or omission within the provisions of this code. It does not require any knowledge of the unlawfulness of such act or omission.

1) Is she guilty of Receiving Stolen Property if, suspecting they were stolen, she decides not to resell the laptops?

2) Is she guilty of Receiving Stolen Property if the seller told her (and she believed him) that the laptops were discarded in a campus trash bin?

3) Is she guilty of Receiving Stolen Property if she decided not to unpack and examine the laptops prior to reselling them because the price was so low? 

4) What result under the same facts if California adopted 2.02(7)?

PROBLEM 2: D is a nurse in a nursing home in an MPC state. After months of sheltering in place, he decides he can’t stand the isolation, and he attends a wild indoor dance party. Unlike most of the guests, who didn’t wear masks, D wore his mask except when he needed to kiss someone. D returns to work the following week, and later tests positive for Covid-19. Then, a week later, one of D’s patients tests positive for the virus. 

D is charged with Felony Transmission of a Communicable Disease:  

A person commits a class “D” felony when the person is reckless as to whether they have a contagious or infectious disease and exposes an uninfected person to the contagious or infectious disease acting with a reckless disregard as to whether the uninfected person contracts the contagious or infectious disease, and the conduct results in the uninfected person becoming infected with the contagious or infectious disease. 

Which of the following is false, and why?

1) D will be found guilty if the failure to recognize the risk he posed was a gross deviation from the standard of care that a reasonable person would have exercised. 

2) D will be found guilty if the evidence shows D consciously disregarded the risk that he had been infected. 

3) D will be found not guilty if it turns out the patient caught the virus from someone else.

4) D will be found not guilty if D had no idea that one could transmit Covid-19 while wearing personal protective equipment (PPE)

PROBLEM 3: D intends to poison her estranged spouse, who now lives with his girlfriend. She places poison gas in his car, and both he and the girlfriend die. How many counts of first-degree murder can Arizona bring against D?

e.g. Arizona law: 13-1105. First degree murder; classification

A person commits first degree murder if…the person causes the death of another person, with premeditation...

Different result under MPC (see 2.03(2), (a) and (b))?

 

PROBLEM 4: D robs bank, and a security guard chases him out, grabbing the car, and holding onto it as D drives away. D guns the engine and swerves, trying to frighten the guard into letting go. The guard falls off and is run over by another car, which kills him instantly.

In MPC-land, is D guilty of Murder 210.2 or Manslaughter 210.3?

PROBLEMS 5-8: Use the following criminal statutes to determine what crimes, if any, might apply to the hypotheticals below.

S 140.17 Criminal trespass in the first degree.

  A person is guilty of criminal trespass in the first degree when he knowingly enters or remains unlawfully in a building that is the property of another.

S 140.10 Theft

  A person is guilty of theft if he unlawfully takes the property of another with the purpose to deprive him thereof.

S 140.20 Burglary in the third degree.

  A person is guilty of burglary in the third degree when he knowingly enters a building with intent to commit a crime therein.

S 140.25 Burglary in the second degree.

A person is guilty of burglary in the second degree when he knowingly enters a building with intent to commit a crime therein, and when:

  1. In effecting entry or while in the building or in immediate flight therefrom, he or another participant in the crime is armed with explosives or a deadly weapon; or
  1. The building is a dwelling.

 
S 140.35 Possession of burglar’s tools.

 A person is guilty of possession of burglar`s tools when he possesses any tool, instrument or other article adapted, designed or commonly used for committing or facilitating offenses involving forcible entry into premises, … under circumstances evincing an intent to use or knowledge that some person intends to use the same in the commission of an offense of such character.

Problem 5:

Sam has agreed to feed his friend's dog when she is away.  Forgetting the address of her house, he enters a stranger's house by mistake.   Looking around and seeing some money lying on a table, he takes it and leaves.

Problem 6:  

Fred, looking for something of value to steal, breaks into what he takes to be an uninhabited house under construction and starts looking around.  He is surprised to find Owen, the owner, living there.

Problem 7:

Bored one night, Jane and his schoolmate Larry break into the local mall at night, as they later say, "just for kicks."  But unknown to Jane, Larry has placed explosives in Jane's backpack.

Problem 8:

Mags is holding a crowbar, and says to an undercover officer, "here's what my buddy Jack uses when he's short on cash."  When arrested, Mags says, "How can I be charged -- who knew that holding a crowbar was a crime?"

4.5 Optional materials 4.5 Optional materials

The following cases and materials are not required reading. You might find them useful as additional sources and examples on the topics in this chapter. As time allows, may use some in class as practice/review problems.

4.5.1 State v. Blurton 4.5.1 State v. Blurton

537 S.E.2d 291

The STATE of South Carolina, Respondent, v. Gregory R. BLURTON, Appellant.

No. 3236.

Court of Appeals of South Carolina.

Heard June 7, 2000.

Decided Aug. 7, 2000.

Rehearing Denied Oct. 7, 2000.

*504Tara Dawn Shurling, of Columbia, for appellant.

Attorney General Charles M. Condon, Chief Deputy Attorney General John W. McIntosh and Senior Assistant Attorney General Harold M. Coombs, Jr., all of Columbia; and Solicitor Walter M. Bailey, Jr., of Summerville, for respondent.

CURETON, Judge:

A jury convicted Gregory R. Blurton of two counts of armed robbery, kidnapping, grand larceny of a motor vehicle, and *505failure to stop for a blue light. The trial court sentenced Blurton to three life sentences for the kidnapping and armed robbery convictions, ten years for grand larceny, and three years for failing to stop for a blue light, with all sentences running concurrently. Blurton appeals. We reverse and remand for a new trial.

FACTS

On April 6, 1997, Blurton entered the Orangeburg WalMart between 11:30 p.m. and 12:00 midnight. After asking for Roger, an assistant manager who was not working that night, Blurton told assistant manager Brandon Beckman he needed access to the electrical room to do some work.1 Beckman tried to open the room but did not have a key to the lock.

As Blurton and Beckman walked away, Blurton pulled out a gun and demanded that Beckman take him to the cash office and give him the money inside. Brenda Arant, the cash office clerk, was still inside the office. Blurton ordered Beckman to get on the floor and demanded Arant fill a shopping bag with money. Blurton screamed his orders and threatened to shoot Beckman and Arant if they did not cooperate. Blurton told Beckman he needed Beckman to drive him down the-road. After Beckman responded that he did not have a car at the store, Blurton demanded Arant’s keys. When Arant informed Blurton her car keys were in her locker, Blurton bit off the end of the phone cord, told Arant not to call anybody for at least five minutes, and pushed Beckman out of the office. Although there was in excess of $200,000 in the cash office, Blurton left with only $8,500.

After leaving the cash office, Blurton put the gun in his pocket and forced Beckman to walk outside the store with him. Once outside, Blurton chose the car of a female customer who had just arrived in the parking lot. As the men approached the woman, Blurton pulled the gun out again. The woman threw down her purse, screamed, and started to run away. While pointing the gun at her, Blurton demanded her keys. The woman threw them at him, whereupon Blurton climbed into her car and drove away.

*506A short time after leaving the parking lot, Blurton ran a red light, attracting the attention of a police officer. When the officer attempted to stop Blurton, Blurton led the police on a high speed chase which ended after he wrecked the customer’s car.

At trial, Blurton did not deny committing the acts of which he was accused, but argued a lack of criminal intent. During his testimony, he explained that James Mayfield, another assistant manager of the Orangeburg Wal-Mart, was a former Navy SEAL who also worked for the Central Intelligence Agency (CIA). He further explained that Mayfield recruited him to assist the CIA in its effort to infiltrate a drug cartel. In exchange for his assistance, Blurton was promised a new identity and a presidential pardon for several bank robberies he committed in the 1970s. Blurton claimed Mayfield gave him surveillance and firearms training and loaned him hand to hand combat videos developed by a Navy SEAL instructor. Additionally, Blurton testified he had completed several assignments in relation to the CIA’s infiltration plan, codenamed “Operation Double White.”

The “staged” robbery of the Wal-Mart was Phase Three of the operation, an attempt to convince the drug cartel that Blurton was a wanted and dangerous man, thereby facilitating his infiltration of the group. He explained the robbery was fake and, according to Mayfield, both Beckman and Roger were aware of the plan. Mayfield furnished the gun for Blurton to use during the staged crimes. While in the cash office, Blurton claims he decided to have Arant fill the bag with one dollar bills, rather than with the sizeable completed deposits, as Mayfield had instructed, because he did not want to be responsible for a large sum of money. Blurton was supposed to leave the money at a specific drop location at the back of a hotel, where Mayfield would retrieve it and promptly return it to Wal-Mart on behalf of the CIA. According to Blurton, he had to pretend the robbery was real to put on a “show” for the security cameras and members of the local sheriffs department who, according to Mayfield, were involved with the drug cartel.

Blurton stated that when he saw the customer’s car in the parking lot, he assumed it was his “ride.” He thought the *507customer was cooperating with the CIA and that her show of fear and panic was an act. He testified that he evaded police officers because he was waiting for the CIA to intercede on his behalf, so he would not get caught.

Although Blurton consistently maintained that he was working for the CIA when he committed the acts involved in each of the charged offenses, his trial counsel simply attempted to prove Blurton genuinely believed he was a CIA operative at the time of the alleged crimes and thus lacked criminal intent.

LAW/ANALYSIS

I. Evidentiary Issues

Blurton’s trial counsel attempted to present evidence which would corroborate his explanation of events and support his defense of lack of criminal intent. The evidence included (1) taped telephone conversations between Blurton and Mayfield; (2) testimony that the investigating officer thought Blurton actually believed he was part of a CIA operation; (3) testimony that others heard and believed Mayfield’s claim that he was a former Navy SEAL; and (4) two newspaper articles which showed Mayfield convinced the local newspaper he was a Navy SEAL. The trial court excluded this evidence. Blurton argues this was error.

Decisions regarding the admissibility of evidence are within the trial court’s sound discretion and will not be reversed on appeal absent a prejudicial abuse of that discretion. State v. Smith, 337 S.C. 27, 522 S.E.2d 598 (1999) (citing State v. Nance, 320 S.C. 501, 466 S.E.2d 349 (1996)); State v. Fulton, 333 S.C. 359, 509 S.E.2d 819 (Ct.App.1998).

A. Taped Conversations

Blurton’s counsel sought to admit taped telephone conversations between Blurton and Mayfield which were made without Mayfield’s knowledge. Blurton made the tapes after the police investigator concluded Mayfield was also a suspect based on interviews with Blurton. During the conversations, Mayfield repeatedly affirmed or failed to deny Blurton’s statements that Mayfield had involved him in the CIA operation and that the robbery was staged. The trial court excluded the *508tape recordings as inadmissible hearsay, but allowed the defense to proffer the tapes and transcripts prepared from them into evidence.2 We agree that this evidence should have been admitted.

Hearsay is generally inadmissible. Rule 802, SCRE. “Hearsay is an out-of-court statement offered in court to prove the truth of the matter asserted.” Jackson v. Speed, 326 S.C. 289, 304, 486 S.E.2d 750, 758 (1997); see also Rule 801(c), SCRE (“ ‘Hearsay’ is a statement, other than one made by the declarant while testifying at the trial or hearing, offered in evidence to prove the truth of the matter asserted.”). A statement that is not offered to prove the truth of the matter asserted should not be excluded as hearsay. See State v. Blackburn, 271 S.C. 324, 247 S.E.2d 334 (1978) (statement implicating defendant in alleged prior crimes, which was not offered to prove the truth of the matter asserted, that is, that defendant in fact committed the prior crimes, but to establish motive, was not “hearsay” and its admission was not error); Hawkins v. Pathology Assocs. of Greenville, P.A., 330 S.C. 92, 498 S.E.2d 395 (Ct.App.1998) (allowing admission of letters, an anniversary card, and video to show close familial bond between the decedent, her husband, and her children in a malpractice action).

In this case, the tapes were not offered to prove the veracity of any statements made during the conversations between Blurton and Mayfield. Rather, they were offered to show that Blurton had been led to believe the robbery was staged and that his actions were sanctioned by the CIA. Because these conversations would have added credence to Blurton’s defense that he lacked criminal intent and were not hearsay, the trial court erred in excluding them. See State v. Harris, 311 S.C. 162, 167, 427 S.E.2d 909, 912 (Ct.App.1993) (“Due process requires that a criminal defendant be given a reasonable opportunity to present a complete defense.”).

B. Investigator’s Opinion on Blurton’s State of Mind

In a preliminary hearing, Detective Jack Coleman, the investigator in this case, said he was convinced Blurton be*509lieved he was involved in a CIA operation when he robbed the Wal-mart. The court excluded the testimony, determining it was the province of the jury to determine whether Blurton believed he was working for the CIA when he committed the charged crimes.3 We find no abuse of discretion.

“The general rule is' that opinion testimony which is determinative of the ultimate fact in issue should be excluded as an invasion of the province of the factfinder. This rule, however, is not inflexible.” Richmond v. Tecklenberg, 302 S.C. 331, 334, 396 S.E.2d 111, 113 (Ct.App.1990) (citing State v. Moorer, 241 S.C. 487, 129 S.E.2d 330 (1963), overruled on other grounds by State v. Torrence, 305 S.C. 45, 406 S.E.2d 315 (1991)); see also State v. Koon, 278 S.C. 528, 298 S.E.2d 769 (1982), overruling on other grounds recognized by State v. Matthews, 291 S.C. 339, 353 S.E.2d 444 (1986) (holding trial court properly excluded testimony concerning whether a mitigating factor was present as such determination was the province of the jury). Lay witnesses are permitted to offer opinion testimony when such testimony is rationally related to the witness’s perception, does not require special knowledge, and may assist the jury’s understanding of the witness’s testimony. See Rule 701, SCRE.

While we believe the trial court could have properly admitted the detective’s prior testimony into evidence, the court’s refusal to do so does not constitute an abuse of discretion. Blurton did not attempt to offer the detective as an expert in the beliefs of others. The detective’s opinion was not necessary for the jury to understand other portions of his testimony. Furthermore, the testimony was the investigator’s opinion on the ultimate factual issue in the case — whether Blurton believed he was working for the CIA and lacked criminal intent. Therefore, we hold the trial court’s refusal to admit the prior testimony was not reversible error.

*510On appeal, Blurton argues the detective’s opinion was admissible as an admission under Rule 801(d)(2)(A), SCRE.4 This rule provides an exception to the hearsay rule. Thus, it would allow the admission of evidence which would otherwise be excluded as hearsay. The detective’s opinion in this case was not excluded as hearsay, but because the trial court determined it was the jury’s province to determine whether Blurton believed he was working for the CIA when he committed the charged crimes. Therefore, whether the detective’s opinion was admissible under Rule 801(d)(2)(A) is not relevant to the trial court’s ruling concerning the testimony’s admissibility.

C. Whether Others Believed Mayfield to be a Navy SEAL

Blurton also attempted to introduce testimony that other people at Wal-Mart heard and believed Mayfield’s statements that he was a former Navy SEAL. The trial court excluded the evidence, concluding it was both hearsay and irrelevant. Blurton asserts the court abused its discretion in excluding the evidence. We agree.

The testimony did not constitute hearsay because it was not offered to prove the truth of any out of court statements and should not have been excluded on those grounds. See Rule 801(c), SCRE. Blurton’s counsel did not offer evidence that Mayfield represented himself to be a former Navy SEAL to prove Mayfield actually was one. In fact, it is undisputed that Mayfield was never a Navy SEAL. The evidence was offered, in part, to corroborate Blurton’s testimony that Mayfield told him and he believed Mayfield was a former SEAL. Therefore, the evidence should not have been excluded as hearsay.

Nor was the testimony irrelevant. Essentially, the excluded testimony would have shown that it was common belief among Wal-Mart employees, fueled by Mayfield’s own statements, that he was a former Navy SEAL. The fact that Mayfield told this falsehood about his past to others, and that they believed him, tended to corroborate Blurton’s claimed *511belief in the same falsehood. This would help explain why Blurton would believe Mayfield’s claim that he worked for the CIA. Furthermore, the fact that it was common knowledge that Mayfield was a former SEAL lends credibility to Blur-ton’s alleged belief that other members of management were aware of his purported CIA status and were cooperating with him in the “staged” robbery. Thus, the evidence shows Blurton’s state of mind at the time he committed the crimes. The evidence was relevant to Blurton’s argument that he lacked criminal intent, a necessary element of the charged crimes. See State v. Ferguson, 302 S.C. 269, 395 S.E.2d 182 (1990) (statutory crimes ordinarily include some elements of intent); State v. Thrailkill, 73 S.C. 314, 53 S.E. 482 (1906) (criminal intent is an essential element in every common law crime).

D. Newspaper Articles

Blurton also offered two newspaper articles printed by Orangeburg’s newspaper, The Times and Democrat.5 In the first article, entitled “Never Give Up,” the newspaper recounted Mayfield’s fictional history as a Navy SEAL. Mayfield was quoted in the article describing the rigorous training and lessons he gained from his experience as a SEAL. The second article, printed after Mayfield’s arrest, discussed the Wal-Mart robbery and, in retracting the prior article, stated that Mayfield had never been a Navy SEAL. The trial court held both articles inadmissible hearsay. We agree with Blur-ton that the articles should have been admitted.

The newspaper articles did not constitute hearsay because they were not offered for the truth of the statements contained in them, but to show Mayfield’s ability to persuade and trick others. Because of the purpose for which they were offered, the articles clearly were not hearsay and were improperly excluded. See Jackson, 326 S.C. 289, 486 S.E.2d 750; Rule 801(c), SCRE.

*512II. State’s Closing Argument

Blurton also asserts the trial court erred in overruling his objection, denying his. motion to strike and his motion for a mistrial when the assistant solicitor accused him of lying in his testimony.

During closing arguments, the assistant solicitor accused Blurton twice of lying based on a defense psychiatrist’s admission that she “diagnosed him as suffering from malingering, which is the deliberate production of false symptoms for the purpose of secondary gain.” Blurton objected and moved to strike, arguing it is improper to call the defendant a liar in closing arguments. The trial court responded by instructing the prosecutor to “Go ahead.” At the close of the State’s argument, Blurton moved for a mistrial, which the court denied.

Our supreme court has previously held it is improper to call a party a liar in closing argument. See Major v. Alverson, 183 S.C. 123, 190 S.E. 449 (1937). However, not all improper closing arguments require reversal. In criminal cases, “[a] new trial will not be granted unless the prosecutor’s comments so infected the trial with unfairness as to make the resulting conviction a denial of due process.” State v. Huggins, 325 S.C. 103, 107 481 S.E.2d 114, 116 (1997). The propriety of the state’s closing argument will be examined in light of the entire record. State v. Nathari, 303 S.C. 188, 399 S.E.2d 597 (Ct.App.1990).

While we agree that the assistant solicitor’s comments were improper, they do not mandate reversal alone. However, the cumulative effect of this error, when coupled with the exclusion of the previously discussed evidence, warrants reversal. Cf. State v. Johnson, 334 S.C. 78, 93, 512 S.E.2d 795, 803 (1999) (citations omitted) (“cumulative error doctrine provides relief to a party when a combination of errors that are insignificant by themselves have the effect of preventing a party from receiving a fair trial and it requires the cumulative effect of the errors to affect the outcome of the trial”). The prosecutor accused Blurton of a recent fabrication, an accusation which could not have been seriously made if the trial court had properly admitted the taped telephone conversations between Blurton and Mayfield. In that regard, the closing *513arguments served to exacerbate the error in excluding evidence critical to Blurton’s defense.

III. Jury Instructions

Blurton also argues the trial court erred in denying his request for a specific jury instruction and in issuing an improper instruction. We find no error.

The law to be charged is determined by the evidence presented at trial. State v. Gourdine, 322 S.C. 396, 472 S.E.2d 241 (1996). If the evidence supports a particular requested charge, the trial court commits reversible error by refusing to issue it. State v. Burriss, 334 S.C. 256, 513 S.E.2d 104 (1999).

In discussing jury instructions, Blurton requested an instruction on actual and apparent authority. The trial court declined the request and noted that it would, contrary to the request, instruct the jury that acting on the instructions of another is no defense. In its charge, the court stated:

Now, I tell you, the law is, if one committed a criminal act it is no defense to show that it was done under the instructions or orders from another. Likewise, it is no defense to a criminal act if it be shown that it was done in partnership or cooperation with another person.

Blurton objected to the instruction. On appeal, Blurton argues the trial court erred in refusing the requested charge and in charging the jury as it did.

The trial court correctly refused the requested instruction. Actual and apparent authority are principles of agency law rather than criminal law. However, even if these concepts could be used as a defense to criminal charges, Blurton did not establish sufficient evidence to require the requested charge. There is no evidence Mayfield had actual authority, as a Wal-Mart assistant manager or as a governmental official, to authorize Blurton to commit any of the alleged crimes.

“Apparent authority to do an act is created as to a third person by written or spoken words or any other conduct of the principal which, reasonably interpreted, causes the third person to believe the principal consents to have the act done on his behalf by the person purporting to act for him.” *514Genovese v. Bergeron, 327 S.C. 567, 575, 490 S.E.2d 608, 612 (Ct.App.1997). Blurton presented no evidence of Mayfield’s apparent authority to permit these crimes. There was no evidence that either Wal-Mart or the CIA, acting as a principal, did any act which would have caused Blurton to believe Mayfield could authorize a real or “staged” robbery of the Wal-Mart store. Because the evidence did not require the requested charge, the court properly refused to issue it. See Dalon v. Golden Lanes, Inc., 320 S.C. 534, 540, 466 S.E.2d 368, 372 (Ct.App.1996) (citations omitted) (“It is the trial court’s function to charge the jury on the applicable law as raised by the pleadings and supported by the evidence. In order to warrant reversal for failure to give a requested charge, the refusal must be both erroneous and prejudicial.”).

Furthermore, the court’s charge that it is no defense that a person acted upon the instructions of, or in cooperation with, another person was proper. The instruction is a correct statement of the law and was triggered by Blurton’s testimony that he took money from Wal-Mart with Mayfield’s assistance and permission. See State v. Hurt, 212 S.C. 461, 48 S.E.2d 313 (1948) (it is no defense to a crime that others who are not on trial participated in the crime); 22 C.J.S. Criminal Law § 88 (1989) (Generally, the fact that one commits a crime on the advice or direction of another person is no defense to his commission of the crime.).

IV. Directed Verdict

Finally, Blurton argues the trial court erred in denying his directed verdict motion on the grand larceny charge. Specifically, he argues that double jeopardy prevents his conviction for both armed robbery, for taking the customer’s car keys, and grand larceny, for taking her car. Although Blurton moved for a directed verdict on the ground of insufficiency of the evidence after the State rested, he never argued to the trial court, as he does now, that he could not be convicted of both crimes. Because he failed to assert his double jeopardy argument at trial, the issue' is not preserved for direct appeal. See State v. Byram, 326 S.C. 107, 485 S.E.2d 360 (1997) (party may not argue one ground at trial and another on appeal); Medlock v. One 1985 Jeep Cherokee VIN 1JCWB7828FT129001, 322 S.C. 127, 470 S.E.2d 373 *515(1996) (double jeopardy issue must be raised to the trial court to be preserved for appellate review).

CONCLUSION

For the reasons discussed, Blurton’s appealed convictions are reversed and his case is remanded for a new trial.

REVERSED AND REMANDED.

GOOLSBY and SHULER, JJ., concur.

4.5.2 Notes and Questions (State v. Blurton) 4.5.2 Notes and Questions (State v. Blurton)

Notes & Questions 

  1. The defense's case for Blurton rested on what is called a "failure of proof" argument. If Blurton genuinely believed the assistant manager's story, it would mean that the state failed to prove that he had the requisite mens rea. This kind of argument is helpful to the defense because the burden to prove each element of a crime beyond a reasonable doubt rests with the prosecution. Could an "insanity defense" or a showing of diminished mental capacity have also worked for Blurton? What evidence would he have needed to make such claims? Which defense (failure-of-proof vs. insanity vs. diminished capacity) strikes you as the best argument in Blurton's favor? Why? 

  2. On a different note: Why should we let Blurton off the hook? Couldn't Blurton's stupidity in believing the Navy Seal-mastermind story easily have resulted in serious harm? What justifications for punishment might be relevant to consider as you answer this question? 

  3. You are the prosecutor in this case. What, if anything, is the mastermind-assistant manager guilty of? 

  4. Even as we continue to apply the legal framework of mens rea, it is important to keep in mind that these legal classifications rest on somewhat antiquated understandings of the human mind. The possibility that new developments in neuroscience might raise concerns about how criminal law understands volition and intent is actually a hotly debated issue within legal scholarship. Consider the following passage, for example: 

Although the Model Penal Code brought much-needed clarity to the concepts of actus reus and mens rea, it has by no means solved the conceptual problems of what kinds of things should fall into the categories of volition, intent, and choice. For one thing, . . . both conscious, voluntary conduct (actus reus) and the various levels of intent (mens rea) require a reasoning process relating what the person perceives to be reality with the situation within which he must act (or not) and how. This reasoning, or imaginative process, may be greatly altered if the brain is not functioning correctly . . . . In other words, the law attempts to separate categories that are not really separable.

[T]he legal meaning of choice, intent, and volition originated, not from empirical studies about human brains and behavior, but from ungrounded beliefs about human nature. Despite a paradigm shift in understanding human behavior that has occurred through new developments in neurobiology, the law still operates on outdated nineteenth century assumptions about how human beings function. As a result, the categories into which some of these human actions belong are porous and indeterminate. The separate legal categories of actus reus and mens rea and their meanings reflect outdated assumptions about human nature and the objectives of criminal justice.

Erica Beecher-Monas and Edgar Garcia-Rill, Actus Reus, Mens Rea, and Brain Science: What Do Volition and Intent Really Mean? Kentucky Law Journal, 106: 265-314, 268 (2017). Entire article available at: https://uknowledge.uky.edu/klj/vol106/iss2/5

4.5.3 People v. Conley 4.5.3 People v. Conley

THE PEOPLE OF THE STATE OF ILLINOIS, Plaintiff-Appellee, v. WILLIAM J. CONLEY, Defendant-Appellant.

First District (3rd Division)

No. 1—86—2651

Opinion filed August 2, 1989.

*236Anna Ahronheim, of State Appellate Defender’s Office, of Chicago, for appellant.

*237Cecil A. Partee, State’s Attorney, of Chicago (Inge Fryklund, Patricia Y. Brown, and Lauren Brown, Assistant State’s Attorneys, of counsel), for the People.

JUSTICE CERDA

delivered the opinion of the court:

The defendant, William J. Conley, was charged with two counts of aggravated battery based on permanent disability and great bodily harm. (Ill. Rev. Stat. 1983, ch. 38, par. 12 — 4(a).) He was found guilty after a jury trial of aggravated battery based solely on permanent disability on July 17, 1986. The defendant’s motions for judgment notwithstanding the verdict or a new trial were denied, and the defendant was sentenced to 30 months’ probation including 40 days of periodic imprisonment. On appeal, it is contended that: (1) the State failed to prove beyond a reasonable doubt that the victim incurred a permanent disability and that the defendant intended to inflict a permanent disability; (2) the trial court erred in prohibiting the defense from asking a State identification witness to describe the offender during defendant’s case in chief; (3) the trial court erred in allowing the admission of evidence elicited during State cross-examination that defense witnesses failed to tell police that the offender was another individual; (4) the State’s use during cross-examination and in closing argument of defendant’s pretrial silence deprived the defendant of a fair trial; and (5) the trial court erred in prohibiting defense counsel from arguing to the jury that the victim had a financial motive in securing a verdict and in telling the jury that financial motive was not in issue. For the following reasons, we affirm.

The defendant was charged with aggravated battery in connection with a fight which occurred at a party on September 28, 1985, in unincorporated Orland Township. Approximately 200 high school students attended the party and paid admission to drink unlimited beer. One of those students, Sean O’Connell, attended the party with several friends. At some point during the party, Sean’s group was approached by a group of 20 boys who apparently thought that someone in Sean’s group had said something derogatory. Sean’s group denied making a statement and said they did not want any trouble. Shortly thereafter, Sean and his friends decided to leave and began walking toward their car which was parked a half block south of the party.

A group of people were walking toward the party from across the street when someone from that group shouted “There’s those guys from the party.” Someone emerged from that group and approached Sean, who had been walking with his friend Marty Carroll *23810 to 15 steps behind two other friends, Glen Mazurowski and Dan Scurio. That individual demanded that Marty give him a can of beer from his six-pack. Marty refused, and the individual struck Sean in the face with a wine bottle, causing Sean to fall to the ground. The offender attempted to hit Marty, but missed as Marty was able to duck. Sean sustained broken upper and lower jaws and four broken bones in the area between the bridge of his nose and the lower left cheek. Sean lost one tooth and had root canal surgery to reposition 10 teeth that had been damaged. Expert testimony revealed that Sean has a permanent condition called mucosal mouth and permanent partial numbness in one lip. The expert also testified that the life expectancy of the damaged teeth might be diminished by a third or a half.

At trial, the State presented Officer Houlihan, Doctor Arnold S. Morof, and five occurrence witnesses. Of the five occurrence witnesses, only Marty Carroll identified Conley as the offender. The only other witness connecting Conley to the crime was another student, Demetrius Kereakas, who testified that he saw Conley throw a bottle at Dan Scurio’s car as the four boys left after the incident. The defense recalled State witness Marty Carroll and presented seven witnesses in addition to the defendant. Four of the defense witnesses testified that the defendant was not the offender, but rather that Sean was hit by a Robert Frazer, who is known in school as “Crazy Bob” or “Terminator.” The party was held at a residence surrounded by open fields. There were no streetlights and most of the witnesses had been drinking before the incident.

At the end of the trial, the jury was furnished with four verdict forms for the two counts of aggravated battery. The jury returned a guilty verdict for aggravated battery based on permanent disability, failing to sign the remaining verdict forms. The State’s Attorney advised the trial judge that the jury had returned only one verdict but that he had no objections. The trial court accepted the verdict and discharged the jury. It must be noted here that when a verdict on less than all the counts is accepted by the trial court and the jury is discharged, the jury’s silence as to other counts is treated as an acquittal on those counts for purposes of double jeopardy. (People v. Thurman (1983), 120 Ill. App. 3d 975, 979, 458 N.E.2d 1038, aff’d in part and rev’d in part on other grounds (1984), 104 Ill. 2d 326, 472 N.E.2d 414; People v. Rollins (1982), 108 Ill. App. 3d 480, 485, 438 N.E.2d 1322.) Therefore, had this court found it necessary to reverse and remand for a new trial, the defendant could not have been retried for aggravated battery based on great bodily harm.

*239 The defendant initially contends on appeal that the State failed to prove beyond a reasonable doubt that Sean O’Connell incurred a permanent disability. Section 12 — 4(a) of the Criminal Code of 1961 provides that: “[a] person who, in committing a battery, intentionally or knowingly causes great bodily harm, or permanent disability or disfigurement commits aggravated battery.” (Ill. Rev. Stat. 1983, ch. 38, par. 12 — 4(a).) The defendant contends there must be some disabling effect for an aggravated battery conviction based on permanent disability. The defendant does not dispute that Sean lost a tooth or that surgery was required to repair damaged teeth. The defendant also does not dispute that Sean will have permanent partial numbness in one lip or suffer from a condition called mucosal mouth. The defendant maintains, however, that there is no evidence as to how these injuries are disabling because there was no testimony of any tasks that can no longer be performed as a result of these injuries.

The parties cite no Illinois decisions, nor have we found any, defining permanent disability in the context of aggravated, battery. The State relies on People v. Post (1982), 109 Ill. App. 3d 482, 440 N.E.2d 631, and People v. Hicks (1984), 101 Ill. 2d 366, 462 N.E.2d 473, for the proposition that loss of function is not required for a finding of permanent disability. In Post, the victim had been stabbed once in the back and three times in the leg, but incurred only permanent scarring. The court affirmed the defendant’s conviction for aggravated battery based on permanent disability. However, Post is not dispositive of the issue as the defendant was also convicted of aggravated battery based on great bodily harm and disfigurement, and the defendant never raised the issue of sufficiency of the evidence regarding permanent disability. In Hicks, a young girl received severe burns on her chest caused by boiling water. Hicks is also not dispositive of the issue as the defendant was convicted of heinous battery. Thus, as the defendant points out in his reply brief, this appears to be a question of first impression.

The function of the courts in construing statutes is to ascertain and give effect to the intent of the legislature. (People v. Steppan (1985), 105 Ill. 2d 310, 316, 473 N.E.2d 1300.) The starting point for this task is the language itself (People v. Boykin (1983), 94 Ill. 2d 138, 141, 455 N.E.2d 1174), and the language should be given its plain and ordinary meaning. (Steppan, 105 Ill. 2d at 317; People v. Pettit (1984), 101 Ill. 2d 309, 313, 461 N.E.2d 991.) The defendant urges the court to adopt the definition found in Webster’s Third New International Dictionary which defines disability as an “inability to do *240something.” The State refers to additional language from the same source that a disability is a “physical or mental illness, injury, or condition that incapacitates in any way.” (Webster’s Third New International Dictionary 642 (1986).) There is some support for defendant’s proposed definition in an old Illinois decision. In Dahlberg v. People (1907), 225 Ill. 485, 80 N.E. 310, a woman was convicted of assault with intent to commit mayhem (aggravated battery incorporates the earlier offense of mayhem) after she threw red pepper at someone’s eyes and missed, hitting an innocent bystander in the eyes instead. Her conviction was reversed because the crime of attempt requires that the offender employ adequate means to accomplish the attempted result, and the evidence revealed that blindness could not have resulted had she succeeded. (Dahlberg, 225 Ill. at 490.) Thus, by necessary implication, anything short of blindness would not have supported a conviction for mayhem.

In arriving at a definition, however, it is also proper to consider the statute’s purpose and the evils sought to be remedied. (Steppan, 105 Ill. 2d at 316.) The Committee Comment explains that section 12 — 4(a) incorporates the old offense of mayhem. (Ill. Ann. Stat., ch. 38, par. 12 — 4(a), Committee Comment at 465 (Smith-Hurd 1979).) At common law the offense of mayhem required the dismemberment or disablement of some bodily part. Initially, the law sought to protect the King’s right to the military services of his subjects. However, modern criminal codes have expanded their protection against a wider range of injuries. As one court explained:

“What, then, originated as the narrow common law offense of mayhem is generally today a statutory offense of considerably larger dimensions. The transition has been accompanied, if not induced, by a shift in emphasis from the military and combative effects of the injury to' the preservation of the human body in normal functioning. The statutory counterparts of non-statutory mayhem doubtless include all that the common law proscribed. But what is important now is not the victim’s capacity for attack or defense, but the integrity of his person.” (Emphasis added.) (United States v. Cook (D.C. Cir. 1972), 462 F.2d 301, 303.)

Under this view, it seems apparent that for an injury to be deemed disabling, all that must be shown is that the victim is no longer whole such that the injured bodily portion or part no longer serves the body in the same manner as it did before the injury. Applying this standard to the case at hand, the injuries Sean O’Connell suffered are sufficient to constitute a permanent disability. Sean will en*241dure permanent partial numbness in one lip and mucosal mouth.1 He lost one tooth and there is also a chance he may lose some teeth before attaining the age of seventy.

The defendant further argues that the State failed to prove beyond a reasonable doubt that he intended to inflict any permanent disability. The thrust of defendant’s argument is that under section 12 — 4(a), a person must intend to bring about the particular harm defined in the statute. The defendant asserts that while it may be inferred from his conduct that he intended to cause harm, it does not follow that he intended to cause permanent disability. The State contends it is not necessary that the defendant intended to bring about the particular injuries that resulted. The State maintains it met its burden by showing that the defendant intentionally struck Sean.

The law on this question is unclear. The defendant relies upon People v. Crosser (1983), 117 Ill. App. 3d 24, 27, 452 N.E.2d 857, and Bay State Insurance Co. v. Wilson (1982), 108 Ill. App. 3d 1096, 440 N.E.2d 131, which both hold that aggravated battery is a specific intent crime. The State, however, relies upon People v. Allen (1969), 117 Ill. App. 2d 20, 254 N.E.2d 103. In Allen, the court wrote that in committing the offense of aggravated battery, “the only mental state required is that the accused knowingly and intentionally cause the social harm defined in the statute, no premeditation or malice being necessary.” (Allen, 117 Ill. App. 2d at 27-28.) The court then went on to state that it is not necessary that the defendant intended to cause the particular injury which resulted. (Allen, 117 Ill. App. 2d at 28.) Resolution of this issue is made difficult as there exist inconsistent decisions within the first district. Allen is cited with approval by the third division in People v. Perry (1974), 19 Ill. App. 3d 254, 259, 311 N.E.2d 341; however, specific intent analysis is applied in another decision by the third division in People v. Farrell (1980), 89 Ill. App. 3d 262, 264-65, 411 N.E.2d 927. The fifth division has also produced inconsistent decisions. In People v. Gomez (1986), 141 Ill. App. 3d 935, 939, 491 N.E.2d 68, it was stated that aggravated battery is a specific intent crime. However, just the opposite was written in People v. Gvojic (1987), 160 Ill. App. 3d 1065, 1069, 513 N.E.2d 1083.

For proper resolution of this issue, it is best to return to the statutory language. Section 12 — 4(a) employs the terms “inten*242tionally or knowingly” to describe the required mental state. The relevant statutes state:

“4 — 4. Intent. A person intends, or acts intentionally or with intent, to accomplish a result or engage in conduct described by the statute defining the offense, when his conscious objective or purpose is to accomplish that result or engage in that conduct.” (Ill. Rev. Stat. 1987, ch. 38, par. 4 — 4.)
“4 — 5. Knowledge. A person knows or acts knowingly or with knowledge of:
(b) The result of his conduct, described by the statute defining the offense, when he is consciously aware that such result is practically certain to be caused by his conduct.” (Ill. Rev. Stat. 1987, ch. 38, par. 4 — 5.)

Section 12 — 4(a) defines aggravated battery as the commission of a battery where the offender intentionally or knowingly causes great bodily harm, or permanent disability or disfigurement. Because the offense is defined in terms of result, the State has the burden of proving beyond a reasonable doubt that the defendant either had a “conscious objective” to achieve the harm defined, or that the defendant was “consciously aware” that the harm defined was “practically certain to be caused by his conduct.” (See People v. Herr (1980), 87 Ill. App. 3d 819, 821, 409 N.E.2d 442.) This is the identical construction found in People v. Farrell (1980), 89 Ill. App. 3d 262, 264-65, 411 N.E.2d 927, which we conclude is the correct statement of the law.

Although the State must establish the specific intent to bring about great bodily harm, or permanent disability or disfigurement under section 12 — 4(a), problems of proof are alleviated to the extent that the ordinary presumption that one intends the natural and probable consequences of his actions shifts the burden of production, though not persuasion, to the defendant. (Farrell, 89 Ill. App. 3d at 265.) If the defendant presents evidence contrary to the presumption, then the presumption ceases to have effect, and the trier of fact considers all the evidence and the natural inferences drawn therefrom. (Farrell, 89 Ill. App. 3d at 265.) Intent can be inferred from the surrounding circumstances, the offender’s words, the weapon used, and the force of the blow. (See, e.g., Macklin v. Commonwealth Life & Accident Co. (1970), 121 Ill. App. 2d 119, 126-27, 257 N.E.2d 256.) As the defendant’s theory of the case was mistaken identity, there was no evidence introduced negating the presumption of intent. However, even if Conley had denied any intention to inflict permanent disability, the surrounding circumstances, the use of a bot-*243tie, the absence of warning and the force of the blow are facts from which the jury could reasonably infer the intent to cause permanent disability. Therefore, we find the evidence sufficient to support a finding of intent to cause permanent disability beyond a reasonable doubt.

The defendant next contends that the trial court improperly restricted the scope of his examination of State witness Marty Carroll during his case in chief. During the evening following the close of the prosecution’s case, Conley remembered having a conversation with Carroll at another party where Carroll allegedly made a statement inconsistent with his testimony. Advised of this development, the trial judge permitted the defendant to recall Carroll so that a foundation could be established for later impeachment when Conley testified. After a foundation had been laid, the trial court prohibited the defendant from pursuing an additional line of questioning when the State objected to questions asking Carroll to describe the offender.

The defendant’s argument assumes that People v. Aughinbaugh (1967), 36 Ill. 2d 320, 223 N.E.2d 117, and People v. Morris (1964), 30 Ill. 2d 406, 197 N.E.2d 433, are controlling. These decisions hold that where identification is a principal issue at trial, the defendant is entitled to wide latitude in conducting cross-examination of identification witnesses. These decisions, however, are limited to cross-examination during the State’s case in chief. It is well established in Illinois that the decision to recall a witness for further cross-examination after the close of the adversary’s case is within the sound discretion of the trial court. (People v. Smith (1986), 149 Ill. App. 3d 145, 152, 500 N.E.2d 605.) The trial court’s decision on this matter will not be reversed absent a clear abuse of discretion. (Smith, 149 Ill. App. 3d at 152.) In People v. Dorsey (1982), 109 Ill. App. 3d 218, 440 N.E.2d 394, the defendant argued that his right to due process was violated when the trial court denied his request to recall three witnesses for additional cross-examination in his case in chief. The court held that there was no abuse of discretion where the defendant had an opportunity to cross-examine the witnesses, the questions the defendant wished to ask could have been presented at defendant’s previous cross-examination during the State’s case, and the evidence the defendant hoped to introduce had little probative value. (Dorsey, 109 Ill. App. 3d at 229. See also People v. Lewis (1980), 89 Ill. App. 3d 840, 845, 412 N.E.2d 565.) Here, the defendant had an opportunity to cross-examine Marty Carroll during the State’s case and his questions regarding the offender’s description *244should have been presented at that time. Therefore, there was no abuse of discretion in prohibiting the defendant from pursuing this line of questioning.

Next, the defendant argues that the trial court erred in permitting improper impeachment of four defense witnesses. Matt Tanzer, Kevin McGinley, and Joseph Longhini testified they were present when Sean was hit and that Bob Frazer, and not the defendant, was the culprit. Scott Bucich testified that he observed the defendant in a different altercation not involving Sean. On cross-examination, the witnesses admitted they failed to volunteer this information to the police which may have exonerated the defendant. The defendant maintains that the trial court erroneously overruled his objections for lack of foundation because there was no showing that the police or other authority ever questioned these witnesses. The defendant relies on People v. Fabian (1976), 42 Ill. App. 3d 934, 356 N.E.2d 982, construing that decision to hold that a witness’ failure to volunteer knowledge of a murderer’s identity to police during earlier conversations was not impeaching because the police never inquired as to his ability to identify the offender. Defendant’s reliance on Fabian is misplaced, however, as the issue in Fabian was the weight, and not the admissibility, of the evidence in determining whether the defendant was proved guilty beyond a reasonable doubt. Fabian, 42 Ill. App. 3d at 938.

The rule for impeachment by omission is that it is permissible to use prior silence to discredit a witness’ testimony if: (1) it is shown that the witness had an opportunity to make a statement, and (2) under the circumstances, a person would normally have made the statement. (People v. McMath (1968), 104 Ill. App. 2d 302, 315, 244 N.E.2d 330, affd (1970), 45 Ill. 2d 33, 256 N.E.2d 835, cert. denied (1970), 400 U.S. 846, 27 L. Ed. 83, 91 S. Ct. 92.) At issue here is whether the State properly established the required evidentiary foundation for this impeachment. In People v. Taylor (1986), 141 Ill. App. 3d 839, 491 N.E.2d 3, two defense witnesses testified that the victim of a shooting had been shot by his own brother, and not by the defendant, when the brother fired a gun into a crowd during a street altercation. The witnesses admitted on cross-examination that they failed to go to the police with this information which may have exonerated the defendant. The court held this impeachment was proper because the witnesses, who were friends of the defendant, knew of the defendant’s arrest eight months before trial. (Taylor, 141 Ill. App. 3d at 845-46.) And in People v. Martinez (1979), 76 Ill. App. 3d 280, 284-85, 395 N.E.2d 86, this court ruled that a prosecu*245tor’s questions regarding a witness' failure to go to the police were permissible where the witness who was a friend of the accused waited eight months to tell his story supporting the defendant’s claim of self-defense. On the other hand, in People v. Watson (1981), 94 Ill. App. 3d 550, 557-58, 418 N.E.2d 1015, similar impeachment of an alibi witness was held improper for lack of foundation where the witness did not learn of the defendant’s alleged crime until five months after his arrest, and the State did not attempt to interview the witness despite receiving notice of defendant’s alibi defense. Read together, these decisions indicate that where a witness is a friend of the accused, and has had knowledge of the friend’s arrest before trial, evidence of the witness’ failure to give exculpatory information to the authorities is admissible to impeach an exculpatory story offered for the first time at trial. However, where the witness has not had sufficient notice, there must be evidence of other circumstances under which a reasonable person would have given exculpatory information to the authorities. For the case before us, resolution of this issue requires an examination of the pertinent testimony of each witness.

Matt Tanzer testified that he was a close friend of the defendant. Tanzer also testified that he had discussed the case with the defendant and other friends before trial. Although there was no testimony elicited on cross-examination as to how much time had elapsed before trial during which Tanzer knew of the defendant’s arrest, we do not believe this less than perfect examination to be reversible error. Thus, a sufficient foundation was established for impeaching Tanzer.

Joseph Longhini testified he had only known the defendant for less than a year and was not a close friend. However, Longhini also testified that he discussed the case with the defendant before trial and told him he would be available if the defendant needed him. Thus, a sufficient foundation was established.

Kevin McGinley testified that he was a close friend of the defendant. However, in addition to the absence of testimony indicating when he first learned of the defendant’s arrest, there was also no testimony that he had discussed the case with anyone. Moreover, no investigators for the police or the State ever questioned McGinley as to what he knew. Therefore, use of his prior silence was improper for lack of foundation.

Scott Bucich testified that he had known the defendant for a number of years. Bucich also testified that he discussed the case with the defendant before trial when the defendant called him to learn what Bucich knew of the incident. However, Bucich was a witness *246only to the defendant’s altercation with another individual named John O’Brien. Under these circumstances, a friend of the accused would not normally go to the police to volunteer information pertaining to a separate fight. Thus, the use of Bucich’s prior inaction was improper.

• 14, 15 Nonetheless, this court finds these errors to be harmless. Error is harmless where a reviewing court can safely conclude, after consideration of the totality of the evidence, that a trial without the error would not produce a different result. (People v. Warmack (1980), 83 Ill. 2d 112, 128-29, 413 N.E.2d 1254.) To say that the failure of McGinley and Bucich to volunteer information to the police affected the outcome of the case is speculative, especially in view of the fact that Tanzer and Longhini were properly impeached.

The fourth issue raised by the defendant is whether the State improperly used the defendant’s pretrial silence to impeach his mistaken identity theory in violation of the due process clause of the fourteenth amendment and Illinois evidentiary law. The State sought to discredit the mistaken identity theory as a recent fabrication since the defendant failed to inform the police about Bob Frazer, who the defendant claimed at trial to be the true offender. Examination of this issue must be in two parts as reference was made at trial to two separate occasions where the defendant did not inform the police of the identity of the individual claimed to be the true offender. We first address the State’s use of the defendant’s pre-arrest silence.

On October 7, Officer Houlihan visited Andrew High School and obtained permission to interview the defendant. Houlihan read the Miranda warnings to the defendant before proceeding with the interview. There was conflicting testimony as to what exactly the defendant had said during his interrogation at Andrew High School. Investigator Houlihan testified that Conley admitted to striking an individual he didn’t know with his fist and that the individual fell to the ground. The defendant, however, testified that he had told Houlihan that he fought an individual by the name of John O’Brien and that he gave a description of O’Brien to Houlihan. The defendant further testified that Houlihan stated he had a report of only one fight involving Sean O’Connell and asked him why his fight had not been reported. Conley responded that he didn’t know but that it probably was not reported because no one was hurt. Nevertheless, it is clear that Conley never mentioned the name of Bob Frazer to Houlihan when questioned about his activities at the party and that this fact was elicited during his cross-examination. The record reveals the following colloquy:

*247“Q. And so you saw Marty Carroll at the second Sandora party that you made?
A. Yes, I did.
Q. And he was pointing a finger at you?
A. He sure did.
Q. He said, ‘That’s the guy who hit Shawn with the bottle.’
A. Yes.
Q. You said to him, ‘It wasn’t me. I know who did it.’ Didn’t you say that?
A. Yes, I did.
Q. And you told Officer Houlihan who that was, didn’t you?
A. No, I did not tell him.
Q. I have nothing further.”

The defendant did not object to the State’s question because he felt that his silence could be adequately explained. On redirect, the defendant testified that he had not learned of Bob Frazer until two to three weeks after his interview with Houlihan. The defendant now argues that the State’s use of this silence was improper under Doyle v. Ohio (1976), 426 U.S. 610, 49 L. Ed. 2d 91, 96 S. Ct. 2240. In Doyle v. Ohio, the United States Supreme Court held that use of a defendant’s post-arrest silence to impeach his exculpatory testimony offered for the first time at trial is a deprivation of due process of law. (Doyle v. Ohio (1976), 426 U.S. 610, 49 L. Ed. 2d 91, 96 S. Ct. 2240.) The court reasoned that silence following recitation of the Miranda warnings may be nothing more than the arrestee’s exercise of those rights and, therefore, post-arrest silence is always “insolubly ambiguous.” (Doyle, 426 U.S. at 617, 49 L. Ed. 2d at 97, 96 S. Ct. at 2244.) The Court further reasoned that implicit in the Miranda warnings is the promise that silence will carry no penalty should the accused invoke that right. (Doyle, 426 U.S. at 618, 49 L. Ed. 2d at 98, 96 S. Ct. at 2245.) Therefore, the Court concluded that it would be fundamentally unfair to permit use of such silence against the accused after inducing him to remain silent. (Doyle, 426 U.S. at 618, 49 L. Ed. 2d at 98, 96 S. Ct. at 2245.) Because Conley was successfully rehabilitated on redirect (cf. United States v. Wilkins (7th Cir. 1981), 659 F.2d 769, 776), we conclude that if the State’s use of Conley’s pre-arrest silence violated Doyle, such error was harmless beyond a reasonable doubt. Chapman v. California (1967), 386 U.S. 18, 17 L. Ed. 2d 705, 87 S. Ct. 824.

• 17 Following redirect, however, the State again sought to impeach the defendant’s mistaken identity theory on recross, using the defendant’s post-arrest silence. On October 16, nine days after his in*248terview with Houlihan, the defendant participated in a lineup. At the conclusion of the lineup, the defendant was placed under arrest at which time no statements were made. The record does not reveal whether the defendant was advised of his Miranda rights, but it appears that Conley was not so advised immediately upon his arrest. Following the defendant’s rehabilitation on redirect, the State countered on recross with a reference to the defendant’s silence following his arrest. The record reveals the following colloquy:

“Q. You saw Houlihan on October the 16th, didn’t you?
A. I believe that was the date for the line-up. I’m not sure.
Q. And after the lineup, that’s when Officer Houlihan told you you’re under arrest, isn’t that right?
A. I believe so.
Q. And at that time you didn’t tell officer — .
DEFENSE COUNSEL: I’m objecting now.
THE COURT: Sustained.
PROSECUTOR: I have nothing further, Judge.”

The State contends, first, that its reference to Conley’s post-arrest silence did not deprive him of due process because he had not been advised of his Miranda rights upon arrest. The State relies upon Fletcher v. Weir (1982), 455 U.S. 603, 71 L. Ed. 2d 490, 102 S. Ct. 1309, which held that cross-examination as to post-arrest silence is permissible where such silence has not been induced by the governmental assurances embodied in the Miranda warnings. (Fletcher, 455 U.S. at 607, 71 L. Ed. 2d at 494, 102 S. Ct. at 1312.) The State also contends there was no “use” of silence because the State was prevented from completing the question when the trial court sustained the defendant’s objection, and the jury was instructed to disregard questions to which objections were sustained. (See Greer v. Miller (1987), 483 U.S. 756, 97 L. Ed. 2d 618, 107 S. Ct. 3102.) The defendant argues, however, that Fletcher is inapplicable because he had been advised of his Miranda rights nine days before his arrest at the October 7 interview. The defendant further contends that Greer v. Miller is inapposite because, unlike Greer, the trial court failed to immediately admonish the jury after sustaining his objection, and the prosecutor defied the trial court by mentioning the defendant’s silence in rebuttal argument. On the other hand, the defendant contends that the State’s inquiry into his post-arrest silence is in violation of Illinois evidentiary law even if he has not been deprived of due process. (See People v. McMullin (1985), 138 Ill. App. 3d 872, 486 N.E.2d 412.) Because Doyle establishes only a minimum constitutional threshold for fairness under the due process clause, each juris*249diction is free to define for itself when silence is more probative than prejudicial under its rules of evidence. (See Jenkins v. Anderson (1980), 447 U.S. 231, 240, 65 L. Ed. 2d 86, 96, 100 S. Ct. 2124, 2130.) The defendant’s silence was not admitted into evidence, however, as the trial court sustained the defendant’s objection. Thus, the defendant’s argument on this point is without merit.

We decline to hold that the Miranda warnings Conley received at the October 7 interrogation induced him to remain silent upon his arrest nine days later. Initially, we note that had Conley made incriminating statements during a second interrogation following arrest without being advised of his Miranda rights, Conley would be taking a position contrary to the one he advocates here— that is, Conley would be arguing that the Miranda warnings given nine days earlier are insufficient. (See People v. Rosario (1972), 4 Ill. App. 3d 642, 645-46, 281 N.E.2d 714.) Moreover, Conley agreed to talk with Officer Houlihan on October 7. If the Miranda warnings did not induce him to remain silent on October 7, then we cannot say those same warnings induced him to remain silent nine days later. Therefore, on the facts presented here, we do not believe Doyle applies.

Nonetheless, we further find that there was no “use” of the defendant’s post-arrest silence. In Greer v. Miller, the Supreme Court stated it was significant that in each of the cases in which it applied Doyle, the trial court had “permitted specific inquiry or argument respecting the defendant’s post-Miranda silence.” (Greer v. Miller, 483 U.S. at 764, 97 L. Ed. 2d at 629, 107 S. Ct. at 3108.) The Court concluded there was no Doyle violation because the trial court sustained the defendant’s objection, no further comments were made during the remainder of the trial, and the trial court instructed the jury to disregard questions to which objections were sustained. (Greer, 483 U.S. at 764, 97 L. Ed. 2d at 629, 107 S. Ct. at 3108.) Here, the trial court did not permit specific inquiry or argument. As in Greer, Conley’s objection was sustained. The jury was also instructed to disregard questions to which objections were sustained, and unlike Greer, the question posited to Conley was never completed. As to the prosecutor’s comment during rebuttal argument, the trial court sustained Conley’s objection and, thus, argument respecting Conley’s silence was not permitted.

The defendant distinguishes Greer on the ground that the trial court here did not admonish the jury after sustaining his objection. A careful reading of Greer reveals, however, that the admonishment referred to by the defendant was only a direction to “ignore [the] ques*250tion for the time being.” (Greer, 483 U.S. at 759, 97 L. Ed. 2d at 626, 107 S. Ct. at 3105.) No specific instruction was given. Further, as in Greer, Conley failed to request the trial court to give a specific instruction. (Greer, 483 U.S. at 764 n.5, 97 L. Ed. 2d at 629 n.5, 107 S. Ct. at 3108 n.5.) Therefore, we hold there was no specific inquiry or argument respecting Conley’s silence.

The defendant’s fifth and final contention is that he was improperly precluded from arguing to the jury that Sean’s family had a financial motive in securing a verdict due to the extensive dental work that had been performed. The defendant was attempting to discredit the credibility of State witness Demetrius Kereakas, who had testified that he identified Conley from a picture in an Andrew High School yearbook. The testimony revealed that Mr. O’Connell had visited Demetrius at Richards High School where Mr. O’Connell had given Demetrius the yearbook. The defendant argues that Kereakas’ credibility is suspect because no one besides Mr. O’Connell was present when Kereakas made the identification, and Mr. O’Connell may have pressured Kereakas into picking Conley.

Closing argument must be based on the evidence or on the reasonable inferences drawn therefrom. (People v. Bullock (1987), 154 Ill. App. 3d 266, 273, 507 N.E.2d 44.) The defendant correctly states that financial bias is a legitimate method of impeaching the credibility of a witness. (People v. Thompson (1979), 75 Ill. App. 3d 901, 903, 394 N.E.2d 422, 425.) Here, however, the individual with the alleged financial bias is Mr. O’Connell, who did not testify. Demetrius Kereakas has no financial interest, and Mr. O’Connell’s bias cannot be transferred to Kereakas. There was no evidence that Mr. O’Connell pressured Kereakas or engaged in suggestive conduct which could have led Kereakas to Conley’s photograph. Therefore, the trial court properly sustained the State’s objection to the defendant’s remarks.

The judgment of the circuit court is affirmed.

Judgment affirmed.

FREEMAN, P.J., and WHITE, J., concur.

4.5.4 Notes and Questions (People v. Conley) 4.5.4 Notes and Questions (People v. Conley)

Notes & Questions 

  1. The operative statute in Conley required a mens rea of "intentionally or knowingly" causing "great bodily harm." (see paragraph 10). Given such statutory language, would it have been an easier/faster decision for the court if it had simply focused on the word "knowingly" instead? See the definitions in paragraphs 19-21 as you consider this question. Next, consider the facts of this case under the MPC: Would Conley be guilty of "purposefully" or "knowingly" causing great bodily harm? 

  2. Reconsider Blurton in light of this case. Would it be reasonable for a jury to infer that Blurton intended/knew the natural and probable consequences of his actions? In other words, if we applied the law from Conley to Blurton, what result? (Note: when we use the word "law" in this context, we mean the holding of the case. "Law" can mean statute, opinion, regulation, or any other legal authority that is relevant.)

  3. Essentially, Conley held that it is reasonable to infer that all the natural and probable consequences of an action were "intended" by the actor. Such an inference may not seem extreme when one swings a bottle at close range at someone's face. But consider the potential limitlessness of the court's logic. Would it make sense to apply this rule to less obvious instances as well? Consider someone cutting off a car in traffic, resulting in an accident. We could say the injuries from the accident were natural and probable consequences of bad driving--but did the bad driver intend the result? What would be some reasonable factual scenarios where it would make sense to apply the Conley rule? 

  4. Conley also introduces us to the concept of transferred intent, which rests on the theory that "the state of mind which one has when about to commit a crime upon one person is considered by the law to exist and to be equally applicable although the intended act affects another person." Wilfred J. Ritz, Felony Murder, Transferred Intent, And The Palsgraf Doctrine In The Criminal Law, 16 Wash. & Lee L. Rev. 169 (1959)(quoting from 1 Burdick, Law of Crime 149 (1946)). 

  5. The treatment of transferred intent in the Model Penal Code can be found in section 2.03(2), reproduced below. Identify the phrases that pay for transferred intent. 

    MPC SECTION 2.03(2)

    When purposely or knowingly causing a particular result is an element of an offense, the element is not established if the actual result is not within the purpose or the contemplation of the actor unless:

    (a)   the actual result differs from that designed or contemplated, as the case may be, only in the respect that a different person or different property is injured or affected or that the injury or harm designed or contemplated would have been more serious or more extensive than that caused; or

    (b)   the actual result involves the same kind of injury or harm as that designed or contemplated and is not too remote or accidental in its occurrence to have a [just] bearing on the actor’s liability or on the gravity of his offense.

  6. Consider the following problems--what result under Conley? Under the MPC? 

    (i)  A fires a gun at B, intending to kill B. A misses B, killing C by accident. Did A kill C purposely, knowingly, recklessly or negligently?

    (ii) A fires a gun at B, intending to kill B. It ricochets, killing both B and C. With what state of mind did A kill C?

     

4.5.5 State v. Nations 4.5.5 State v. Nations

STATE of Missouri, Respondent, v. Sandra J. NATIONS, Appellant.

No. 45349.

Missouri Court of Appeals, Eastern District, Division One.

Aug. 28, 1984.

*283Harvey I. Feldman, Clayton, for appellant.

George R. Westfall, Pros. Atty. by James E. Baker, Asst. Pros. Atty., Clayton, for respondent.

SATZ, Judge.

Defendant, Sandra Nations, owns and operates the Main Street Disco, in which police officers found a scantily clad sixteen year old girl “dancing” for “tips”. Consequently, defendant was charged with endangering the welfare of a child “less than seventeen years old,” § 568.050 RSMo 1978.1 Defendant was convicted and fined $1,000.00. Defendant appeals. We reverse.

Defendant contends the state failed to make a submissible case. Defendant failed to preserve this issue for review on appeal.2 We must, however, consider the issue of submissibility under the doctrine of plain error. It is manifest injustice for a trial court to submit a case to the fact finder on evidence insufficient to make a submissible case. E.g., State v. Russell, 581 S.W.2d 61, 63 (Mo.App.1979).

Specifically, defendant argues the state failed to show she knew the child was under seventeen and, therefore, failed to show she had the requisite intent to endanger the welfare of a child “less than seventeen years old.” We agree.

The pertinent part of § 568.050 provides:

“1. A person commits the crime of endangering the welfare of a child if:
(2) He knowingly encourages, aids or causes a child less than seventeen years old to engage in any conduct which causes or tends to cause the child to come within the provisions of subdivision (l)(c) .. of section 211.031, RSMo ....”

The reference to “subdivision (l)(c)” is to § 211.031.l(l)(c) RSMo (Supp. 1976), which was in effect when § 568.050 was enacted. This “subdivision” vested in the juvenile court exclusive original jurisdiction of any proceeding in which a child is alleged to be in need of care and treatment because “[t]he behavior, environment or associations of the child are injurious to his welfare or to the welfare of others”.3 *284Thus, § 568.050 requires the state to prove the defendant “knowingly” encouraged a child “less than seventeen years old” to engage in conduct tending to injure the child’s welfare, and “knowing” the child to be less than seventeen is a material element of the crime. See § 562.021.

“Knowingly” is a term of art, whose meaning is limited to the definition given to it by our present Criminal Code. Literally read, the Code defines “knowingly” as actual knowledge — “A person ‘acts knowingly’, or with knowledge, (1) with respect ... to attendant circumstances when he is aware ... that those circumstances exist _” (Emphasis original). § 562.016.3.4 So read, this definition of “knowingly” or “knowledge” excludes those cases in which “the fact [in issue] would have been known had not the person wilfully ‘shut his eyes’ in order to avoid knowing.” Perkins, Criminal Law 942 (2d ed. 1969). The Model Penal Code, the source of our Criminal Code, does not ex-elude these cases from its definition of “knowingly”. Instead, the Model Penal Code proposes that “[w]hen knowledge of the existence of a particular fact is an element of an offense, such knowledge is established if a person is aware of a high probability of its existence_” (Emphasis added). Model Penal Code § 2.02(7) (Proposed Official Draft 1962). This definition sounds more like a restatement of the definition of “recklessly” than “knowingly”.5 The similarity is intentional. The Model Penal Code simply proposes that wilful blindness to a fact “be viewed as one of acting knowingly when what is involved is a matter of existing fact, but not when what is involved is the result of the defendant’s conduct, necessarily a matter of the future at the time of acting.” 6 Thus, as noted, the Model Penal Code proposes that “[w]hen knowledge of the existence of a particular fact is an element of an offense, such knowledge is established if a person is aware of a high probability of its existence *285Model Penal Code § 2.02(7) (Proposed Official Draft 1962).

Our legislature, however, did not enact this proposed definition of “knowingly”. Although the definitions of “knowingly” and “recklessly” in our Criminal Code are almost identical to the primary definitions of these terms as proposed in the Model Penal Code, see Model Penal Code § 2.02(2)(b)-(c) (Proposed Official Draft 1962), the Model Penal Code’s proposed expanded definition of “knowingly”, encompassing wilful blindness of a fact, is absent from our Criminal Code. The sensible, if not compelling, inference is that our legislature rejected the expansion of the definition of “knowingly” to include wilful blindness of a fact and chose to limit the definition of “knowingly” to actual knowledge of the fact.7 Thus, in the instant case, the state’s burden was to show defendant actually was aware the child was under seventeen, a heavier burden than showing there was a “high probability” that defendant was aware the child was under seventeen. In short, the state’s burden was to prove defendant acted “knowingly”, not just “recklessly”. The state proved, however, that defendant acted “recklessly”, not “knowingly”. This we conclude from our review of the record.

In our review of the record, we do not weigh the evidence; rather, we simply determine whether there was sufficient proof for the fact finder to find the defendant guilty beyond a reasonable doubt. E.g., State v. Turner, 623 S.W.2d 4, 6 (Mo. banc 1981), cert. denied, 456 U.S. 931, 102 S.Ct. 1982, 72 L.Ed.2d 448 (1982). In so doing, we consider only those facts and reasonable inferences favorable to the state. E.g., State v. Franco, 544 S.W.2d 533, 534 (Mo. banc 1976), cert. denied, 431 U.S. 957, 97 S.Ct. 2682, 53 L.Ed.2d 275 (1977).

The record shows that, at the time of the incident, the child was sixteen years old. When the police arrived, the child was “dancing” on stage for “tips” with another female. The police watched her dance for some five to seven minutes before approaching defendant in the service area of the bar. Believing that one of the girls appeared to be “young,” the police questioned defendant about the child’s age. Defendant told them that both girls were of legal age and that she had checked the girls’ identification when she hired them. When the police questioned the child, she initially stated that she was eighteen but later admitted that she was only sixteen. She had no identification.

Aside from the child’s age, these facts were established by the testimony of a police officer. The state also called the child as a witness. Her testimony was no help to the state. She testified the defendant asked her for identification just prior to the police arriving, and she was merely crossing the stage to get her identification when the police took her into custody.8 Nor can the state secure help from the defendant’s testimony. She simply corroborated the child’s testimony; i.e., she asked the child for her identification; the child replied she would “show it to [her] in a minute”; the police then took the child into custody.

These facts simply show defendant was untruthful. Defendant could not have checked the child’s identification, because the child had no identification with her that day, the first day defendant “hired” the child. This does not prove that defendant knew the child was less than seventeen years old. At best, it proves defendant did not know or refused to learn'the child’s age. The latter is the best case for the state. But defendant’s refusal to learn the age of this “young” child who was “dancing” “scantily clad” in her disco bar simply proves that defendant was “aware of a *286high probability” that the child was under seventeen, or, stated otherwise, in the definitional language of our Criminal Code, proves that defendant was conscious of “a substantial and unjustifiable risk” that the child was under seventeen and that defendant’s disregard of the risk was a “gross deviation” from the norm. See § 562.016.-4. This, however, is not “knowledge” under our Criminal Code. It is “recklessness”, nothing more. Having failed to prove defendant knew the child’s age was less than seventeen, the state failed to make a submissible case.9

Judgment reversed.

SIMON, P.J., and KAROHL, J., concur.

4.5.6 Notes and Questions (State v. Nations) 4.5.6 Notes and Questions (State v. Nations)

Notes & Questions 

  1. How reasonable was Nations's ignorance regarding the dancers' age? Should she have been charged with having had "inquiry notice" at least? Do you think the statute reduces the harm that prompted the statute's enactment if only "actual knowledge" is criminalized? 

  2. On the other hand, rates of conviction and subsequent incarceration would certainly rise if a lesser mens rea would suffice for conviction. Is overcarceration a price we are willing to pay considering the harm to minors in this context? Are there other crimes for which we would (or not) be willing to take such risks? How might we balance these divergent policy goals? 

  3. Because individual and subjective biases play into how we estimate characteristics like age, is there always a risk that some minors tend to be more protected than others? For example, social workers, police, and various non-family caregiving adults tend to see black children as less innocent and in need of protection than  white children. With that in mind, what should be the right standard for knowledge within the factual scenario of Nations?  

  4. As you saw in this case, a legislature might adopt MPC language in its entirety or choose to adopt it partially or in piecemeal. Can we assume that the legislature meant to adopt the MPC framework even if it didn't adopt all of the MPC's language, or was this a deliberate choice? In Nations, does the court assume that actual knowledge was a deliberate move or an oversight? Contrast this case with how Morissette (below) addresses the same question regarding statutory construction. 

    One way to clarify this legislative intent, of course, is to pore over the documents (earlier versions of bills, committee notes, transcripts, etc.) leading up to the legislation. When should a court pay attention to legislative intent to aid its statutory construction? 

     

     

4.5.7 State v. Miles 4.5.7 State v. Miles

State v. Miles

South Carolina Court of Appeals

421 S.C. 154, 805 S.E.2d 204

Appellate Case No. 2015-000308; Opinion No. 5511

2017-08-23

Appellate Defender John Harrison Strom, of Columbia, for Appellant.

Attorney General Alan McCrory Wilson, Senior Assistant Deputy Attorney General Megan Harrigan Jameson, Assistant Deputy Attorney General David A. Spencer, all of Columbia; and Solicitor Samuel R. Hubbard, III, of Lexington, for Respondent.

HILL, J.:

Lance L. Miles appeals his conviction for trafficking in illegal drugs in violation of section 44-53-370(e)(3) of the South Carolina Code (Supp. 2016). He argues the trial court erred by: (1) instructing the jury, in reply to a question they posed during deliberation, that the State did not have to prove Miles knew the drugs were oxycodone; (2) denying his directed verdict motion; and (3) admitting three statements he contends were obtained in violation of Miranda v. Arizona , 384 U.S. 436, 86 S.Ct. 1602, 16 L.Ed.2d 694 (1966). We affirm.

I.

While scanning parcels for illegal drugs at the Federal Express office in West Columbia, agents from the Lexington County Sheriff's Office became suspicious of a package. They arranged for a controlled delivery to the listed address, which was within an apartment complex. Surveilling the delivery, they observed the delivery person ring the doorbell and leave the package by the front door. A few moments later, an agent noticed Miles exit a nearby apartment and begin walking around the parking lot. The agent then saw a young female emerge from the delivery address. She looked at the box, got on her phone, quickly hung up and went back inside. Miles then got on his phone while walking towards the box. Miles picked up the box and started back to his apartment. Seeing the agents advancing to intercept him, he tried to ditch the box. The agents apprehended and handcuffed him.

Agent Edmonson immediately questioned Miles about the contents of the box. Miles claimed he did not know what was inside. Edmonson then asked if there were drugs inside the box; Miles responded there probably were, but he did not know what kind. At this point, Edmonson read Miles his Mirandarights and asked Miles again whether there were drugs in the box. Miles again responded the box could contain drugs, but he did not know what kind. Upon obtaining a search warrant and Miles' consent, the agents opened the box and discovered three hundred pills that a chemist later testified contained a total of nine grams of oxycodone. Edmonson next asked Miles to write down everything he knew about the box and the drugs. Edmonson then reread Miles his Miranda rights, and Miles wrote a statement admitting he had been paid one hundred dollars to pick up the box, someone named "Mark" had called him to pick it up, and the "owner" was a "Stacks" from Tennessee.Edmonson then wrote out two questions. First, "Did you know drugs are in the parcel ‘box’?" Miles wrote, "Yes." The second question and answer—related to Miles' admission that he had previously picked up packages for money—were redacted and not presented to the jury.

Miles was indicted for trafficking in illegal drugs, in violation of section 44-53-370(e)(3). He did not testify at his trial and moved unsuccessfully for directed verdict, arguing in part there was insufficient evidence he knew the box contained oxycodone. During the jury charge, the trial court gave the following instruction:

Mr. Miles is charged with trafficking in illegal drugs and in this case we are referring to [o]xycodone. The State must prove beyond a reasonable doubt that the Defendant knowingly delivered, purchased, brought into this state, provided financial assistance or otherwise aided, abetted, attempted or conspired to sell, deliver, purchase, or bring into this state and was knowingly in actual or constructive possession or knowingly attempted to become in actual or constructive pos[session] of the [o]xycodone. Possession may be either ... actual or constructive.

The trial court charged that the State bore the burden of proving the amount of oxycodone was more than four grams. The trial court further instructed that the State had to prove criminal intent, which required a "conscious wrongdoing," and that intent may be inferred from the conduct of the parties and other circumstances. After deliberating for some time, the jury asked the following question: "Does the [S]tate have to prove that the defendant knowingly brought into the state four grams or more of [o]xycodone or just any amount of illegal drugs in order to consider this trafficking?"

The trial court, over Miles' objection, replied to the jury as follows:

[T]he law in South Carolina is the State does not have to prove that the Defendant knew that the drugs in the package were [o]xycodone, just that he knew that the package contained illegal drugs. However, the State does have to prove beyond a reasonable doubt that the illegal drugs that were in the package w[ere] more than four grams of [o]xycodone.

The jury later returned with a verdict of guilty. Because Miles had at least two prior drug convictions, he was sentenced to the mandatory minimum term of twenty-five years, and ordered to pay a $100,000 fine.

II.

Miles' primary argument on appeal is the trial court's supplemental charge misinformed the jury that the State did not need to prove beyond a reasonable doubt that Miles knew the drug he possessed was oxycodone. We review jury instructions to determine whether they, as a whole, adequately communicate the law in light of the issues and evidence presented at trial. State v. Logan , 405 S.C. 83, 90, 747 S.E.2d 444, 448 (2013).

Section 44-53-370(e)(3) provides in part:

Any person who knowingly sells, manufactures, cultivates, delivers, purchases, or brings into this State, or who provides financial assistance or otherwise aids, abets, attempts, or conspires to sell, manufacture, cultivate, deliver, purchase, or bring into this State, or who is knowingly in actual or constructive possession or who knowingly attempts to become in actual or constructive possession of: ... four grams or more of any morphine, opium, salt, isomer, or salt of an isomer thereof, including heroin, as described in Section 44-53-190 or 44-53-210, or four grams or more of any mixture containing any of these substances, is guilty of a felony which is known as "trafficking in illegal drugs"....

(emphases added).

Miles contends the term "knowingly" as used in subsection (e) applies to each element of the trafficking offense, including the specific type of drugs listed in (e)(3). The issue of whether trafficking requires proof that the defendant not only knowingly intended to "sell[ ], manufacture [ ], cultivate[ ] ..." or "posses[ ]" illegal drugs, but also had knowledge of the precise identity of the illegal drug being trafficked, has, surprisingly, never been addressed by our appellate courts.We are mindful that "statutory interpretation begins (and often ends) with the text of the statute in question. Absent an ambiguity, there is nothing for a court to construe, that is, a court should not look beyond the statutory text to discern its meaning." Smith v. Tiffany , 419 S.C. 548, 555–56, 799 S.E.2d 479, 483 (2017) (citations omitted).

Courts grapple often with that tricky adverb "knowingly." In United States v. Jones , 471 F.3d 535, 538 (4th Cir. 2006), the court construed a federal statute that punished "[a] person who knowingly transports an individual who has not attained the age of 18 years in interstate or foreign commerce ... with intent that the individual engage in prostitution, or in any sexual activity for which any person can be charged with a criminal offense." (quoting 18 U.S.C. § 2423(a) (2000 & Supp. 2003) ). Rejecting the argument that the government was required to prove the defendant knew the person transported was a minor, Judge Wilkinson noted:

[C]onstruction of the statute demonstrates that it does not require proof of the defendant's knowledge of the victim's minority. It is clear from the grammatical structure of § 2423(a) that the adverb "knowingly" modifies the verb "transports." Adverbs generally modify verbs, and the thought that they would typically modify the infinite hereafters of statutory sentences would cause grammarians to recoil. We see nothing on the face of this statute to suggest that the modifying force of "knowingly" extends beyond the verb to other components of the offense.

Id. at 539.

The United States Supreme Court has not been so gun-shy about the adverb. They ordinarily read a "statute that introduces the elements of a crime with the word ‘knowingly’ as applying that word to each element." Flores-Figueroa v. Unit ed States , 556 U.S. 646, 652, 129 S.Ct. 1886, 173 L.Ed.2d 853 (2009). They have also found "the word ‘knowingly’ applies not just to the statute's verbs but also to the object of those verbs." McFadden v. United States , ––– U.S. ––––, 135 S.Ct. 2298, 2304, 192 L.Ed.2d 260 (2015).

We suspect the bar for causing grammarians to recoil is low.

But the Court has not gone so far as to hold that a criminal statute that opens with "knowingly" invariably requires each element be proven by that level of intent. It is commonplace that "different elements of the same offense can require different mental states." Staples v. United States , 511 U.S. 600, 609, 114 S.Ct. 1793, 128 L.Ed.2d 608 (1994). Even in Flores-Figueroa , the Court acknowledged that "knowingly" does not always modify every element, particularly where the statutory sentences at issue "involve special contexts or ... background circumstances that call for such a reading." 556 U.S. at 652, 129 S.Ct. 1886. The Court emphasized that "the inquiry into a sentence's meaning is a contextual one." Id. ; see also Avis Rent A Car Sys., Inc. v. Hertz Corp. , 782 F.2d 381, 385 (2d Cir. 1986) ("Fundamental to any task of interpretation is the principle that text must yield to context.") (Friendly, J.).

Our duty is to determine legislative intent, and the text of the statute is often the best evidence of that intent. Hodges v. Rainey , 341 S.C. 79, 85, 533 S.E.2d 578, 581 (2000). Yet the text "must be construed in context and in light of the intended purpose of the statute in a manner which harmonizes with its subject matter and accords with its general purpose." Cabiness v. Town of James Island, 393 S.C. 176, 192, 712 S.E.2d 416, 425 (2011) (citation and internal quotations omitted).

We find that by using "knowingly" in subsection (e), the Legislature did not intend to require the State to prove a defendant knew the specific type of illegal drug he was trafficking. Section 44-53-370 is concerned with criminalizing numerous forms of conduct involving illegal drugs. Thus, subsection (c) decrees "[i]t shall be unlawful for any person knowingly or intentionally to possess a controlled substance," subject to certain exceptions not relevant here. S.C. Code Ann. § 44-53-370(c) (Supp. 2016). Our supreme court has held the language now codified in subsection (c) requires the State to prove beyond a reasonable doubt that the defendant knew he possessed a "controlled substance." State v. Attardo , 263 S.C. 546, 549, 211 S.E.2d 868, 869 (1975). Subsection (d) then sets forth the penalties for possession based on the type of controlled substance. S.C. Code Ann. § 44-53-370(d) (Supp. 2016).

This brings us to trafficking, subsection (e). Tellingly, our supreme court has explained "[i]t is the amount of [the controlled substance], rather than the criminal act, which triggers the trafficking statute, and distinguishes trafficking from distribution and simple possession." State v. Raffaldt, 318 S.C. 110, 117, 456 S.E.2d 390, 394 (1995). While the court in Raffaldt was not confronted with the mental state required for a trafficking conviction, that issue was addressed in State v. Taylor , 323 S.C. 162, 166, 473 S.E.2d 817, 819(Ct. App. 1996). In Taylor , the defendant was charged with trafficking more than ten grams of crank, in violation of section 44-53-375(C) of the South Carolina Code (Supp. 1995), which contains language nearly identical to section 44-53-370(e), including placement of the adverb "knowingly." Taylor argued the language required the trial court to charge the jury that "they could not find [her] guilty of trafficking in crank unless she knew there were ten grams or more." Taylor , 323 S.C. at 167, 473 S.E.2d at 819. Relying on Raffaldt , we disagreed. Id .

Raffaldt and Taylor illuminate the "special context" revealed by viewing section 44-53-370 as a whole. Because section 44-53-370(c) only requires knowledge that the substance is "controlled," and because Raffaldt and Taylor tell us the only difference between the elements of distribution and simple possession and the elements of trafficking is the amount of the controlled substance involved, there is no reason to suspect the Legislature meant to require knowledge of the specific type of controlled substance in trafficking prosecutions. Miles' interpretation depends upon isolating "knowingly" in subsection (e) and extending its modifying reach not only to "possession," but to the specific type of drugs listed. Magnifying individual words of a statute and insisting they be interpreted concretely can lead to strange results. One could, for example, myopically diagram subsection (e)(3) and conclude it criminalizes the possession of more than four grams of table salt, or even the conduct of the delivery person in this case. Further, were we to adopt Miles' version of subsection (e), the State would have to convince the jury beyond a reasonable doubt the defendant not only knew the drugs were oxycodone, but also knew that oxycodone is a "morphine, opium, salt, isomer, or salt of an isomer thereof, including heroin, as described in Section 44-53-190 or 44-53-210, or ... any mixture containing any of these substances." We doubt the Legislature, in passing the drug trafficking laws, meant to create a scenario where a defendant is culpable only if armed with a proficiency in chemistry on par with a pharmacist or Walter White. That is why considering the words in their surrounding environment is essential, especially here where the statute runs to nearly five-thousand words and represents the Legislature's will in the massive field of drug interdiction. Given this background, "[i]f ever we are justified in reading a statute, not narrowly as through a keyhole, but in the broad light of the evils it aimed at and the good it hoped for, it is here." United States ex rel. Marcus v. Hess , 317 U.S. 537, 557, 63 S.Ct. 379, 87 L.Ed. 443(1943) (Jackson, J., dissenting).

Breaking Bad (AMC 2008–13).

Our emphasis on context and structure bears on the threshold decision of whether the statute is ambiguous, and is not meant to dilute the rule of lenity, as we later discuss.
--------

When a statute can be read in its ordinary sense, courts have no right to engineer an extraordinary one. That the Legislature titled the offense defined by subsection (e)(3) as "trafficking in illegal drugs" affirms our conclusion that a defendant need not know the precise identity of the controlled substance to be guilty. See Univ. of S.C. v. Elliott , 248 S.C. 218, 221, 149 S.E.2d 433, 434 (1966) ("[I]t is proper to consider the title or caption of an act in aid of construction to show the intent of the legislature."). This sense becomes inescapable when we consider subsection (e)(3)'s reference to sections 44-53-190 and 44-53-210 of the South Carolina Code (Supp. 2016), which set forth Schedules I and II governing classification of controlled substances. While we can interpret statutes by bringing in rules of grammar, logic, and other tools, we must be careful not to construe common sense out.

Courts in many other states share our conclusion that proving the defendant knew the specific type of drug is not required in trafficking and other controlled substance offenses. See, e.g. , State v. Stefani , 142 Idaho 698, 132 P.3d 455, 461 (Idaho Ct. App. 2005) ; People v. Bolden , 62 Ill.App.3d 1009, 20 Ill.Dec. 79, 379 N.E.2d 912, 915 (1978) ; Com. v. Rodriguez , 415 Mass. 447, 614 N.E.2d 649, 653 (1993) ; State v. Ali , 775 N.W.2d 914, 919 (Minn. Ct. App. 2009) ; State v. Edwards , 257 N.J.Super. 1, 607 A.2d 1312, 1313(App.Div.1992) ; State v. Engen , 164 Or.App. 591, 993 P.2d 161, 170 (1999) ; State v. Sartin , 200 Wis.2d 47, 546 N.W.2d 449, 455 (1996).

We cannot leave this issue without discussing the important canon of statutory construction that penal statutes are to be strictly construed. This rule of lenity applies when a criminal statute is ambiguous, and requires any doubt about a statute's scope be resolved in the defendant's favor. Berry v. State , 381 S.C. 630, 633, 675 S.E.2d 425, 426 (2009). But the rule of lenity is not a device to create ambiguity, nor should a court invoke it before considering the words of the statute in context. State v. Dawkins , 352 S.C. 162, 166–67, 573 S.E.2d 783, 785 (2002) ; State v. Firemen's Ins. Co. of Newark, N.J., 164 S.C. 313, 162 S.E. 334, 338 (1931) ( "The rule that a penal statute must be strictly construed does not prevent the courts from calling to their aid all the other rules of construction and giving each its appropriate scope, and is not violated by giving the words of the statute a reasonable meaning according to the sense in which they were intended, and disregarding ... even the demands of exact grammatical propriety." (citation and internal quotations omitted)); see also United States v. Bass, 404 U.S. 336, 347, 92 S.Ct. 515, 30 L.Ed.2d 488 (1971) (court should rely on lenity only if, "[a]fter ‘seiz[ing] every thing from which aid can be derived,’ " it is "left with an ambiguous statute" (quoting United States v. Fisher , 6 U.S. (2 Cranch) 358, 386, 2 L.Ed. 304 (1805) (Marshall, C.J.))).

One of the foundations of the rule of lenity is the concept of fair notice—the idea that those trying to walk the straight and narrow are entitled to know where the line is drawn between innocent conduct and illegality. McBoyle v. United States , 283 U.S. 25, 27, 51 S.Ct. 340, 75 L.Ed. 816 (1931) ("[I]t is reasonable that a fair warning should be given to the world in language that the common world will understand, of what the law intends to do if a certain line is passed. To make the warning fair, so far as possible the line should be clear."). The line for conduct involving contraband is not merely clear but fluorescent. At least since State v. Freeland, 106 S.C. 220, 91 S.E. 3 (1916), we have required a defendant to know or be willfully ignorant that he was dealing with contraband drugs to satisfy criminal intent. This removes innocent activity, inadvertence or accident from the law's grasp. At any rate, we need not apply the rule of lenity here, as context has convinced us section 44-53-370(e)(3) does not require proof of knowledge of the specific identity of the controlled substance. Carter v. United States, 530 U.S. 255, 269, 120 S.Ct. 2159, 147 L.Ed.2d 203 (2000) (courts are required "to read into a statute only that mens rea which is necessary to separate wrongful conduct from ‘otherwise innocent conduct’ ").

Another foundation of the rule of lenity is the separation of powers. Our Constitution commits the task of defining criminal offenses solely to the Legislative Branch. Bass , 404 U.S. at 347-48, 92 S.Ct. 515 ; United States v. Wiltberger , 18 U.S. (5 Wheat.) 76, 95, 5 L.Ed. 37 (1820). If the Legislature believes our interpretation expands or is otherwise contrary to the scope it intended section 44-53-370(e)(3) and its harsh penalty scheme to have, it can amend the statute.

The trial judge's instructions—including his initial charge that criminal intent consists of "conscious wrongdoing"—conveyed the pertinent legal standards to the jury. He further correctly charged that the State still bore the burden of proving the drug quantity and identity.

III.

Miles next argues he was entitled to a directed verdict because the State presented insufficient evidence that he knowingly trafficked oxycodone. As we have held, the State needed only to prove Miles knew the item was a controlled substance. Because there was evidence Miles possessed the box, the jury was free to infer he knew what was in it. As the assistant solicitor pointed out, the evidence was literally lying at Miles' feet. See State v. Gore , 318 S.C. 157, 163, 456 S.E.2d 419, 422 (Ct. App. 1995) ("Possession gives rise to an inference of the possessor's knowledge of the character of the substance."). Of course, Miles also admitted he knew the box contained drugs. Viewing the evidence in the light most favorable to the State, these circumstances go far beyond mere suspicion. There was ample direct and substantial circumstantial evidence from which Miles' guilt could be fairly and logically deduced. Rule 19, SCRCrimP ; State v. Odems, 395 S.C. 582, 586, 720 S.E.2d 48, 50 (2011).

IV.

Miles contends the series of three statements he gave to law enforcement should have been suppressed because the agents engaged in the "question-first" manipulation of Miranda forbidden by Missouri v. Seibert , 542 U.S. 600, 124 S.Ct. 2601, 159 L.Ed.2d 643 (2004), and State v. Navy , 386 S.C. 294, 688 S.E.2d 838 (2010). He asserts Agent Edmonson's immediate questioning of him upon arrest was a custodial interrogation triggering Miranda . At trial, the State conceded as much and agreed not to present evidence of Miles' first two statements. But, during a later bench conference, Miles agreed to their admissibility, which is unsurprising as this strategy allowed Miles to get his theory of the case—that he didn't know what kind of drugs were in the package—before the jury without having to take the stand. See State v. Bryant , 372 S.C. 305, 642 S.E.2d 582 (2007) (stating an issue conceded at trial cannot be argued on appeal).

The issue of whether admission of Miles' third, written statement violated Seibert and Navy is unpreserved. Miles did not raise these cases or the "question-first" principle to the trial court. See State v. Byers , 392 S.C. 438, 446, 710 S.E.2d 55, 59 (2011) ("For an admissibility error to be preserved, the objection must include a specific ground ‘if the specific ground was not apparent from the context.’ " (quoting Rule 103(a)(1), SCRE )); In re Michael H. , 360 S.C. 540, 546, 602 S.E.2d 729, 732 (2004) ("In order to preserve an issue for appeal, it must be raised to and ruled upon by the trial court. In other words, the trial court must be given an opportunity to resolve the issue before it is presented to the appellate court." (citation omitted)).

Even if the issue was preserved, any error in admitting the redacted written statement was harmless. The statement was cumulative and could not have reasonably contributed to the verdict. It did not contradict Miles' earlier statements that he did not know the type of drugs in the box, and added he was paid one-hundred dollars to retrieve it. See State v. Martucci , 380 S.C. 232, 261, 669 S.E.2d 598, 614 (Ct. App. 2008) ("The admission of improper evidence is harmless where the evidence is merely cumulative to other evidence."). We cannot imagine the vague references to others involved packed any punch with the jury.

V.

The trial court did not err in its supplemental instruction to the jury that the State was only required to prove Miles knowingly trafficked in a controlled substance. There was sufficient evidence to carry the case to the jury, and even if the Miranda issue was preserved, we find no prejudice. Miles' conviction is therefore

AFFIRMED.

GEATHERS and MCDONALD, JJ., concur.

4.5.8 Notes & Questions (State v. Miles) 4.5.8 Notes & Questions (State v. Miles)

1.    In Flores-Figueroa (discussed in Miles), Flores-Figueroa was convicted of illegal entry into the U.S. without inspection, misusing immigration documents, and aggravated identity theft. Aggravated identity theft (§1028A(a)(1) of the U.S. Code) adds a mandatory two-year sentence to anyone who “knowingly transfers, possesses, or uses, without lawful authority, a means of identification of another person.” The facts showed that Mr. Flores-Figueroa presented to his employer counterfeit Social Security and alien registration cards that contained identification numbers that had been assigned to other people. But he testified that, although he knew the documents he presented were counterfeit, he did not know that the numbers on the documents in fact belonged to other people. The U.S. Supreme Court reversed, holding that Flores-Figueroa had to know that the means of identification belonged to another person--that is, it was not enough merely to show that he knew the identification materials were false.

How would Flores-Figueroa be decided under State v. Miles?

2.    A statute reads, “It is unlawful to knowingly fire a machine gun in a densely populated neighborhood and thereby injure a bystander.” What words within this statute are modified by the adverb "knowingly"? In other words, what knowledge on the defendant's part is required by this statute for a successful conviction? Apply the holdings in Miles, Nations, and Flores-Figueroa to interpret this statute. 

3.    Two years ago, alarmed by the noise created when residents drove motorized vehicles through Central Park, the town of Quietville passed this ordinance: “No vehicles permitted in the park.” Shortly thereafter, the police arrested five teenagers, claiming they had broken the law by riding their bicycles through the park. In response to the teens’ successful challenge to their arrest, Quietville amended its law. It now reads: "Out of respect for the quiet of our town, it is illegal to enter the park with a motorized vehicle."

Last month, a group of patriots sought to place a functioning military truck in the park as a war memorial. Police arrested the group as they were driving the truck into Central Park.

You are their defense lawyer. Write a paragraph with no more than 3 sentences stating the relevant rule of law and explaining why application of that rule to these facts means that defendants are not guilty.

4.    Revisiting the facts of Miles: What are some "good facts" for the prosecution? What facts allow us to reasonably infer that Miles knew what he was picking up? By the same token, what are some "good facts" for the defense? In what way could one rebut the prosecution's case regarding what Miles knew?  

5.    A mistake of fact is a defense against a crime requiring knowledge. That is, an honest misunderstanding about the crucial facts can negate the required mens rea. (Note: mistake of fact does not apply to strict liability crimes where knowledge would be irrelevant.) 

Why didn't a mistake of fact defense work for Miles? 

6.    Consider the following alternative scenarios for Miles and assess whether a judge or jury might find a mistake of fact: 

a) Miles received an email from his mother to expect a care package on that very day. That was, in fact, what he believed he was picking up. 

b) Miles believed the package contained Adderall, which he uses on days he needs to attend classes or take exams. 

c) Miles believed the package contained Oxycodone, but testing revealed the contents to be table salt. 

d) Miles believed the package contained Oxycodone, but did not believe it was against the law to receive such a package. (See "mistake of law.") 

4.5.9 Kougl v. Board of Liquor License Commissioners 4.5.9 Kougl v. Board of Liquor License Commissioners

137 A.3d 1062

Steven KOUGL, et al. v. The BOARD OF LIQUOR LICENSE COMMISSIONERS FOR BALTIMORE CITY.

No. 935,

Sept. Term, 2015.

Court of Special Appeals of Maryland.

June 2, 2016.

*316Peter A. Prevas (Prevas and Prevas, on the brief) Baltimore, MD, for appellants.

Shelley Johnson, Annapolis, MD, for appellees.

Panel: WOODWARD, GRAEFF, and ARTHUR, JJ.

WOODWARD, J.

The eagle suffers little birds to sing,
And is not careful what they mean thereby,
Knowing that with the shadow of his wings
He can at pleasure stint their melodyf.]
—Tamora, Titus Andronicus, Act 4, Scene 4 (W. Shakespeare)

In the instant case, we are called upon to decipher the meaning of the word “suffer,” and its analogs, “permit” and “allow.” Specifically, we must decide whether these terms, when used in rules governing the conduct of holders of a liquor license in Baltimore City, require proof of knowledge on the part of such licensees.

Appellant, Steven Kougl, owns Club Harem, a Baltimore tavern and adult entertainment business, and holds a liquor license for that location issued by appellee, the Board of Liquor License Commissioners for Baltimore City (“the Liquor Board”). On July 2, 2014, the Liquor Board charged Kougl with violating three Liquor Board Rules related to solicitation of prostitution, indecent exposure, and violation of public morals, all occurring on April 25, 2013, at Club Harem, when an employee exposed herself to an undercover police officer and then solicited sexual intercourse from the same officer. The Liquor Board found Kougl guilty of all three violations and ordered that his liquor license be suspended for one month. Kougl filed for judicial review in the Circuit Court for Baltimore City, which affirmed the decision of the Liquor Board.

On appeal to this Court, Kougl raises four questions for our review, which we have condensed and rephrased as two ques*317tions:1

1. Did the Liquor Board make sufficient findings of fact in support of the three charged violations of the Liquor Board Rules?
2. Did the Liquor Board err in concluding that Kougl was guilty of the three violations even though there was no evidence that Kougl had knowledge of his employee’s behavior?

For reasons set forth herein, we answer both questions in the affirmative, and thus reverse the judgment of the circuit court and remand the case to that court for entry of a judgment reversing the decision of the Liquor Board.

BACKGROUND

On April 25, 2013, Detective Fletcher Jackson, a Baltimore City police officer assigned to the Special Enforcement Section, entered Club Harem in plain clothes to conduct a prostitution investigation. Jamaica Brickhouse, a woman who worked at Club Harem, approached Det. Jackson at the bar and asked if she could join him. Det. Jackson said “sure,” and bought Brickhouse a drink. After some initial conversation, Brickhouse exposed her breasts to Det. Jackson. Brickhouse invited Det. Jackson to touch her breasts, which he did. Brickhouse then suggested a lap dance or a trip to “the VIP,” where they could “do whatever up there.” Det. Jackson asked how much it would cost him to have sex with Brickhouse; Brickhouse replied that it would cost $170 “for the room,” plus a tip for her services. Det. Jackson said that he could tip $100, and Brickhouse agreed. Brickhouse returned to the *318stage to perform, and Det. Jackson notified other officers. Brickhouse, however, was not issued a criminal summons until December 10, 2013, almost eight months later.2

Fifteen months after the incident, on July 2, 2014, the Liquor Board charged Kougl with violating three Liquor Board Rules related to solicitation of prostitution in violation of Rule 4.17(a), indecent exposure in violation of Rule 4.17(b), and violation of public morals in violation of Rule 4.18. On July 17, 2014, the Liquor Board held a hearing on the charges against Kougl. Det. Jackson was the only witness for the prosecution; Kougl testified in his own defense. The Liquor Board voted 2-1 that Kougl was guilty of the three violations3 and suspended his liquor license for one month.

On July 18, 2014, Kougl filed a Petition for Judicial Review in the circuit court. On March 2, 2015, Kougl filed his Memorandum, in which he argued that (1) the Liquor Board’s decision was not based on substantial evidence, because “[t]here [wa]s nothing in [Det. Jackson’s] testimony to indicate that the [licensee knew, allowed or permitted this type of activity, which is needed to sustain the violations”; (2) its decision was unreasonable and arbitrary, because the Chairman of the Liquor Board stated that a licensee “would be responsible for actions of an employee” regardless of “whether the [licensee knew or did not know or allowed or did not allow or permitted or did not permit” the prohibited activity; and (3) the Liquor Board failed to make specific findings of fact and conclusions of law as required. On April 15, 2015, the Liquor Board filed its Response to Kougl’s Memorandum, in which it contended that (1) its decision was based on substantial evidence; (2) its decision was reasonable; and (3) the Liquor Board made sufficient findings of fact and conclusions of law.

*319On May 22, 2015, the circuit court held oral argument and issued an oral ruling affirming the decision of the Liquor Board. On May 28, 2015, the court entered its order affirming the Liquor Board’s decision. On June 22, 2015, Kougl filed a timely notice of appeal.

STANDARD OF REVIEW

Maryland law provides by statute that the action of a local liquor board is presumed to be proper and places the burden of proof upon the licensee to show that the decision complained of was arbitrary, fraudulent, unsupported by substantial evidence, illegal, or against the public interest. Md.Code (1957, 2011 Repl. Vol), Art. 2B, § 16 — 101(e)(l)(i). This Court has explained appellate review of a Liquor Board’s decision as follows:

While the last sentence of § 16-101(e)(l)(i) expressly permits the trial court, under certain circumstances, to hear additional evidence, the court may hear such evidence only to ascertain the veracity of findings of fact and conclusions of law reached by the Board. As the section does not authorize appeals de novo, the trial court may not hear additional evidence on matters not addressed by the Board. This Court has consistently explained that judicial review of a decision by the Board is similar to review of decisions by most other administrative agencies. It is a cardinal rule of administrative appeals that a reviewing court ... shall apply the substantial evidence test to final decisions of an administrative agency such as the Board, but it must not itself make independent findings of fact or substitute its judgment for that of the agency.
Judicial review of administrative action differs from appellate review of a trial court judgment. In the latter context the appellate court will search the record for evidence to support the judgment and will sustain the judgment for a reason plainly appearing on the record whether or not the reason was expressly relied upon by the trial court. However, in judicial review of agency action the court may not uphold the agency order unless it is sustaina*320ble on the agency’s findings and for the reason stated by the agency.

Blackburn v. Bd. of Liquor License Comm’rs for Balt. City, 130 Md.App. 614, 623-24, 747 A.2d 725 (2000) (emphasis added) (citations and internal quotation marks omitted).

“Of course, the reviewing court may substitute its judgment for that of the [Liquor] Board on questions of law.” Id. at 624, 747 A.2d 725. In deciding whether to substitute its judgment on a question of law, a court should accord a degree of deference to the position of the administrative agency. Md. Aviation Admin. v. Noland, 386 Md. 556, 572, 873 A.2d 1145 (2005). “Thus, an administrative agency’s interpretation and application of the statute which the agency administers should ordinarily be given considerable weight by reviewing courts.” Id. “An agency conclusion will not be upheld upon review, however, if based upon an error of law.” Hoyle v. Bd. of Liquor License Comm’rs for Balt. City, 115 Md.App. 124, 129, 692 A.2d 1 (1997).

DISCUSSION

I.

Sufficient Findings of Fact

“There is no express requirement that the Board set forth specific findings of fact and conclusions of law. In order for any meaningful review to be conducted, however, the Board must do so, at least informally.” Blackburn, 130 Md.App. at 624, 747 A.2d 725 (citation omitted).

Kougl argues that the Liquor Board did not make sufficient findings of fact to allow for meaningful judicial review. Specifically, Kougl contends that the Liquor Board’s decision consists of the following two statements, neither of which contain any findings of fact:

All right. The verdict is responsible for all three charges. The police department is not on trial here. Mr. Kougl is. And bar owners or licensees are responsible for the conduct *321of their employees, as I have said before, in all cases, in all bars, in the city of Baltimore.
All right, my finding is responsible to all three charges and close him for one month, effective immediately.

According to Kougl, the above statements were insufficient to support a legal conclusion that Kougl violated the Rules at issue.

The Liquor Board responds that there is no requirement that it set forth specific findings of fact so long as its informal findings are sufficient to allow for “meaningful appellate review.” Furthermore, the Liquor Board argues that it made informal findings of fact that were “sufficient for this Court to determine whether substantial evidence supports the Board’s conclusion that Kougl violated the three Liquor Board Rules as charged.” We agree with the Liquor Board and shall explain.

As previously indicated, Kougl, as the licensee, was charged ■with violating Rules 4.17(a), 4.17(b), and 4.18. Rule 4.17 provides:

Sexual Practices and Obscenity
(a) No licensee shall permit or suffer his premises to be used for the purpose of any sexual activity, nor shall any licensee permit or suffer any employee, patron or frequenter to solicit any person for prostitution or other immoral purposes.
(b) No licensee shall permit or suffer any person to appear in any act or other performance with breasts or the lower torso uncovered; nor shall any licensee knowingly permit or suffer his premises to be used for the conduct, exhibition or performance of an obscene act or other performance.

(Emphasis added).

Rule 4.18 provides:

Illegal Conduct
*322No licensee shall commit or allow the commission on his premises of any act which shall be contrary to any federal, state or local statute, law or ordinance or against the public peace, safety, health, welfare, quiet or morals.[4]

At the conclusion of the hearing on the charges against Kougl, the following occurred:

CHAIRMAN WARD: All right. The verdict is responsible for all three charges. The police department is not on trial here. Mr. Kougl is. And bar owners or licensees are responsible for the conduct of their employees, as I have said before, in all cases, in all bars, in the city of Baltimore.
The only question now is punishment----
All right, my finding is responsible to all three charges and close him for one month, effective immediately. And I’d be glad to hear from the other commissioners.
*323COMMISSIONER JONES: I agree. I agree that there’s a guilty for all charges. It’s clear to me that this thing took place. I have confidence in the police department. If I didn’t, the system would have a great defect in it, so I agree to the one month close order.
CHAIRMAN WARD: Commissioner?
COMMISSIONER MOORE: I disagree. I don’t think that the evidence substantiates the charges. And I’m concerned that there was a lapse of eight months. And I think that is extraordinary. It is unusual and it is not acceptable practice. And I think that that argues against a finding of guilt. So I say no.
CHAIRMAN WARD: Two to one. You heard the thing, you heard the penalty, sir. Good luck to you.

(Emphasis added).

The excerpt above makes clear that Commissioner Jones expressly relied on Det. Jackson’s testimony, and Chairman Ward implicitly relied on such testimony. Moreover, Kougl, in his testimony, did not dispute any of the evidence presented in Det. Jackson’s testimony, so a finding of guilty necessarily meant that Chairman Ward and Commissioner Jones credited Det. Jackson’s testimony as true.5

Furthermore, a “Board Summary” was issued after the hearing, which set forth the factual basis for each violation:

Violation of Rule 4.17(a) “No licensee shall permit or suffer his premises to be used for the purpose of any sexual activity, nor shall any licensee permit or suffer any employee, patron or frequenter to solicit any person for prostitution or other immoral purposes.” (Re: April 25, 2013, *324dancer solicited undercover police officer for sexual intercourse ) GUILTY 2-1
Violation of Rule 4.17(b) “No licensee shall permit or suffer any person to appear in any act or other performance with breasts or the lower torso uncovered; no[r] shall any licensee knowingly permit or suffer his premises to be used for the conduct, exhibition or performance of an obscene act or other performance” (Re: April 25, 2013, dancer exposed herself to undercover police officer) GUILTY 2-1
Violation of Rule 4.18 “No licensee shall commit or allow the commission on his premises of any act which shall be contrary to any federal, state or local statue [sic], law or ordinance or against the public peace, safety, health, welfare, quiet or morals.” (Re: April 25, 2013, dancer solicited undercover police officer for sexual intercourse; dancer exposed herself to undercover police officer) GUILTY 2-1

(Italics added) (bold in original). The facts supporting each violation, as emphasized in italics above, tracks Det. Jackson’s testimony, and provides a sufficient factual basis to allow for meaningful judicial review.

II.

Strict Liability

Kougl argues that Rules 4.17 and 4.18 require some evidence or inference therefrom that Kougl suffered, permitted, or allowed solicitation and nudity to occur, and there was no evidence to support such a finding. According to Kougl, there was insufficient evidence for the Liquor Board to infer that Kougl permitted such conduct to occur “merely by reason that the dancer solicited the detective.”

The Liquor Board responds that the three violations “are, as a matter of law, strict liability offenses,” none of which require “a licensee’s knowledge of the impermissible conduct.” The Liquor Board analogizes Rules 4.17(a) and (b) to Rule 4.01(a), which prohibits sales of alcohol to minors, to argue that both Rules “contain[] both a strict liability prohibition and a second provision with a knowledge requirement.” As *325for Rule 4.18, the Liquor Board argues that it “contains no knowledge or mens rea language. Therefore, it describes a strict liability offense.” According to the Liquor Board, Kougl was found guilty of violating the strict liability portion of the Rules.

By virtue of its argument that the charged violations are strict liability offenses, the Liquor Board concedes that the evidence adduced at the hearing was insufficient to find that Kougl had actual or constructive knowledge of the violations. Upon review of the record, we agree that there was simply no evidence of Kougl’s actual or constructive knowledge of the violations.

Whether Rules 4.17 and 4.18 impose a strict liability standard on a licensee depends on the meaning of the words “suffer,” “permit,” and “allow” as used in the Rules. In determining the meaning of such words, “we apply well-settled rules of statutory construction, the cardinal rule of [which] is to ascertain and effectuate the intent of the Legislature.” Assateague Coastkeeper v. Md. Dep’t of Env’t, 200 Md.App. 665, 708-09, 28 A.3d 178 (2011) (citations and internal quotation marks omitted).

In so doing, we look first to the normal, plain meaning of the language of the statute, read as a whole so that no word, clause, sentence or phrase is rendered surplusage, superfluous, meaningless or nugatory. If the language of a statute is clear and unambiguous, we need not look beyond the statute’s provisions and our analysis ends. Where the language of the statute is ambiguous and may be subject to more than one interpretation, however, we look to the statute’s legislative history, case law, purpose, structure, and overarching statutory scheme in aid of searching for the intention of the Legislature.

Whitley v. Md. State Bd. of Elections, 429 Md. 132, 149, 55 A.3d 37 (2012) (citations and internal quotation marks omitted).

*326In accord with the above standard, we turn to the plain meaning of the subject terms in the Rules. Black’s Law Dictionary defines “suffer” as:

To allow, to admit, or to permit. It includes knowledge of what is to be done under sufferance. To suffer an act to be done or a condition to exist is to permit or consent to it; to approve of it, and not to hinder it. It implies knowledge, a willingness of the mind and responsible control or ability to prevent.

Black’s Law Dictionary 1432 (6th ed.1990) (emphasis added) (citations omitted). “Permit” is defined as: “To suffer, allow, consent, let; to give leave or license; to acquiesce, by failure to prevent, or to expressly assent or agree to the doing of an act.” Id. at 1140. Finally, Black’s Law Dictionary defines “allow” as:

The word has no rigid or precise meaning, its import varying according to circumstances or context in connection with which it is used. It may mean to bestow or assign to any one as his right or due. To approve of, accept as true, admit, concede, adopt, or fix. To grant something as a deduction or an addition; to abate or deduct; as, to allow a sum for leakage. To sanction, either directly or indirectly, as opposed to merely suffering a thing to be done; to acquiesce in; to suffer; to tolerate.

Id. at 76 (emphasis added) (citations omitted).

No Maryland appellate opinion exists regarding the meaning of these words in the context of the Liquor Board Rules. Other jurisdictions that use “suffer,” “permit,” or “allow,” in their liquor board rules are divided on whether such terms impose a strict liability standard, or whether knowledge on the part of the licensee must be proven.

Wisconsin, Oregon, and New Jersey courts have held that their states’ liquor board rules, all of which use the words “suffer,” “permit,” or “allow,” impose strict liability on the part of the licensee. In City of West Allis v. Megna, the Supreme Court of Wisconsin held that an ordinance that stated that no licensee shall “either directly or indirectly *327suffer or permit any person of either sex under the age of 21 years ... to enter or be on such licensed premises for any purpose,” imposed a strict liability standard. 26 Wis.2d 545, 133 N.W.2d 252, 253 (1965). The Court determined that “[p]roof of knowledge on the part of the tavern keeper that the patron is actually under age is not required by the statute in order to obtain a conviction,” and that such standard “is a price that the operator pays for the privilege of becoming licensed.” Id. at 254.

Similarly, the Court of Appeals of Oregon held in DK Entertainment, LLC v. Oregon Liquor Control Commission that, when interpreting an ordinance stating that “ ‘[n]o licensee or permittee will permit any unlawful activity on the licensed premises or in areas the licensee controls that are adjacent to or outside the premises,’ ” the Oregon Liquor Control Commission “may impute to the licensee the knowledge of the licensees employees.” 249 Or.App. 659, 278 P.3d 112, 113 n. 2, 114 (2012). Finally, in Division of Alcoholic Beverage Control v. Maynards Inc., the Supreme Court of New Jersey held an ordinance providing that “[n]o licensee shall allow, permit or suffer in or upon the licensed premises any unlawful possession of or any unlawful activity pertaining to” narcotics and other controlled dangerous substances imposed a strict liability standard. 192 N.J. 158, 927 A.2d 525, 536 (2007). The Court noted that “[i]t has long been the law in New Jersey that, in the context of the regulation of alcoholic beverages, ‘the word suffer ... imposes responsibility on a licensee, regardless of knowledge, where there is a failure to prevent the prohibited conduct by those occupying the premises with his authority.’ ” Id. at 538 (ellipses and italics in original).

Conversely, other courts, such as those in Colorado, California, New York, and Washington, have held that the terms “suffer,” “permit,” or “allow” require actual or constructive knowledge. In Full Moon Saloon, Inc. v. City of Loveland, the Colorado Court of Appeals held that a statute making it unlawful to, among other things, “permit the sale, serving, giving, or procuring of any alcohol beverage to or for a person *328under the age of twenty-one years” required that “some level of knowledge by the licensee must be established by the evidence.” Ill P.3d 568, 569-70 (Colo.App.2005). The Court explained:

The word “permit” connotes affirmative or knowing conduct. Thus, licensees and their employees and agents “permit” such conduct if they have actual knowledge of the violation or have constructive knowledge that it is occurring.
Our conclusion that constructive knowledge of liquor code violations is sufficient to hold a licensee responsible for permitting the violation is consistent with cases from other jurisdictions. See Spitz v. Mun. Court, 127 Ariz. 405, 621 P.2d 911 (1980) (licensee is deemed to have constructive knowledge of the age of underage buyer if licensee provides alcohol to the minor and fails to follow certain procedures); Laube v. Stroh, 2 Cal.App.4th 364, 3 Cal.Rptr.2d 779 (1992) (to suspend liquor license, evidence must be presented that licensee had either actual or constructive knowledge of activity); Pinacoteca Corp. v. Dep’t of Bus. Regulation, 580 So.2d 881 (Fla.Dist.Ct.App.[4th DCA]1991) (where activity is persistent and recurring, licensee’s knowledge of the activity may be inferred); State v. Engberg, 109 Idaho 530, 708 P.2d 935 (Ct.App.1985) (violation may be found if licensee had constructive knowledge of the prohibited activity); Town & Country Lanes, Inc. v. Liquor Control Comm’n, 179 Mich.App. 649, 446 N.W.2d 335 (1989) (licensee violated liquor license rule by failing to exercise reasonable diligence to ascertain the age of underage customer); Leake v. Sarafan, 35 N.Y.2d 83, 358 N.Y.S.2d 749, 315 N.E.2d 796 (1974) (to sustain a violation, licensee must have knowledge of the activity or the opportunity through reasonable diligence to acquire knowledge of the alleged acts); Smith v. Bd. of Liquor Control, 96 Ohio App. 396, 121 N.E.2d 920 (1954) (licensee must have actual or constructive knowledge of prohibited activity) Tex. Alcoholic Beverage Comm’n v. Sanchez, 96 S.W.3d 483 (Tex.App.2002) (to suspend liquor *329license, evidence must be presented that licensee had either actual or constructive knowledge of activity); Reeb, Inc. v. Wash. State Liquor Control Bd., 24 Wash.App. 349, 600 P.2d 578 (1979) (to “permit” a violation, licensee must have actual or constructive knowledge of the activity).

Id. at 570-71 (emphasis added).

Similarly, in Laube v. Stroh, the Court of Appeal of California “h[e]ld that a licensee must have knowledge, either actual or constructive, before he or she can be found to have ‘permitted’ unacceptable conduct on a licensed premises.” 2 Cal.App.4th 364, 3 Cal.Rptr.2d 779, 788 (1992). The Court continued: “It defies logic to charge someone with permitting conduct of which they are not aware.” Id. Finally, the Court noted that “[failure to prevent the problem from recurring, once the licensee knows of it, is to ‘permit’ by a failure to take preventive action.” Id. at 789.

Likewise, in Leake, the Court of Appeals of New York noted the following regarding the statutory words “suffer” or “permit”:

There is no particular mystery or magic in the language of the statute. This court in Matter of Migliaccio v. O’Connell [307 N.Y. 566, 122 N.E.2d 914 (1955)] defined the intent of the statute when it stated that in considering what is implied by the phrase suffer or permit as employed in the statute quoted above, we are guided by what was written for this court (per Cardozo, J.) in People ex rel Price v. Sheffield Farms [225 N.Y. 25, 121 N.E. 474 (1918) ]: Sufferance as here prohibited implies knowledge or the opportunity through reasonable diligence to acquire knowledge. This presupposes in most cases a fair measure at least of continuity and permanence.

358 N.Y.S.2d 749, 315 N.E.2d at 797 (emphasis added) (citations and internal quotation marks omitted).

Finally, in Reeb, the Court of Appeals of Washington held: The word “permit” as used in the regulation does not imply that the licensee must have permanently sanctioned the prohibited act; it refers to the licensee’s actual or con*330structive knowledge of the circumstances which would foreseeably lead to the prohibited activity. Reeb was aware of the propensity of its dancers to engage in questionable conduct and chose nevertheless to maintain topless dancing on its licensed premises. Under these circumstances, the Board could find that Reeb permitted the conduct in violation of [the Liquor Board Rules]. This violation is sufficient to support the 7-day license suspension imposed by the Board.

600 P.2d at 581 (citation omitted).

Considering the plain meaning of the words “suffer,” “permit,” and “allow,” we conclude that the use of those words in Rules 4.17 and 4.18 necessarily require that “some level of knowledge by the licensee must be established by the evidence.” Full Moon Saloon, 111 P.3d at 569. As indicated above, the definition of “suffer” expressly states that “[i]t includes knowledge of what is to be done under sufferance.” Black’s Law Dictionary at 1432. The terms “permit” and “allow” also imply knowledge of what is to be permitted or allowed. See First Nat’l Bank & Trust Co. of Port Chester v. New York Title Ins. Co., 171 Misc. 854, 12 N.Y.S.2d 703, 709 (1939) (“It has been said that every definition of ‘suffer’ and ‘permit’ includes knowledge of what is to be done under the sufferance and permission, and intention that what is done is to be done.”).

We are fully cognizant of the strong public policy considerations behind requiring a strict liability standard in the Liquor Board Rules. Unlawful or immoral activities may be associated with places where alcoholic beverages are sold. However, one of the primary purposes of the Liquor Board Rules is to clearly inform licensees of their responsibilities regarding prohibited activities so that they can conform their conduct and the conduct of their businesses to be in compliance with the Rules. Interpreting the words used in the Liquor Board Rules at variance with their plain meaning does not advance the public purpose of achieving compliance with the Rules. Although we are obligated to give deference to the Liquor *331Board’s interpretation of its Rules, see Noland, 386 Md. at 572, 873 A.2d 1145, “[a]n agency conclusion will not be upheld upon review, however, if based upon an error of law,” Hoyle, 115 Md.App. at 129, 692 A.2d 1. Accordingly, we hold that the Liquor Board erred in interpreting the words “suffer,” “permit,” and “allow” in Rules 4.17 and 4.18 to impose a strict liability standard on licensees.6

We also hold that the knowledge requirement implicit in the terms “suffer,” “permit,” and “allow” can be satisfied by proof of either actual or constructive knowledge on the part of the licensee. See Full Moon Saloon, 111 P.3d at 570. Actual knowledge is, of course, actual awareness of the prohibited activity. Actual knowledge also includes the concept of “deliberate ignorance” or “willful blindness.” See Rice v. State, 136 Md.App. 593, 604, 766 A.2d 663 (2001). In State v. McCallum, Judge Chasanow explained the meaning of this form of knowledge in a concurring opinion:

There is more than one mental state that may constitute “knowledge.” The first and highest form of “knowledge” is actual knowledge, that is, an actual awareness or an actual belief that a fact exists. A second form of “knowledge” is what has often been called “deliberate ignorance” or “willful blindness.” R. Perkins, Criminal Law, Ch. 7, § 4 at 687 (1957). The latter form of “knowledge” exists where a person believes that it is probable that something is a fact, but deliberately shuts his or her eyes or avoids making reasonable inquiry with a conscious purpose to avoid learning the truth. See 1 W. LaFave & A. Scott, Substantive Criminal Law, § 3.5 at 307 (1986), and authorities cited therein. In United States v. Jewell, 532 F.2d 697 (9th Cir.), cert. denied, 426 U.S. 951, 96 S.Ct. 3173, 49 L.Ed.2d 1188 (1976), the 9th Circuit Court of Appeals noted that the *332deliberate ignorance or willful blindness form of knowledge has been accepted by leading commentators in the United States and in England. The opinion quoted from Professor Glanville Williams’ Criminal Law: The General Part, § 57 at 157, 159 (2d ed.1961), as follows:
“To the requirement of actual knowledge there is one strictly limited exception.... [T]he rule is that if a party has his suspicion aroused but then deliberately omits to make further enquiries, because he wishes to remain in ignorance, he is deemed to have knowledge____The rule that wilful blindness is equivalent to knowledge is essential, and is found throughout the criminal law.”

Id. at 700.

321 Md. 451, 458-59, 583 A.2d 250 (1991) (Chasanow, J., concurring) (emphasis added).

Constructive knowledge, however, implies knowledge where a licensee should have known of the prohibited activity if reasonable care and diligence had been exercised. See Leake, 358 N.Y.S.2d 749, 315 N.E.2d at 797 (stating that “it must be demonstrated that the licensee had knowledge or the opportunity through reasonable diligence to acquire knowledge of the alleged acts”); Morgan v. Bd. of Water Works of Pueblo, 837 P.2d 300, 303 (Colo.App.1992) (stating that, if in the exercise of ordinary diligence a party should have known a fact, it will be deemed to have knowledge).

The Colorado Court of Appeals explained constructive knowledge as follows:

Constructive knowledge may be inferred if the conduct occurs openly, such that a reasonable person would observe it. If knowledge of the prohibited conduct could have been obtained through the exercise of reasonable care and diligence, constructive knowledge may be inferred.
Applying a constructive knowledge standard does not place an undue burden on the licensee because constructive knowledge requires only reasonable care and diligence and does not require extraordinary vigilance. Constructive *333knowledge means knowledge that one using reasonable care or diligence should have, and therefore that is attributed by law to a given person.

Full Moon Saloon, 111 P.3d at 570 (citations and internal quotation marks omitted).

Nevertheless, the Liquor Board argues that Rule 4.17 is similar to Rule 4.01(a), which we held in Hoyle to impose strict liability. We disagree.

Rule 4.01(a) provides: “No licensee shall sell or furnish alcoholic beverages to any person under twenty-one (21) years of age or to any person with the knowledge that such person is purchasing or acquiring such beverages for consumption by any person under twenty-one (21) years of age.” Hoyle, 115 Md.App. at 130, 692 A.2d 1. In Hoyle, the Liquor Board ruled that Martini’s Bar had violated Rule 4.01(a) when one of its employees served alcohol to a minor. Id. at 128, 692 A.2d 1. Martini’s Bar appealed, arguing that this Court “should read a ‘due caution’ provision into the [R]ule.” Id. at 131, 692 A.2d 1. We declined to do so, and instead held that the Rule “imposes strict liability for the sale on the licensees.” Id. at 133, 692 A.2d 1.

Our holding in Hoyle is clearly distinguishable from our holding in the case sub judice. Rule 4.01(a), unlike Rule 4.17, does not use the words “suffer,” “permit,” or “allow,” which, as explained above, necessarily include an element of knowledge. Instead, Rule 4.01(a) sets forth a simple directive — “No licensee shall sell or furnish ....”, which carries no implication of knowledge.

Finally, because the word “knowingly” appears in the second independent clause (“second clause”), and not in the first independent clause (“first clause”), of Rule 4.17(b), the Liquor Board claims that such sentence construction necessarily implies a strict liability standard in the first clause. Again, we disagree.

To reiterate, Rule 4.17(b) states:

*334No licensee shall permit or suffer any person to appear in any act or other performance with breasts or the lower torso uncovered; nor shall any licensee knowingly permit or suffer his premises to be used for the conduct, exhibition or performance of an obscene act or other performance.

The first clause clearly identifies the prohibited act — any person “appearing] ... with breasts or the lower torso uncovered.” On the other hand, the second clause prohibits the performance of an “obscene act or other performance.” In our view, the word “knowingly” appears in the second clause because an element of knowledge is required to constitutionally regulate obscenity, a class of unprotected speech that is not clearly defined in First Amendment jurisprudence. See generally Miller v. California, 413 U.S. 15, 20, 93 S.Ct. 2607, 37 L.Ed.2d 419 (1973) (summarizing “the somewhat tortured history of the Court’s obscenity decisions”).

Accordingly, we hold that the Liquor Board erred in finding that Kougl was guilty of violating Rules 4.17(a), 4.17(b), and 4.18, because there was no evidence that Kougl had actual or constructive knowledge of his employee’s prohibited activity.

JUDGMENT OF THE CIRCUIT COURT FOR BALTIMORE CITY REVERSED. CASE REMANDED TO THAT COURT WITH INSTRUCTIONS TO ENTER A JUDGMENT REVERSING THE DECISION OF THE LIQUOR BOARD RENDERED ON JULY 17, 2014. COSTS TO BE PAID BY APPELLEE.

4.5.10 Notes & Questions (Kougl v. Board) 4.5.10 Notes & Questions (Kougl v. Board)

Review Questions

  1. What was the defendant's argument against the charges (regarding solicitation of prostitution and indecent exposure)? 
  2. Why did the prosecution believe the provisions ought to have been read as strict-liability provisions?   
  3. Recall State v. Nations. Does the court here adopt the same interpretation of the knowledge requirement? Could the State have shown D had constructive knowledge? Why was the argument not made? 
  4. Do you agree with the court’s decision rejecting strict liability in this case? Consider cases in which the threat to public welfare is great, the cost of proving mental state is high, and the sanctions are minimal and not stigmatizing. When might it be worth it to require only strict liability? 
  5. Use the following statutes when answering the questions below regarding the Kougl case: 

Rule 4.15 — Sexual Practices and Obscenity

(a) A licensee may not permit the licensed premises to be used for any sexual activity. A licensee, any employee, patron or frequenter may not solicit any person for prostitution or other immoral purposes.

(b) A licensee may not permit a person to appear in an act or other performance with breasts or the lower torso uncovered. A licensee may not knowingly permit the licensed premises to be used for the conduct, exhibition, or any unlawful performance….

Rule 4.16 — Illegal Conduct

A licensee may not commit or allow the commission on the licensed premises of an act that is contrary to any federal, State or local statute, law or ordinance or that is against the public peace, safety, health, welfare, quiet, or morals.

  1. What was D’s defense against the charges that his establishment violated rules regarding solicitation of prostitution and indecent exposure? 
  2. Explain the Prosecution’s argument for why these provisions ought to be read as “Strict Liability” provisions. 
  3. Use the statute below, cited by the state, to bolster your argument: 

Rule 4.01 – Minors (a) Sales to Minors - No licensee shall sell, furnish, or allow to be furnished by his/her agent or employee alcoholic beverages to any person under twenty-one (21) years of age or to any person with the knowledge that such person is purchasing or acquiring such beverages for consumption by any person under twenty-one (21) years of age. 

          Explain how State asked the court to interpret 4.15 (below), given its interpretation of 4.01: 

Rule 4.15 — Sexual Practices and Obscenity

(a) A licensee may not permit the licensed premises to be used for any sexual activity. A licensee, any employee, patron or frequenter may not solicit any person for prostitution or other immoral purposes.

(b) A licensee may not permit a person to appear in an act or other performance with breasts or the lower torso uncovered. A licensee may not knowingly permit the licensed premises to be used for the conduct, exhibition, or any unlawful performance….

Problem: Kougl and Strict Liability

Ever since a closed-head injury left her with chronic pain, Diana has become a frequent user of painkillers. When her original prescriptions ran out, she began buying the drugs illegally from local dealers. Like many women, for years she has carried pepper spray (a tear gas spray) in her purse in the hopes that it would protect her in the event of a sexual assault. It’s been at the bottom of her purse for so long that she’d forgotten about it. Last month, police arrested her attempting to buy  drugs from an undercover officer. When they searched her purse and found the pepper spray, in addition to the drug-related charge, she was charged with the crime below, a misdemeanor which may add time to her sentence. The relevant statute is as follows: 

CA Penal Code Chap. 1, 22810. Notwithstanding any other provision of law, any person may purchase, possess, or use tear gas or any tear gas weapon for the projection or release of tear gas if the tear gas or tear gas weapon is used solely for self-defense purposes, subject to the following requirements:

… (b) No person addicted to any narcotic drug shall purchase, possess, or use tear gas or any tear gas weapon. 

Assume that Diana hires you as her defense attorney. Answer the following questions: 

1) Assuming that Diana is found to be addicted to painkillers (narcotics), explain the State’s case against her.  

2) Which of the following, assuming the jury believes Diana, might provide a defense? 

  • Her testimony that she did not know she possessed pepper spray.
  • Her testimony that she did not know she was an addict
  • Her testimony that she did not know about the law against addicts possessing pepper spray.
  • Her testimony that she was not negligent in carrying pepper spray because breaking the law was not unreasonable under the circumstances.

3) Reconsider the justifications for and against strict liability laws, and articulate the case for and against convicting Diana. 

4) Can you reconcile the outcome you predict in Diana’s case with the outcome in Kougl? Explain. 

4.5.11 People v. Navarro 4.5.11 People v. Navarro

Appellate Department, Superior Court, Los Angeles

People v. Navarro
99 Cal. App. 3d Supp. 1

Crim. A. No. 17137.

October 22, 1979; THE PEOPLE Plaintiff and Respondent, v. WALDO NAVARRO, Defendant and Appellant.

(Opinion by Dowds, J., with Cole, P. J., and Saeta, J., concurring.)

COUNSEL

Harland W. Braun for Defendant and Appellant.

John K. Van de Kamp, District Attorney, Donald J. Kaplan and George M. Palmer, Deputy District Attorneys, for Plaintiff and Respondent.

OPINION

DOWDS, J.

Defendant, charged with a violation of Penal Code section 487, subdivision 1, grand theft, appeals his conviction after a jury trial of petty theft, a lesser but necessarily included offense. His contention on appeal is that the jury was improperly instructed. The only facts [99 Cal. App. 3d Supp. 3] set forth in the record on appeal are that defendant was charged with stealing four wooden beams from a construction site and that the state of the evidence was such that the jury could have found that the defendant believed either (1) that the beams had been abandoned as worthless and the owner had no objection to his taking them or (2) that they had substantial value, had not been abandoned and he had no right to take them.

The court refused two jury instructions proposed by defendant reading as follows:

Defendant's A

"If one takes personal property with the good faith belief that the property has been abandoned or discarded by the true owner, he is not guilty of theft. This is the case even if such good faith belief is unreasonable. The prosecutor must prove beyond a reasonable doubt that the defendant did not so believe for you to convict a defendant of theft."

Defendant's B

"If one takes personal property with the good faith belief that he has permission to take the property, he is not guilty of theft. This is the case even if such good faith belief is unreasonable.

The prosecutor must prove beyond a reasonable doubt that the defendant did not so believe for you to convict a defendant of theft."

Instead, the court instructed the jury in the words of the following modified instructions:

Modified-Defendant's A

"If one takes personal property in the reasonable and good faith belief that the property has been abandoned or discarded by the true owner, he is not guilty of theft."

Modified-Defendant's B

"If one takes personal property in the reasonable and good faith belief that he has the consent or permission of the owner to take the property, he is not guilty of theft. [99 Cal. App. 3d Supp. 4]

"If you have a reasonable doubt that the defendant had the required criminal intent as specified in these instructions, the defendant is entitled to an acquittal."

Accordingly, the question for determination on appeal is whether the defendant should be acquitted if there is a reasonable doubt that he had a good faith belief that the property had been abandoned or that he had the permission of the owner to take the property or whether that belief must be a reasonable one as well as being held in good faith.

A recent decision by the California Supreme Court throws light on this question. In People v. Wetmore (1978) 22 Cal. 3d 318 [149 Cal. Rptr. 265, 583 P.2d 1308], defendant was charged with burglary, like theft a specific intent crime. fn. 1 The Supreme Court held that the trial court had erroneously refused to consider at the guilt phase of the trial evidence that, because of mental illness, defendant was incapable of forming the specific intent required for conviction of the crime, instead of receiving such evidence only in respect of his plea of not guilty by reason of insanity. The court disapproved dictum in People v. Wells (1949) 33 Cal. 2d 330 [202 P.2d 53] that if psychiatric reports described a defendant's insanity as well as his diminished capacity, the evidence was inadmissible to prove his lack of specific intent, stating, at pages 323-324: "In holding that defendant's psychiatric evidence could not be utilized to prove that he lacked the specific intent required for the offense of burglary, the trial court followed a dictum laid down in our decision in People v. Wells, supra, 33 Cal. 2d 330. Wells, the seminal decision which established the doctrine of diminished capacity in California law, held that 'evidence of diminished mental capacity, whether caused by intoxication, trauma, or disease, can be used to show that a defendant did not have a specific mental state essential to an offense.' (People v. Conley (1966) 64 Cal. 2d 310, 316 [49 Cal.Rptr. [99 Cal. App. 3d Supp. 5] or other essential mental state, then that evidence is inadmissible under the not guilty plea ... (P. 351.)

"As we shall explain, the Wells dictum imposes an illogical and unworkable rule which has not been followed in subsequent cases. Wells spoke of excluding evidence which tended to prove 'lack of mental capacity ... because of legal insanity.' (P. 350.) Mental incapacity does not occur 'because of legal insanity;' instead both insanity and diminished capacity are legal conclusions derived from evidence of defendant's mental condition. (See Comment (1971) 18 UCLA L.Rev. 561, 563-564, fn. 11.) Consequently, if the evidence of a defendant's mental illness indicates that the defendant lacked the specific intent to commit the charged crime such evidence cannot reasonably be ignored at the guilt trial merely because it might (but might not) also persuade the trier of fact that the defendant is insane.

"Wells' distinction between evidence that defendant did not entertain the requisite intent, which is admissible, and evidence that he could not entertain that intent, which is inadmissible, cannot be supported. '[A]s a matter of logic, any proof tending to show that a certain mental condition could not exist is relevant and should be admissible to show that it did not exist. And, of course, proof that something could not exist is the best possible evidence that it did not exist.' (Louisell & Hazard, Insanity as a Defense: The Bifurcated Trial (1961) 49 Cal.L.Rev. 805, 819.) Moreover, as Justice Kaus pointed out in People v. Steele (1965) 237 Cal. App. 2d 182, 190-191 [46 Cal. Rptr. 704], evidence which tends to prove that a defendant could not entertain a certain intent may, when subject to cross-examination, convince the trier of fact that defendant was able to entertain the intent but did not do so on the occasion of the crime. Thus, Steele concludes, the trial court cannot refuse to admit such evidence when offered to prove diminished capacity."

The court concluded, at page 327: "We therefore hold that evidence of diminished capacity is admissible at the guilt phase whether or not that evidence may also be probative of insanity. The trial court erred when, relying on the Wells dictum, it refused to consider evidence of diminished capacity in determining defendant's guilt."

The instant case, does not, of course, involve evidence of mental illness. Evidence was presented, however, from which the jury could have concluded that defendant believed that the wooden beams had been abandoned and that the owner had no objection to his taking them, i.e., [99 Cal. App. 3d Supp. 6] that he lacked the specific criminal intent required to commit the crime of theft (intent permanently to deprive an owner of his property). A similar situation existed in People v. Photo (1941) 45 Cal. App. 2d 345 [114 P.2d 71], where defendant's conviction of grand theft for the taking of certain boxes of oranges which he thought he had purchased was reversed, the Court of Appeal stating at page 353: "Felonious intent is an essence of the crime of larceny. The general rule stated in 36 C. J., section 105, page 764, is: 'If one, in good faith, takes the property of another, believing it to be legally his own, or that he has a legal right to its possession, he is not guilty of larceny, although his claim is based on a misconception of the law or of his rights under it, for although ignorance of law and honest intentions cannot shield a man from civil liability for a trespass committed by him, yet they do protect him from criminal liability, by divesting the act of the felonious intent without which it cannot be a crime. It is necessary, however, in all cases that the claim of right to be a bona fide one, and not a mere cover for a felonious taking, and must be something more than a vague impression; it must amount to an honest conviction. Knowledge of the existence of an adverse claim by another person does not negative the existence of good faith.'

"From the evidence it appears that Photo apparently took the fruit, under a claim of title in himself, and if done in good faith after receiving what he thought was legal advice though it might have been erroneous, a presumption arose in his favor that the taking lacked the elements necessary to constitute larceny."

Earlier California cases are to the same effect. In People v. Devine (1892) 95 Cal. 227 [30 P. 378], defendant's conviction of larceny was reversed. He had driven away in a wagon, without any attempt at secrecy, a number of hogs, his own and three bearing another's mark or brand. The Supreme Court pointed out: "There are cases in which all the knowledge which a person might have acquired by due diligence is to be imputed to him. But where a felonious intent must be proven, it can be done only by proving what the accused knew. One cannot intend to steal property which he believes to be his own. He may be careless, and omit to make an effort to ascertain that the property which he thinks his own belongs to another; but so long as he believes it to be his own, he cannot feloniously steal it ..." (Id. at pp. 230-231.)

In re Bayles (1920) 47 Cal. App. 517 [190 P. 1034] concerned the owner and manager of an apartment house who was convicted of larceny [99 Cal. App. 3d Supp. 7] based on evidence that she had seized a sewing machine and victrola of a former tenant under claim of lien to secure certain charges allegedly owed by the tenant. The Court of Appeal declined to decide such fine points relating to the validity of the claim of lien as whether the tenant had left the apartment sufficiently clean, stating at pages 520-521: "[E]ven though the standard of cleanliness exacted by the petitioner should be found by the jury to be an unreasonable standard, and not the standard contemplated by the agreement between the parties, yet, we think petitioner would not be guilty of grand larceny. The record discloses no evil or felonious intent upon the part of the petitioner; she was merely seeking to enforce her civil rights as she believed them to exist. Larceny is the felonious stealing, taking, carrying, leading, or driving away the personal property of another. (Sec. 484, Pen. Code.) Every taking by one person of the personal property of another, without his consent, is not larceny. Felonious intent is of the essence of the crime of larceny. (People v. Devine, 95 Cal. 227 [30 Pac. 378].) If a jury should determine that the apartment was clean, according to the standard contemplated by the parties at the time they made their agreement, at the time petitioner took possession of the goods, and that, therefore, no money was due petitioner for cleaning the same, Mr. Tucker could be amply compensated in damages for the wrongful detention of his property. (Sec. 667, Code Civ. Proc., and sec. 3336, Civ. Code.) But this question must be tried out in a civil action, and not in a criminal proceeding. (Sec. 3379, Civ. Code; sec. 3380, Civ. Code; secs. 870, 509, 510, 511 and 512, Code Civ. Proc.)"

Defendant was discharged from custody pursuant to a writ of habeas corpus.

Cases in other jurisdictions also hold that where the law requires a specific criminal intent, it is not enough merely to prove that a reasonable man would have had that intent, without meeting the burden of proof that the defendant himself also entertained it. For example, in State v. Ebbeller (1920) 283 Mo. 57. [222 S.W. 396], a conviction of knowingly receiving a stolen automobile was reversed because the court gave the following erroneous jury instruction: "'By the term "knowing" that the property was stolen is not meant absolute personal and certain knowledge on the part of the defendant that the property mentioned in the indictment had been stolen, but such knowledge and information in his possession at the time he received the same, if you believe he did receive it, as would put a reasonably prudent man, exercising ordinary caution, on his guard, and would cause such a man exercising such caution, [99 Cal. App. 3d Supp. 8]and under circumstances which you believe defendant received the property, to believe and be satisfied that the property had been stolen.'" (222 S.W. at p. 397.)

In reversing, the court pointed out the error in the instruction as follows: "It will be noticed that the instruction does permit a conviction if the facts were such as (in the opinion of the jury) would have caused a reasonably prudent person, exercising ordinary caution, to have believed that the property had been stolen at the time received.

"We are inclined to the view ... that the learned attorney representing the appellant is correct in stating that--"'The question is not what some other person would have believed and known from the circumstances attending the receipt of the property, but what did this defendant believe and know.'" (Id.)

Similarly, in Kasle v. United States (6th Cir. 1916) 233 Fed. 878 a conviction of receiving stolen goods was reversed because of error in jury instructions which the appellate court read as informing the jury that the defendant could be convicted if a reasonable and honest man of average intelligence would have known the goods were stolen under the facts existing at the time, the court stating: "The effect of such tests was to charge the accused with guilty knowledge or not upon what the jury might find would have induced belief in the mind of a man such as they were told to consider, rather than the belief that was actually created in the mind of the accused; or, at last, the accused might be condemned even if his only fault consisted in being less cautious or suspicious than honest men of average intelligence are of the acts of others. The result of the rule of the charge would be to convict a man, not because guilty, but because stupid. The issue was whether the accused had knowledge--not whether some other person would have obtained knowledge--that the goods had been stolen." (Id at p. 887)

In State v. Aschenbrenner (1943) 171 Ore. 664 [138 P.2d 911] the following instruction in a larceny case was held to be erroneous: "You are instructed that in order to convict the defendants of the crime charged, it is necessary for the state to prove beyond a reasonable doubt that the defendants had knowledge, or notice of such facts and circumstances as would have convinced a reasonable man of ordinary intelligence and observation, that estray sheep were in the Aschenbrenner flock, and if upon the whole evidence in this case you have a reasonable doubt that the facts and circumstances known to the defendants [99 Cal. App. 3d Supp. 9] (if you find any were known to them), were such that a reasonable man of ordinary intelligence and observation would have known that estray sheep were in the Aschenbrenner flock, then you should find the defendants not guilty of the crime charged.

"On the other hand, if you find from the evidence beyond a reasonable doubt that the circumstances were such as to lead a rational man, a man of ordinary intelligence and observation, to know that estray sheep were in the Aschenbrenner flock, such circumstances would be sufficient to justify you in finding that the defendants knew estray sheep were in the Aschenbrenner flock." (171 Ore. at pp. 668-669.) The Supreme Court of Oregon said: "We are of the opinion that the circuit court erred in instructing the jury that circumstances which would lead a man of ordinary intelligence and observation to know that estray sheep were in the Aschenbrenner flock 'would be sufficient to justify' it 'in finding that the defendants knew' that fact. It seldom happens that direct evidence can be produced that the accused had actual knowledge of a fact. And in the absence of direct evidence knowledge may be inferred from circumstances. The issue, however, was whether the defendants had knowledge that estray sheep were in the flock, not whether some other person would or could have obtained that knowledge under the circumstances disclosed by the evidence. It might have appeared to the jury that the defendants, or some of them, did not possess the intelligence or observation of an ordinary man, or that in a given instance they, or some of them, acted unwisely, unreasonably, imprudently or carelessly, but not with criminal intent. The instruction complained of was not only erroneous, but highly prejudicial to the defendants' rights." (v. at p. 674) The Supreme Court of Arizona, in Reser v. State (1924) 27 Ariz. 43 [229 P. 936] held erroneous a jury instruction in a receiving stolen property case setting forth a test of whether the defendant received the property "under such circumstances that a man of ordinary prudence and caution would be satisfied that it was stolen property," the court stating: "It is contended that these instructions are erroneous, because they authorize a conviction even though appellant may not have had guilty knowledge that the property was stolen. It will be observed that they do permit a verdict of guilty whether defendant's knowledge of the theft was actual or merely imputed. Under the statement that, if the facts under which the property was received by the accused were such that a man of ordinary prudence and caution would have been satisfied that it was stolen, the law imputed to the defendant knowledge of this fact, the jury's only duty on the question of guilty knowledge was to decide what an ordinarily prudent [99 Cal. App. 3d Supp. 10] and cautious person would have done under the circumstances. This was equivalent to advising the jury that, if it concluded such a man would have been satisfied the property was stolen, the law would attribute the same knowledge to the accused. It relieved the jury from the necessity of considering whether the circumstances under which the defendant received the property were such as to cause him to realize individually it was stolen, and permitted his conviction upon the jury's determination of what an ordinarily prudent and cautious man would have done under the circumstances, without taking cognizance of the fact that there was nothing to show whether the accused possessed the prudence and caution of the one by whose actions his were tested. The question for the jury was not whether the facts would have given the ordinary person knowledge of the theft, but whether they had such effect upon the defendant himself with his understanding of their significance. Many people are not as cautious and prudent as the ordinary person, hence the circumstances might have meant knowledge to him, and nothing to the accused." (229 P. at p. 937.) Other cases from other jurisdictions setting forth the same rule could be cited and we appreciate that other cases can be found in which its application is not so clear. [1a] The proper rule, it seems to us, is set forth in Perkins on Criminal Law (2d ed. 1969) at pages 940-941: "If no specific intent or other special mental element is required for guilt of the offense charged, a mistake of fact will not be recognized as an excuse unless it was based upon reasonable grounds ... [On the other hand,]because of the requirement of a specific intent to steal there is no such thing as larceny by negligence. One does not commit this offense by carrying away the chattel of another in the mistaken belief that it is his own, no matter how great may have been the fault leading to this belief, if the belief itself is genuine."

[2] La Fave and Scott, Handbook on Criminal Law (1972) sets forth at page 357 what the authors call the "...rather simple rule that an honest mistake of fact or law is a defense when it negates a required mental element of the crime ..." As an example they refer to the crime of receiving stolen property, stating "...if the defendant by a mistake of either fact or law did not know the goods were stolen, even though the circumstances would have led a prudent man to believe they were stolen, he does not have the required mental state and thus may not be convicted of the crime."

[1b] In the instant case the trial court in effect instructed the jury that even though defendant in good faith believed he had the right to [99 Cal. App. 3d Supp. 11] take the beams, and thus lacked the specific intent required for the crime of theft, he should be convicted unless such belief was reasonable. In doing so it erred. fn. 2 It is true that if the jury thought the defendant's belief to be unreasonable, it might infer that he did not in good faith hold such belief. If, however, it concluded that defendant in good faith believed that he had the right to take the beams, even though such belief was unreasonable as measured by the objective standard of a hypothetical reasonable man, defendant was entitled to an acquittal since the specific intent required to be proved as an element of the offense had not been established. fn. 3

The People's reliance on People v. Mayberry (1975) 15 Cal. 3d 143 [125 Cal. Rptr. 745, 542 P.2d 1337] is misplaced. The discussion in that case involved the propriety of an instruction on mistake of fact in respect of charges of rape and kidnaping, general intent crimes, a different question from that here presented.

The judgment is reversed.

Cole, P. J., and Saeta, J., concurred.

FN 1. Wetmore had occupied an apartment in the owner's absence and used or destroyed some of the contents, allegedly in the good faith but unreasonable belief, induced by mental illness, that he owned the apartment and its contents.

FN 2. The portion of the court's instruction reading "If you have a reasonable doubt that the defendant had the required criminal intent ... the defendant is entitled to an acquittal" is, however, correct.

FN 3. Instruction No. 4.35 in the fourth edition of California Jury Instructions--Criminal, relating to ignorance or mistake of fact, reads as follows: "An act committed or an omission made under an ignorance or mistake of fact which disproves any criminal intent is not a crime.

"Thus a person is not guilty of a crime if he commits an act or omits to act under an honest and reasonable belief in the existence of certain facts and circumstances which, if true, would make such act or omission lawful." The use note for this instruction omits the caveat found under instruction No. 4.36, relating to ignorance or mistake of law, to the effect that the instruction would be inappropriate if the mistake may negative a specific intent or other mental state which the crime requires. We think trial judges would be well advised also to eschew or modify instruction No. 4.35 in cases involving crimes requiring a specific intent or mental state.

4.5.12 Notes & Questions (People v. Navarro) 4.5.12 Notes & Questions (People v. Navarro)

Review Questions

Navarro was charged with “trespassory taking and carrying away of the personal property of another with the intent to steal the property.” 

  1. What is the mens rea in this statute?
  2. Is this a general intent or a specific intent crime? 
  3. What does the court mean when it says, “There is no such thing as larceny by negligence?” 
  4. Summarize the debate over the jury instructions in this case. What were the trial court's instructions on mistake? What happened on appeal? 
  5. What result if evidence showed that the D knew the beams belonged to another, but that, when he took them, he mistakenly thought they were oak, when actually they were pine?
  6. What result if D knew the beams were not abandoned, but thought it was legal to take things from construction sites? 
  7. What result had this case arisen in an MPC jurisdiction (applying  MPC 2.04(1))? 

Problem on Mistake

(1) Ignorance or mistake as to a matter of fact or law is a defense if:

(a) the ignorance or mistake negatives the purpose, knowledge, belief, recklessness or negligence required to establish a material element of the offense;...

Sara gives Dan a clear plastic bag with white powder in it and says, “Please deliver this bag to my 9th grade friend Frank when you see him at school today.” When Dan asks what it is, Sara says “It’s baby powder.” Dan sincerely believes it is baby powder. It’s cocaine. He’s charged with “intentional delivery of a controlled substance to a minor.”

  1. Assuming the jury believes D’s testimony that he thought it was baby powder, will D be acquitted of the crime? Why or why not? 

  2. Is D guilty if Sara tells D it’s legal for minors to deliver drugs to other minors, and after verifying D’s age is 15, has him take bag of cocaine to her 9th grade friend, Frank?

  3. Same facts as in B., except that Sara was the on-duty police officer assigned to Dan’s school. 

4.5.13 People v. Snyder 4.5.13 People v. Snyder

32 Cal.3d 590

People v. Snyder
32 Cal.3d 590
[Crim. No. 22293. Supreme Court of California. October 18, 1982.]

THE PEOPLE, Plaintiff and Respondent, v. NEVA B. SNYDER, Defendant and Appellant (Opinion by Richardson, J., with Bird, C. J., Mosk and Kaus, JJ., concurring. Separate dissenting opinion by Broussard, with Newman and Reynoso, JJ., concurring.)

OPINION RICHARDSON, J.

Defendant Neva Snyder appeals from a judgment convicting her of possession of a concealable firearm by a convicted felon (Pen. Code, § 12021), based upon her 1973 conviction for sale of marijuana, a felony (former Health & Saf. Code, § 11531). Defendant contends that the trial court erred in excluding evidence of her mistaken belief that her prior conviction was only a misdemeanor. We will conclude that defendant's asserted mistake regarding her legal status as a convicted [32 Cal. 3d 592] felon did not constitute a defense to the firearm possession charge. Accordingly, we will affirm the judgment.

At trial, defendant offered to prove the following facts supporting her theory of mistake: The marijuana possession charge resulted from a plea bargain not involving a jail or prison sentence. At the time the bargain was struck, defendant's attorney advised her that she was pleading guilty to a misdemeanor. Believing that she was not a felon, defendant thereafter had registered to vote, and had voted. On one prior occasion, police officers found a gun in her home but, after determining that it was registered to her husband, the officers filed no charges against defendant.

The trial court refused to admit any evidence of defendant's mistaken belief that her prior conviction was a misdemeanor and that she was not a felon. The court also rejected proposed instructions requiring proof of defendant's prior knowledge of her felony conviction as an element of the offense charged.

Penal Code section 12021, subdivision (a), provides: "Any person who has been convicted of a felony under the laws of the ... State of California ... who owns or has in his possession or under his custody or control any pistol, revolver, or other firearm capable of being concealed upon the person is guilty of a public offense, ..."

[1] The elements of the offense proscribed by section 12021 are conviction of a felony and ownership, possession, custody or control of a firearm capable of being concealed on the person. (People v. Bray (1975) 52 Cal. App. 3d 494, 497 [124 Cal. Rptr. 913]; People v. Neese (1969) 272 Cal. App. 2d 235, 245 [77 Cal. Rptr. 314]; People v. Nieto (1966) 247 Cal. App. 2d 364, 368 [55 Cal. Rptr. 546].) No specific criminal intent is required, and a general intent to commit the proscribed act is sufficient to sustain a conviction. (People v. Neese, supra, at p. 245; People v. McCullough (1963) 222 Cal. App. 2d 712, 718 [35 Cal. Rptr. 591].) With respect to the elements of possession or custody, it has been held that knowledge is an element of the offense. (People v. Burch (1961) 196 Cal. App. 2d 754, 770-771 [17 Cal. Rptr. 102]; People v. Gonzales (1925) 72 Cal. App. 626, 630-631 [237 P. 812].)

[2] Does section 12021 also require knowledge of one's legal status as a convicted felon? No case has so held. Penal Code section 26 provides that a person is incapable of committing a crime if he acted under a "mistake of fact" which disproves criminal intent. In this regard, the cases have distinguished between mistakes of fact and mistakes of law. As we stated in an early case: "It is an emphatic postulate of both civil and penal law that [32 Cal. 3d 593] ignorance of a law is no excuse for a violation thereof. Of course it is based on a fiction, because no man can know all the law, but it is a maxim which the law itself does not permit any one to gainsay .... The rule rests on public necessity; the welfare of society and the safety of the state depend upon its enforcement. If a person accused of a crime could shield himself behind the defense that he was ignorant of the law which he violated, immunity from punishment would in most cases result." (People v. O'Brien (1892) 96 Cal. 171, 176 [31 P. 45]; see Brown v. State Department of Health (1978) 86 Cal. App. 3d 548, 554-555 [150 Cal. Rptr. 344], and cases cited.) Accordingly, lack of actual knowledge of the provisions of Penal Code section 12021 is irrelevant; the crucial question is whether the defendant was aware that he was engaging in the conduct proscribed by that section. (People v. Norton (1978) 80 Cal. App. 3d Supp. 14, 21 [146 Cal. Rptr. 343]; People v. Howard (1976) 63 Cal. App. 3d 249, 256 [133 Cal. Rptr. 689]; People v. Mendoza (1967) 251 Cal. App. 2d 835, 843 [60 Cal. Rptr. 5].)

In the present case, defendant was presumed to know that it is unlawful for a convicted felon to possess a concealable firearm. (Pen. Code, § 12021.) She was also charged with knowledge that the offense of which she was convicted (former Health & Saf. Code, § 11531) was, as a matter of law, a felony. That section had prescribed a state prison term of from five years to life, and the express statutory definition of a "felony" is "a crime which is punishable with death or by imprisonment in the state prison." (Pen. Code, § 17, subd. (a).)

Thus, regardless of what she reasonably believed, or what her attorney may have told her, defendant was deemed to know under the law that she was a convicted felon forbidden to possess concealable firearms. Her asserted mistake regarding her correct legal status was a mistake of law, not fact. It does not constitute a defense to section 12021.

None of the California cases relied on by defendant is apposite here. People v. Hernandez (1964) 61 Cal. 2d 529 [39 Cal. Rptr. 361, 393 P.2d 673, 8 A.L.R.3d 1092], and People v. Mayberry (1975) 15 Cal. 3d 143 [125 Cal. Rptr. 745, 542 P.2d 1337], each involved mistakes of fact, not law. In Hernandez, the mistake concerned the age of the alleged victim of a statutory rape. In Mayberry, defendant erred in assuming that the adult victim of forcible rape consented to his acts. People v. Vogel (1956) 46 Cal. 2d 798 [299 P.2d 850], involved the good faith belief of a defendant charged with bigamy that he is free to remarry. We were careful to explain that defendant's mistake was a factual one: "We have concluded that defendant is not guilty of bigamy, if he had a bona fide and reasonable belief that facts existed that left him free to remarry." (Id., at p. 801; italics [32 Cal. 3d 594] added.) Moreover, Vogel characterized bigamy as a crime which "has been regarded for centuries as ... involving moral turpitude, ..." Obviously a bona fide belief that one is free to remarry nullifies the moral opprobrium attached to the charge. On the other hand, being an ex-felon in possession of a concealable firearm, while illegal, hardly stamps the person charged as a moral leper. His belief that he is not a felon thus does not affect the criminality of his conduct.

Our conclusion is confirmed by federal cases interpreting a similar federal statute forbidding possession of a firearm by one convicted of a felony. (18 U.S.C.A. Appen. § 1202(a).) The only element of the federal offense which is not found in section 12021 of the Penal Code is an effect upon "commerce." (See United States v. Bass (1971) 404 U.S. 336, 347-349 [30 L. Ed. 2d 488, 496-497, 92 S. Ct. 515].) The federal statute has been uniformly interpreted as requiring only that the defendant was in fact a convicted felon, and not that he actually knew he was a felon. (United States v. Locke (9th Cir. 1976) 542 F.2d 800, 801; United States v. Mathews (9th Cir. 1975) 518 F.2d 1296; United States v. Crow (9th Cir. 1971) 439 F.2d 1193, 1196.) As stated in Locke, "Because the crimes here charged do not require a specific intent [citation], the fact that appellant may have been advised by a public defender that he was not a convicted felon, has no relevance." (P. 801.)

Defendant relies primarily upon People v. Bray, supra, 52 Cal.App.3d at p. 494, but that case is distinguishable. There defendant pleaded guilty in Kansas to being an accessory before the fact and was placed on two years' summary probation, which he successfully completed. When he subsequently sought to register to vote, he filled out an explanatory form referring to a Kansas offense, and indicating that he was uncertain whether he had been convicted of a felony. He was permitted to vote. Seeking employment as a security guard, he stated that he had not been convicted of a felony but described the circumstances of his arrest and probation. The Bureau of Collection and Investigative Services registered him as a guard. On several other job applications he indicated his uncertainty as to his status while fully setting forth the circumstances of his arrest and probation.

In Bray, the court concluded that under these unusual circumstances the trial court erred in refusing to instruct on mistake or ignorance of fact and knowledge of the facts which make the act unlawful. (52 Cal.App.3d, at p. 499.) The court cautioned, however, that its decision "should not be interpreted to mean instructions on mistake or ignorance of fact and knowledge of the facts are required every time a defendant claims he did not know he was a felon .... It is only in very unusual circumstances such as these that the giving of these instructions is necessary." (Ibid.) [32 Cal. 3d 595]

In the present case, unlike Bray, defendant made no attempt to inform government officials of the circumstances of her conviction or to seek their advice regarding her correct legal status. (Some authorities have suggested that reliance upon the erroneous advice of governmental authorities might constitute an exception to the general rule that a mistake of law is no defense. See Perkins on Criminal Law (2d ed. 1969) p. 938; A.L.I. Model Pen. Code (Proposed Official Draft 1962) § 2.04(3)(b).)

We conclude that the trial court properly excluded evidence of defendant's asserted mistake regarding her status as a convicted felon.

The judgment is affirmed.

Bird, C. J., Mosk J. and Kaus, J. concurred.

BROUSSARD, J.

I dissent.

The two elements of a violation of Penal Code section 12021 are felony status and possession of a concealable firearm. While no specific criminal intent is required, a general criminal intent should be required as to both elements in accordance with long-settled rules of statutory interpretation, and an honest and reasonable mistake as to either element of the offense, however induced, should negate the requisite general criminal intent. Defendant's testimony if believed would have established an honest and reasonable mistaken belief that her prior offense was not a felony but a misdemeanor, and it was prejudicial error to refuse to admit the evidence and to refuse instructions on the mistake doctrine.

The majority have adopted a special strict liability rule as to section 12021, holding that a felon is charged with knowledge of his status and that an honest and reasonable mistaken belief as to the nature of the conviction is not a defense unless apparently it is induced or corroborated in whole or in part by governmental conduct. The traditional and longstanding defense of mistake negating criminal intent should not be limited to situations where the mistake is induced or corroborated by government officials. Irrebutable presumptions of knowledge are not favored in the criminal law, and because the source of an honest and reasonable mistake does not affect the question of the existence of criminal intent, we should not accept the government source limitation.

During a lawful search of defendant and her husband's home in 1979, officers found one loaded handgun and two other handguns which were partially disassembled. In 1973 she had been convicted upon a guilty plea of sale of marijuana, a felony. (Former Health & Saf. Code, § 11531.) [32 Cal. 3d 596]

Defendant sought to testify that she believed that her marijuana possession conviction had been for a misdemeanor rather than a felony. She offered to testify that she had not been sentenced to jail or prison but was on probation for two years, that her attorney told her at the time of the plea bargain that she was pleading guilty to a misdemeanor, and that believing she was not a felon she had since registered to vote and voted. She also offered to testify that on a prior occasion, officers found a pistol in her home but that after determining the gun was registered to her husband, no charges were filed for possession of the gun. Other charges were filed but dismissed. Her husband had also been convicted in 1973 of the same marijuana charge.

The trial court refused to admit the evidence of defendant's mistaken belief that the prior conviction was a misdemeanor and that she was not a felon. The court also rejected offered instructions to require knowledge of a prior felony conviction as an element of the offense, and to define "knowingly," to explain the effect of ignorance or mistake of fact disproving criminal intent. (CALJIC Nos. 1.21, 4.35, 3.31.5 [paraphrased].) fn. 1 The instructions, if given, would have required the jury to find that defendant knew she was a felon as an element of the crime.

Penal Code section 12021, subdivision (a) provides: "Any person who has been convicted of a felony under the laws of the ... State of California ... who owns or has in his possession or under his custody or control any pistol, revolver, or other firearm capable of being concealed upon the person is guilty of a public offense, ..."

We recently recognized our duty to reconcile the provisions of section 12021 with other Penal Code provisions in People v. King (1978) 22 [32 Cal. 3d 597] Cal.3d 12, 23 [148 Cal. Rptr. 409, 582 P.2d 1000]. In that case, a felon during a melee involving violence by intruders took a pistol and fired it. We held that it was error to refuse to instruct the jury on self-defense and defense of others. (22 Cal.3d at p. 26.) We reasoned that the section 12021 prohibition of possession by a felon of a concealable firearm must be reconciled with the statutory provisions of the Penal and Civil Codes permitting any person the right to use force in defense of self or others. (22 Cal.3d at p. 23.)

In the instant case we are similarly called upon to reconcile section 12021 with Penal Code sections 26 and 20.

Penal Code section 26 provides generally that a person is incapable of committing a crime when the act was committed under a mistake of fact disproving any criminal intent. Section 20 of that code provides: "In every crime or public offense there must exist a union, or joint operation of act and intent, or criminal negligence.

"The word 'intent' in section 20 means 'wrongful intent.' (See People v. Vogel (1956) 46 Cal. 2d 798, 801, fn. 2 [299 P.2d 850].) 'So basic is this requirement [of a union of act and wrongful intent] that it is an invariable element of every crime unless excluded expressly or by necessary implication.' (Id., at p. 801.)" (People v. Mayberry (1975) 15 Cal. 3d 143, 154 [125 Cal. Rptr. 745, 542 P.2d 1337].)

At common law an honest and reasonable belief in circumstances which, if true, would make the defendant's conduct innocent was held to be a good defense. (People v. Hernandez (1964) 61 Cal. 2d 529, 535 [39 Cal. Rptr. 361, 393 P.2d 673, 8 A.L.R.3d 1092].) The concept of mens rea, the guilty mind, expresses the principle that it is not conduct alone but conduct accompanied by certain mental states which concerns, or should concern the law. While in some cases, culpability had been completely eliminated as a necessary element of criminal conduct, the court has moved away from imposition of criminal liability in the absence of culpability where the governing statute, by implication or otherwise, expresses no legislative intent or policy to be served by strict liability. (Id., at pp. 532-533.)

The elements of the offense proscribed by section 12021 are conviction of a felony and ownership, possession, custody or control of a firearm capable of being concealed on the person. (People v. Bray (1975) 52 Cal. App. 3d 494, 497 [124 Cal. Rptr. 913]; People v. Neese (1969) 272 Cal. App. 2d 235, 245 [77 Cal. Rptr. 314]; People v. Nieto (1966) 247 Cal. App. 2d 364, 368 [55 Cal. Rptr. 546].) While no specific criminal intent is required, a general intent to commit the proscribed act is [32 Cal. 3d 598] necessary. (People v. Neese, supra, 272 Cal. App. 2d 235, 245; People v. McCullough (1963) 222 Cal. App. 2d 712, 718 [35 Cal. Rptr. 591].) As to the element of possession or custody, it has been held that knowledge is an element of the offense. (People v. Burch (1961) 196 Cal. App. 2d 754, 770-771 [17 Cal. Rptr. 102]; People v. Gonzales (1925) 72 Cal. App. 626, 630-631 [237 P. 812].)

There does not appear to be any provision in section 12021 by implication or otherwise indicating legislative intent or policy to be served by refusing to apply the general criminal intent requirement to both of the elements of the offense. The Attorney General argues that, because knowledge of possession or custody of the gun are essential to conviction, strict liability would not result from holding that knowledge of a felony conviction is irrelevant. While under such construction the offense would not involve strict liability as to those who were unaware that they possessed a handgun, it would impose strict liability upon those who were unaware of their felony convictions and could legally possess handguns in the absence of conviction. The language of section 12021 sets forth both elements of the offense in parallel construction, and there is no basis in the language or grammatical construction of the statute warranting a distinction between the two elements with respect to the mens rea requirement. In the absence of any provision reflecting legislative intent or policy to establish strict liability, the mens rea requirement is applicable to the felony conviction element of the offense as well as the possession and custody element. (People v. Bray, supra, 52 Cal. App. 3d 494, 498-499; cf. People v. Mendoza (1967) 251 Cal. App. 2d 835, 843 [60 Cal. Rptr. 5].)

To hold otherwise is contrary to the settled California rule that a mens rea requirement is an "invariable" element of every crime unless excluded expressly or by necessary implication. People v. Mayberry, supra, 15 Cal. 3d 143, 154; People v. Vogel, supra, 46 Cal. 2d 798, 801.) Having established the rule, we must assume the Legislature is aware of it and acting in accordance with it, and the absence of any provision to establish strict liability must be read as reflecting legislative intent to require wrongful intent.

The majority rely upon federal cases interpreting a federal statute forbidding possession of a firearm by one convicted of a felony or certain misdemeanors. (18 U.S.C. Appen. § 1202(a)(1); United States v. Locke (9th Cir. 1976) 542 F.2d 800, 801; United States v. Mathews (9th Cir. 1975) 518 F.2d 1296; United States v. Crow (9th Cir. 1971) 439 F.2d 1193, 1196.) In particular they rely upon the following statement from Locke: "Because the crimes here charged do not require specific intent, United States v. Quiroz, 449 F.2d 583, 585 (CA9 1971), the fact that [32 Cal. 3d 599] appellant may have been advised by a public defender that he was not a convicted felon, has no relevance." (542 F.2d at p. 801.) (Maj. opn., ante, p. 595.)

None of the federal cases cite or discuss statutes comparable to Penal Code sections 20 and 26. Concluding that there is no scienter or mens rea requirement, the courts reason that the crime is a statutory offense rather than a common law offense and that as to statutory offenses there ordinarily is no scienter requirement in the absence of express provision therefor. (United States v. Crow, supra, 439 F.2d 1193; United States v. Quiroz, supra, 449 F.2d 583, 585.) Penal Code sections 20 and 26 are part of our statutory law, and there is no basis for a conclusion that they apply only to common law offenses and not to statutory offenses. Because of those statutes, we have adopted a rule of construction directly contrary to the federal rule, i.e., wrongful intent "is an invariable element of every crime unless excluded expressly or by necessary implication." (People v. Vogel, supra, 46 Cal. 2d 798, 801.) Just as the federal courts adhere to their rules as to the effect of congressional silence, we should adhere to the legislative direction in sections 20 and 26 and our rules as to legislative silence. Having established the ground rules for statutory interpretation, we may not abrogate them on the basis of federal cases applying contrary ground rules.

In determining whether a defendant's mistaken belief disproves criminal intent pursuant to Penal Code section 26, the courts have drawn a distinction between mistakes of fact and mistakes of law. Criminal intent is the intent to do the prohibited act, not the intent to violate the law. (1 Witkin, Cal. Crimes (1963) § 148, p. 141.) "It is an emphatic postulate of both civil and penal law that ignorance of a law is no excuse for a violation thereof. Of course it is based on a fiction, because no man can know all the law, but it is a maxim which the law itself does not permit any one to gainsay .... The rule rests on public necessity; the welfare of society and the safety of the state depend upon its enforcement. If a person accused of a crime could shield himself behind the defense that he was ignorant of the law which he violated, immunity from punishment would in most cases result." (People v. O'Brien (1892) 96 Cal. 171, 176 [31 P. 45].) Accordingly, lack of knowledge of the provisions of Penal Code section 12021 is irrelevant; the crucial question is whether the defendant was aware that he was engaging in the conduct proscribed by that section. (People v. Howard (1976) 63 Cal. App. 3d 249, 256 [133 Cal. Rptr. 689]; People v. Mendoza, supra, 251 Cal. App. 2d 835, 843.)

While mistake as to whether the conduct is violative of a statute is not a defense, a mistaken impression as to the legal effect of a collateral matter [32 Cal. 3d 600] may mean that a defendant does not understand the significance of his conduct and may negate criminal intent. When the victim's status is an element of the crime, a mistaken belief as to the status has been held a defense in several decisions by this court. In People v. Hernandez, supra, 61 Cal. 2d 529, 532 et seq., it was held that a reasonable and honest belief that the prosecutrix was 18 years or more of age would be a defense to a charge of statutory rape, negating the requisite mental intent. Similarly, in People v. Atchison (1978) 22 Cal. 3d 181, 183 [148 Cal. Rptr. 881, 583 P.2d 735], it was held that a reasonable and honest belief that the victim was 19 years of age was a defense to charges of annoying or molesting a child under age 18 and of contributing to the delinquency of a minor. And in People v. Mayberry, supra, 15 Cal. 3d 143, 153-158, it was held that a mistaken belief that the prosecutrix had consented would be a defense to a charge of forcible rape and kidnaping.

This court has also held that criminal intent may be negated by defendant's reasonable and bona fide but erroneous belief as to his status. In People v. Vogel, supra, 46 Cal. 2d 798, 801 et seq., the defendant was prosecuted for bigamy, and it was held that the defendant's bona fide and reasonable belief that his first wife had divorced him and remarried would be a good defense. The court reasoned in part that it would not be reasonable to hold "that a person is guilty of bigamy who remarries in good faith in reliance on a judgment of divorce or annulment that is subsequently found not to be the 'judgment of a competent court' (Pen. Code, § 282), particularly when such a judgment is obtained by the former husband or wife of such person in any one of the numerous jurisdictions in which such judgments can be obtained. Since it is often difficult for laymen to know when a judgment is not that of a competent court, we cannot reasonably expect them always to have such knowledge and make them criminals if their bona fide belief proves to be erroneous." (46 Cal.2d, at p. 804; see People v. Roessler (1963) 217 Cal. App. 2d 603, 604 [31 Cal. Rptr. 684] (defendant remarried after service of the interlocutory decree of divorce but before entry of final decree).) The court also pointed out that at common law an honest and reasonable belief in circumstances which, if true, would make the act for which the person is indicted an innocent act, has always been held to be a good defense. (46 Cal.2d, at p. 805.)

People v. Flumerfelt (1939) 35 Cal. App. 2d 495, 497-498 [96 P.2d 190], also illustrates the distinction between mistake of fact and mistake of law. In that case, the defendant was charged with selling corporate securities without a permit. The defendant claimed that before she sold the securities, her attorney told her that a permit to sell had been obtained, and it was held that her honest but mistaken belief that a permit to sell had been [32 Cal. 3d 601] issued constituted a defense. However, the court distinguished the situation where counsel erroneously advises that the instrument to be sold is not a security, pointing out that a mistake as to the legal consequences of the act which constitutes a violation of the statute would not be a defense. (35 Cal.App.2d at pp. 498-499; see 1 Witkin, Cal. Crimes, supra, § 151, p. 145.)

The O'Brien, Hernandez, Atchison, Mayberry, Vogel and Flumerfelt cases, read together, make clear that a mistake of law is one premised on ignorance of the terms of the statute which the defendant is charged with violating. However, when the defendant reasonably and honestly believes that the statute is not applicable to him or that he has complied with it, there is a mistake of fact. There is a mistake of fact even though the matter as to which the defendant is mistaken is a question of law. The questions of age in Hernandez and Atchison were matters resolved as a matter of law as was marital status in Vogel and the nonissuance of a permit in Flumerfelt.

The Court of Appeal has held that a mistaken belief that a conviction was not a felony conviction could negate criminal intent in a prosecution for violation of Penal Code section 12021. (People v. Bray, supra, 52 Cal. App. 3d 494, 497-499.) Bray graphically illustrates the injustice which results from holding that a reasonable and good faith belief of lack of felony status is not a defense. Bray pled guilty in Kansas to being an accessory before the fact and was placed on two years summary probation which he successfully completed. When he subsequently sought to register to vote, he filled out an explanatory form referring to the Kansas offense, indicating he was uncertain whether he had been convicted of a felony. He was permitted to vote. Seeking employment as a security guard, he stated he had not been convicted of a felony but set forth the circumstances of his arrest and probation. The Bureau of Collection and Investigative Services registered him as a guard. On several other job applications he indicated his uncertainty as to his status setting forth the circumstances of his arrest and probation. At Bray's trial for violation of section 12021, the prosecutor recognized "'in even our own jurisdiction, let alone a foreign jurisdiction such as the State of Kansas, it's extremely difficult to determine whether a sentence was a felony or a misdemeanor.'" (52 Cal.App.3d, at p. 498.)

In Bray, it was concluded that in the circumstances the trial court erred in refusing to instruct on mistake or ignorance of fact and knowledge of the facts which make the act unlawful. (52 Cal.App.3d, at p. 499.)

The Court of Appeal stated that its decision should not be interpreted to mean instructions on mistake or ignorance of fact or knowledge of the facts are required every time a defendant claims he did not know he was a felon. [32 Cal. 3d 602](Id.) Relying on that statement, the majority concludes that Bray should be limited to situations where a state agency has misled the defendant. The statement relied upon merely reflects that only in rare cases will there be a basis for a reasonable belief that a felony conviction was a misdemeanor conviction. The reasoning in Bray applies to any case where there is a reasonable and good faith mistake and is in accord with the common law and our statutory rule. The source of the reasonable and good faith mistake does not affect the existence of criminal intent. fn. 2

Had the trial court in the instant case admitted the offered evidence and given the requested instruction, the jury could properly have concluded that defendant had a reasonable and good faith belief that her conviction was not a felony conviction. She was granted probation without jail or prison sentence. Her attorney had advised her that the offense was a misdemeanor, fn. 3 and there were additional circumstances reflecting a good faith belief.

The errors in excluding the offered evidence and refusing the offered instructions denied defendant the right to have the jury determine substantial issues material to her guilt and require reversal of the conviction. (People v. King (1978) 22 Cal. 3d 12, 27 [148 Cal. Rptr. 409, 582 P.2d 1000]; People v. Mayberry, supra, 15 Cal. 3d 143, 157-158.)

I would reverse the judgment.

Newman, J., and Reynoso, J., concurred.

FN 1. CALJIC No. 1.21: "'Knowingly -- Defined

"The word 'knowingly', as used in my instructions, imports only a knowledge of the existence of the facts in question, when those facts are such as bring the act or omission within the provision of the law. The word does not require in its meaning any knowledge of the unlawfulness of such act or omission."

CALJIC No. 4.35: "Ignorance or Mistake of Fact

"An act committed or an omission made under an ignorance or mistake of fact which disproves any criminal intent is not a crime. [¶] Thus a person is not guilty of a crime if he commits an act or omits to act under an honest and reasonable belief in the existence of certain facts and circumstances which, if true, would make such act or omission unlawful."

CALJIC No. 3.31.5, as paraphrased: "In violation of the crime charged in Section 12021 of the Penal Code the information namely, the possession of a concealed weapon by an ex-felon, there must exist a union or joint operation of act or conduct and a certain mental state in the mind of the perpetrator and unless such mental state exists the crime to which it relates is not committed. [¶] In the crime of 12021 of the Penal Code, the necessary mental state is knowledge on the part of the Defendant that he has been previously convicted of a felony offense."

FN 2. I am perplexed by the majority's apparent limitation of the mistake doctrine to would-be "moral leper[s]." The more heinous the crime the more reason to limit defenses, and the majority's suggested limitation appears to turn the usual relationship between law and morality upside down.

FN 3. It has been held that advice of counsel that prohibited conduct is lawful is not a defense because it would place the advice of counsel above the law. (See 1 Witkin, Cal. Crimes, supra, § 150, pp. 143-144.) Counsel's advice in the instant case is relevant to establish good faith; it does not in and of itself establish a defense.

4.5.14 Notes & Questions (People v. Snyder) 4.5.14 Notes & Questions (People v. Snyder)

Mistake of Law

Review Questions

The following is the operative statute in People v. Snyder

CA Penal Code section 12021, subdivision (a): "Any person who has been convicted of a felony under the laws of the ... State of California ... who owns or has in his possession or under his custody or control any pistol, revolver, or other firearm capable of being concealed upon the person is guilty of a public offense...."

  1. What does the Court identify as the material elements in this statute? 
  2. What result if the defendant didn’t know she had a gun in her purse?
  3. What is the defendant's mistake in this case, and what was the trial court's rationale for excluding her testimony regarding it? 
  4. The Court understands the defendant's mistake as a mistake of law. Why? Might you argue that her mistake was actually a mistake of fact? 
  5. How, do you think, the defense lawyer may have tried to use People v. Vogel, cited in the opinion, to support their case?
  6. How does the majority distinguish People v. Vogel here? 
  7. Explain the dissent’s position. (Consider how the majority concedes that prior, binding precedent interprets “possession” as requiring the state to prove knowledge.) 
  8. What result if, rather than relying on her lawyer’s declaration that she had been convicted only of a misdemeanor, the defendant could show that it was her judge who had told her it was a misdemeanor?
  9. Finally, what result in People v. Snyder if it arose in MPC land?
    (HINT: remember to apply all of the MPC rules for interpreting statutes lacking mens rea. Do you have enough information to answer this question? See below).

MPC 2.04 (1) Ignorance or mistake as to a matter of fact or law is a defense if:

(a) the ignorance or mistake negatives the purpose, knowledge, belief, recklessness or negligence required to establish a material element of the offense; ...

“MPC” Penal Code section 12021(a): "Any person who has been convicted of a felony under the laws of the ... Model Penal Code Land ... who owns or has in his possession or under his custody or control any pistol, revolver, or other firearm capable of being concealed upon the person is guilty of a public offense...."